Module 6 - Chapter 12: Oncologic Management

Réussis tes devoirs et examens dès maintenant avec Quizwiz!

A patient with metastatic pancreatic cancer underwent surgery for removal of a malignant tumor in his pancreas. Despite the tumor being removed, the physician informs the patient that he needs to start chemotherapy. Which of the following may be a reason for the physician to opt for chemotherapy?

Metastasis

Which type of vaccine uses the patient's own cancer cells, which are prepared for injection, back into the patient?

Autologous

A nurse is caring for a client receiving chemotherapy. Which assessment finding places the client at the greatest risk for an infection?

Stage 3 pressure ulcer on the left heel A stage 3 pressure ulcer is a break in the skin's protective barrier, which could lead to infection in a client who is receiving chemotherapy. The WBC count and temperature are within normal limits. Eating 75% of meals is normal and doesn't increase the client's risk for infection. A client who is malnourished is at a greater risk for infection.

The lethal tumor dose is defined as the dose that will eradicate what percentage of the tumor yet preserve normal tissue?

95%

A benign tumor of the blood vessels is a(n)

hemangioma. A hemangioma is a benign tumor of the blood vessels. An osteoma is a tumor of the connective tissue. A neuroma is a tumor of the nerve cells. A chondroma is a tumor of the cartilage.

The nurse is caring for a patient undergoing an incisional biopsy. Which of the following statements does the nurse understand is true about an incisional biopsy?

It removes a wedge of tissue for diagnosis.

Which statement by a client undergoing external radiation therapy indicates the need for further teaching?

"I'm worried I'll expose my family members to radiation." The client undergoing external radiation therapy requires further teaching when he voices a concern that he might expose his family to radiation. Internal radiation, not external radiation, poses a risk to the client's family. The client requires no further teaching if he states that he should wash his skin with mild soap and water, wear protective clothing when outside, and avoid using a heating pad.

A patient is scheduled for cryosurgery for cervical cancer and tells the nurse, "I am not exactly sure what the doctor is going to do." What is the best response by the nurse?

"The physician is going to use liquid nitrogen to freeze the area."

A nurse is performing a home visit for a client who received chemotherapy within the past 24 hours. The nurse observes a small child playing in the bathroom, where the toilet lid has been left up. Based on these observations, the nurse modifies the client's teaching plan to include:

chemotherapy exposure and risk factors. The raised toilet lid exposes the child playing in the bathroom to the risk of inhaling or ingesting chemotherapy agents. The nurse should modify her teaching plan to include content related to chemotherapy exposure and its associated risk factors. Because the client has received chemotherapy, the plan should already include information about expected adverse effects, signs and symptoms of infection, and reinforcement of the medication regimen.

A patient is scheduled for cryosurgery for cervical cancer and tells the nurse, "I am not exactly sure what the doctor is going to do." What is the best response by the nurse?

"The physician is going to use liquid nitrogen to freeze the area." Cryoablation, or cryosurgery, is the use of liquid nitrogen or a very cold probe to freeze tissue and cause cell destruction. It is used for cervical, prostate, and rectal cancers.

A client with a brain tumor is undergoing radiation and chemotherapy for treatment of cancer. The client has recently reported swelling in the gums, tongue, and lips. Which is the most likely cause of these symptoms?

Stomatitis The symptoms of swelling in gums, tongue, and lips indicate stomatitis. This usually occurs 5 to 10 days after the administration of certain chemotherapeutic agents or radiation therapy to the head and neck. Chemotherapy and radiation produce chemical toxins that lead to the breakdown of cells in the mucosa of the epithelium, connective tissue, and blood vessels in the oral cavity.

Your client is receiving radiation therapy. The client asks you about oral hygiene. What advice regarding oral hygiene should you offer?

Use a soft toothbrush and avoid an electronic toothbrush. The nurse advises the client undergoing radiation therapy to use a soft toothbrush and avoid electronic toothbrushes to avoid skin lacerations. Gargling after each meal, flossing before going to bed, and treating cavities immediately are general oral hygiene instructions.

Which should a nurse thoroughly evaluate before a bone marrow transplant (BMT) procedure?

Blood studies

Palliation refers to

Relief of symptoms of disease and promotion of comfort and quality of life

After being seen in the oncology clinic, a client with severe bone marrow suppression is admitted to the hospital. The client's cancer therapy consisted of radiation and chemotherapy. When developing the care plan for this client, which nursing diagnosis takes priority?

Risk for infection

A decrease in circulating white blood cells is 1. thrombocytopenia. 2. leukopenia. 3. neutropenia. 4. granulocytopenia.

2

A serum sodium concentration lower than 115 mEq/L (115 mmol/L) is associated with:

Seizures

A side-effect of chemotherapy is renal damage. To prevent this, the nurse should:

Encourage fluid intake, if possible, to dilute the urine.

The clinic nurse is caring for a client who has just been diagnosed with a tumor. The client says to the nurse "The doctor says my tumor is benign. What does that mean?" What is the nurse's best response?

"Benign tumors don't usually cause death."

The clinic nurse is caring for a client who has just been diagnosed with a tumor. The client says to the nurse "The doctor says my tumor is benign. What does that mean?" What is the nurse's best response?

"Benign tumors don't usually cause death." -Benign tumors remain at their site of development. They may grow large, but their growth rate is slower than that of malignant tumors. They usually do not cause death unless their location impairs the function of a vital organ, such as the brain.

While doing a health history, a client tells the nurse that her mother, her grandmother, and her sister died of breast cancer. The client asks what she can do to keep from getting cancer. What is the best response by the nurse?

"Cancer prevention and detection can be done with blood analysis for tumor markers to measure your risk level."

A client's family member asks the nurse why disseminated intravascular coagulation (DIC) occurs. Which statement by the nurse correctly explains the cause of DIC?

"DIC is caused by abnormal activation of the clotting pathway, causing excessive amounts of tiny clots to form inside organs."

A patient with cancer who developed neutropenia several days ago has consequently been placed in a single-bed room that has positive pressure. His daughter has just come to visit her father after arriving from her home in another state and has asked you for his room number. You notice that the daughter has reddened eyes, sniffles, and a dry cough. What instruction should you provide to the daughter?

"Even though it might be difficult, it's best for your father's health if you get well before visiting him in person, since he's so vulnerable right now."

A nurse is caring for a client who is receiving chemotherapy and has a platelet count of 30,000/mm3. Which statement by the client indicates a need for additional teaching?

"I floss my teeth every morning."

A nurse is caring for a client who is receiving chemotherapy and has a platelet count of 30,000/mm3. Which statement by the client indicates a need for additional teaching? "I use an electric razor to shave." "I floss my teeth every morning." "I removed all the throw rugs from the house." "I take a stool softener every morning."

"I floss my teeth every morning."

A nurse is caring for a client who is receiving chemotherapy and has a platelet count of 30,000/mm3. Which statement by the client indicates a need for additional teaching? "I use an electric razor to shave." "I take a stool softener every morning." "I floss my teeth every morning." "I removed all the throw rugs from the house."

"I floss my teeth every morning."

A nurse is caring for a client who is receiving chemotherapy and has a platelet count of 30,000/mm3. Which statement by the client indicates a need for additional teaching?

"I floss my teeth every morning." A client with a platelet count of 30,000/mm3 is at risk for bleeding and shouldn't floss his teeth. Flossing may increase the risk of bleeding in a client with a platelet count less than 40,000/mm3. Using an electric razor is appropriate because doing so helps minimize the risk of cutting when shaving. Taking a stool softener helps decrease potential trauma to the GI tract that may cause bleeding. Removing throw rugs from the house helps prevent falls, which could lead to uncontrolled bleeding.

The nurse is collecting data for a client who has been diagnosed with iron-deficiency anemia. What subjective findings does the nurse recognize as symptoms related to this type of anemia?

"I have difficulty breathing when walking 30 feet."

A client receiving chemotherapy has a nursing diagnosis of Deficient diversional activity related to decreased energy. Which client statement indicates an accurate understanding of appropriate ways to deal with this deficit?

"I'll play card games with my friends."

A client receiving chemotherapy has a nursing diagnosis of Deficient diversional activity related to decreased energy. Which client statement indicates an accurate understanding of appropriate ways to deal with this deficit?

"I'll play card games with my friends." During chemotherapy, playing cards is an appropriate diversional activity because it doesn't require a great deal of energy. To conserve energy, the client should avoid such activities as taking long trips, bowling, and eating in restaurants every day. However, the client may take occasional short trips and dine out on special occasions.

A client receiving chemotherapy has a nursing diagnosis of Deficient diversional activity related to decreased energy. Which client statement indicates an accurate understanding of appropriate ways to deal with this deficit? a) "I'll play card games with my friends." b) "I'll eat lunch in a restaurant every day." c) "I'll bowl with my team after discharge." d) "I'll take a long trip to visit my aunt."

"I'll play card games with my friends." During chemotherapy, playing cards is an appropriate diversional activity because it doesn't require a great deal of energy. To conserve energy, the client should avoid such activities as taking long trips, bowling, and eating in restaurants every day. However, the client may take occasional short trips and dine out on special occasions.

A female client with the beta-thalassemia trait plans to marry a man of Italian ancestry who also has the trait. Which client statement indicates that she understands the teaching provided by the nurse?

"I'll see a genetic counselor before starting a family."

A nurse is teaching a client who is receiving radiation treatment for left lower lobe lung cancer. Which client statement indicates a need for further teaching? "I'll use hats to protect my head from the sun when my hair falls out." "If I get nauseous, I'll try to eat several small, bland meals each day." "I'll allow myself plenty of time to rest between activities." "Most of the adverse effects should go away shortly after my last radiation treatment."

"I'll use hats to protect my head from the sun when my hair falls out."

Which statement by a client undergoing external radiation therapy indicates the need for further teaching?

"I'm worried I'll expose my family members to radiation."

Which statement by a client undergoing external radiation therapy indicates the need for further teaching? "I'll wash my skin with mild soap and water only." "I'm worried I'll expose my family members to radiation." "I'll wear protective clothing when outside." "I'll not use my heating pad during my treatment."

"I'm worried I'll expose my family members to radiation."

Which statement by a client undergoing external radiation therapy indicates the need for further teaching? a) "I'll not use my heating pad during my treatment." b) "I'll wear protective clothing when outside." c) "I'll wash my skin with mild soap and water only." d) "I'm worried I'll expose my family members to radiation."

"I'm worried I'll expose my family members to radiation." Explanation: The client undergoing external radiation therapy requires further teaching when he voices a concern that he might expose his family to radiation. Internal radiation, not external radiation, poses a risk to the client's family. The client requires no further teaching if he states that he should wash his skin with mild soap and water, wear protective clothing when outside, and avoid using a heating pad.

Which statement by a client undergoing external radiation therapy indicates the need for further teaching? a) "I'll wash my skin with mild soap and water only." b) "I'll wear protective clothing when outside." c) "I'll not use my heating pad during my treatment." d) "I'm worried I'll expose my family members to radiation."

"I'm worried I'll expose my family members to radiation." Explanation: The client undergoing external radiation therapy requires further teaching when he voices a concern that he might expose his family to radiation. Internal radiation, not external radiation, poses a risk to the client's family. The client requires no further teaching if he states that he should wash his skin with mild soap and water, wear protective clothing when outside, and avoid using a heating pad.

A patient will be receiving radiation for 6 weeks for the treatment of breast cancer and asks the nurse why it takes so long. What is the best response by the nurse?

"It will allow time for the repair of healthy tissue."

The nurse is caring for a client with the diagnosis of colon cancer with metastasis to the liver. Which statement made by the client indicates an understanding of the diagnosis?

"My cancer has now spread to my liver." The client stating the cancer has spread to the liver shows that the client has an understanding the primary cancer of the colon with spread to the liver. The client stating that they will be fine after the surgery does not address the metastasis. The client stating they will be happy after the surgery is incorrect because metastases are not always resectable. The client stating having cancer twice is incorrect because it shows a lack of understanding about what metastasis is.

You are caring for a client who has just been told that they have stage IV colon cancer. The client asks you what "stage IV" means. What would be your best response?

"Stage IV means that the cancer has spread to other organs of the body."

The nurse is caring for a client who is scheduled for chemotherapy. Which is the best statement the nurse can make about the client experiencing chemotherapy-induced alopecia?

"The hair loss is temporary."

The nurse is caring for a client who is scheduled for chemotherapy. Which is the best statement the nurse can make about the client experiencing chemotherapy-induced alopecia?

"The hair loss is usually temporary."

The nurse is caring for a client who is scheduled for chemotherapy. Which is the best statement the nurse can make about the client experiencing chemotherapy-induced alopecia? "New hair growth will return without any change to color or texture." "Wigs can be used after the chemotherapy is completed." "Clients with alopecia will have delay in grey hair." "The hair loss is usually temporary."

"The hair loss is usually temporary."

The nurse is caring for a client who is scheduled for chemotherapy. Which is the best statement the nurse can make about the client experiencing chemotherapy-induced alopecia? "New hair growth will return without any change to color or texture." "The hair loss is usually temporary." "Wigs can be used after the chemotherapy is completed." "Clients with alopecia will have delay in grey hair."

"The hair loss is usually temporary."

A patient, age 67 years, is admitted for diagnostic studies to rule out cancer. The patient is Caucasian, married, has been employed as a landscaper for 40 years, and has a 36-year history of smoking a pack of cigarettes daily. What significant risk factors does the nurse recognize this patient has? (Select all that apply.)

- Age - Cigarette smoking - Occupation

The nurse is caring for a client who is scheduled for chemotherapy. Which is the best statement the nurse can make about the client experiencing chemotherapy-induced alopecia?

"The hair loss is usually temporary." Alopecia associated with chemotherapy is usually temporary and will return after the therapy is completed. New hair growth may return unchanged, but there is no guarantee and color, texture, and quality of hair may be changed. There is no correlation between chemotherapy and delay in greying of hair. Use of wigs, scarves, and head coverings can be used by clients at any time during treatment plan.

A bowel resection is scheduled for a client with the diagnosis of colon cancer with metastasis to the liver and bone. Which statement by the nurse best explains the purpose of the surgery?

"Tumor removal will promote comfort."

A bowel resection is scheduled for a client with the diagnosis of colon cancer with metastasis to the liver and bone. Which statement by the nurse best explains the purpose of the surgery? "This surgery will prevent further tumor growth." "Tumor removal will promote comfort." "Once the tumor is removed, cell pathology can be determined." "Removing the tumor is a primary treatment for colon cancer."

"Tumor removal will promote comfort."

A client with advanced cancer makes the following comment to the nurse: "Why are you bathing me? I am going to die no matter what." What is the most appropriate response of the nurse?

"Would you like to talk about what you are feeling?" By asking the client talk may open the door for further discussion and sharing of feelings, fears, etc. A bath will make the client feel better and asking if the client wishes to skip the bath today are matter-of-fact comments and disconnect, resulting in a shutdown to further communication. The nurse stating that medication could be given is a quick fix and demonstrates a nontherapeutic response.

While doing a health history, a client tells the nurse that her mother, grandmother, and sister died of cancer. The client wants to know what she can do to keep from getting cancer. What would be the nurse's best response?

"You can't prevent cancer, but you can have your blood analyzed for tumor markers to see what your risk level is."

While doing a health history, a client tells you that her mother, her grandmother, and her sister died of breast cancer. The client wants to know what she can do to keep from getting cancer. What would be your best response?

"You can't prevent cancer, but you can have your blood analyzed for tumor markers to see what your risk level is."

While doing a health history, a client tells you that her mother, grandmother, and sister died of cancer. The client wants to know what she can do to keep from getting cancer. What would be your best response? a) "With your family history, there is nothing you can do to prevent getting cancer." b) "Cancer often skips a generation, so don't worry about it." c) "If you eat right, exercise, and get enough rest, you can prevent breast cancer." d) "You can't prevent cancer, but you can have your blood analyzed for tumor markers to see what your risk level is."

"You can't prevent cancer, but you can have your blood analyzed for tumor markers to see what your risk level is." Specialized tests have been developed for tumor markers, specific proteins, antigens, hormones, genes, or enzymes that cancer cells release. Options B and C are incorrect, and giving the client these responses would be giving inaccurate information. Options D is incorrect because it minimizes and negates the clients concern.

While doing a health history, a client tells the nurse that her mother, grandmother, and sister died of cancer. The client wants to know what she can do to keep from getting cancer. What would be the nurse's best response?

"You can't prevent cancer, but you can have your blood analyzed for tumor markers to see what your risk level is." Specialized tests have been developed for tumor markers, specific proteins, antigens, hormones, genes, or enzymes that cancer cells release. Telling the client to make lifestyle changes and that there is nothing the client can do with their family history would be giving inaccurate information. Telling the client that cancer often skips a generation and not to worry is incorrect because it minimizes and negates the clients concern.

A young female client has received chemotherapeutic medications and asks about any effects the treatments will have related to her sexual health. The most appropriate statement by the nurse is

"You will need to practice birth control measures."

A young female client has received chemotherapeutic medications and asks about any effects the treatments will have related to her sexual health. The most appropriate statement by the nurse is

"You will need to practice birth control measures." Following chemotherapy female clients may experience normal ovulation, early menopause, or permanent sterility. Clients are advised to use reliable methods of birth control until reproductivity is known.

A client has been receiving chemotherapy. Upon assessing the client during morning rounds, the nurse notes the client is now bleeding from intravenous and venipuncture sites. Stool is positive for occult blood. The client is requesting to sit in a chair for a meal. The nurse implements the following interventions: (Select all that apply.)

- Assess level of consciousness. - Apply pressure to the bleeding sites. - Check intake and output records.

The root cause of cancer is damage to cellular deoxyribonucleic acid (DNA) which can be caused by many factors, or carcinogens. What factors can be carcinogenic? Select all that apply.

- dietary substances - environmental factors - viruses

A client with cancer is receiving chemotherapy and reports to the nurse that his mouth is painful and he has difficulty ingesting food. The nurse does which of the following:

-Asks the client to open his mouth to facilitate inspection of the oral mucosa -Instructs the client to brush the teeth with a soft toothbrush -Consults with the healthcare provider about use of nystatin (Mycostatin) The description of the client's report is stomatitis following chemotherapy treatment. The nurse should assess the oral mucosa based on the client's report of pain and difficulty eating. The client is to use a soft toothbrush to minimize trauma to the mouth. Nystatin is a topical medication that may provide healing for the client's mouth. The client avoids alcohol-based mouthwashes as these are irritants. Flossing the teeth may cause additional trauma to the mouth.

You are teaching clients about cancer prevention and explain that the root cause of cancer is damage to cellular deoxyribonucleic acid (DNA) as a result from multiple factors. You explain that which of the following are possible carcinogens? Select all that apply. -Gender -Dietary substances -Viruses -Age -Environmental factors

-Dietary substances -Environmental factors -Viruses Carcinogens include chemical agents, environmental factors, dietary substances, viruses, lifestyle factors, and medically prescribed interventions. Although age and gender may increase a person's risk for developing certain types of cancer, they are not carcinogens in and of themselves.

A client has cancer of the neck and is receiving external beam radiation therapy to the site. The client is experiencing trauma to the irradiated skin. The nurse does all of the following. (Select all that apply.) -inspects for skin damage of the chest area -assesses the client for any sun exposure -avoids shaving the irradiated skin -uses cool water to wash the neck area -applies an over-the-counter ointment to the skin

-assesses the client for any sun exposure -avoids shaving the irradiated skin The client receiving external beam radiation therapy may experience trauma to the irradiated skin. To prevent further skin damage, the client is to avoid sun exposure and shaving the irradiated skin area. Other skin areas are not damaged, only the irradiated skin. Lukewarm water is to be used to bathe the area. Water of extreme temperature should be avoided. Many over-the-counter ointments contain metals and may cause additional skin damage.

A 5-year-old client has just received stem cell transplantation as treatment for his leukemia. What are the post procedural nursing interventions for clients receiving any form of stem cell transplantation? 1. Closely monitor the client for at least 3 days. 2. Closely monitor the client for at least 3 months. 3. Closely monitor the client for at least 5 months. 4. Closely monitor the client for at least 4 weeks.

2

A client is receiving chemotherapy to treat breast cancer. Which assessment finding indicates a chemotherapy-induced complication? 1. Blood pressure of 120/64 to 130/72 mm Hg 2. Serum potassium level of 2.6 mEq/L 3. Urine output of 400 ml in 8 hours 4. Sodium level of 142 mEq/L

2

A client, 66 years old, has just been diagnosed with multiple myeloma (a cancer of the plasma) and will be initiating chemotherapy. The nurse, in an outpatient clinic, reviews the medications the client has been taking at home. The medications include pantoprazole (Protonix) for gastroesophageal reflux disease (GERD) and an over-the-counter calcium supplement to prevent osteoporosis. The nurse does the following interventions: (Select all that apply.)

-instructs the client to discontinue calcium -asks about nausea and vomiting -teaches the client to report abdominal or bone pain The client with cancer is at risk for hypercalcemia from bone breakdown. The client should not take an over-the-counter calcium supplement that would increase blood levels of calcium. Signs and symptoms of hypercalcemia include nausea and vomiting. The client may also report abdominal or bone pain with cancer. The client should increase fluid intake to 2 to 4 L per day. Intake would have to be adjusted based on the client's other medical conditions. GERD would not negate an increase in fluid intake. The client most likely would have constipation with hypercalcemia, not diarrhea.

Which of the following is the single largest preventable cause of cancer? a) Tobacco b) Pesticides c) Asbestos d) Arsenic

...

A client receiving external radiation to the left thorax to treat lung cancer has a nursing diagnosis of Risk for impaired skin integrity. Which intervention should be part of this client's care plan? 1. Avoiding using soap on the irradiated areas 2. Wearing a lead apron during direct contact with the client 3. Applying talcum powder to the irradiated areas daily after bathing 4. Removing thoracic skin markings after each radiation treatment

1

A nurse is caring for a recently married, 29-year-old female client, who was diagnosed with acute lymphocytic leukemia. The client is preparing for an allogeneic bone marrow transplant. Which statement by the client demonstrates she understands the informed consent she gave about the diagnosis and treatment? 1. "I'll only need chemotherapy treatment before receiving my bone marrow transplant." 2. "I'll have to remain in the hospital for about 3 months after my transplant." 3. "I always had a good appetite. Even with chemo I shouldn't have to make any changes to my diet." 4. "I should be able to finally start a family after I'm finished with the chemo."

1

According to the tumor-node-metastasis (TNM) classification system, T0 means there is which of the following? 1. No evidence of primary tumor 2. Distant metastasis 3. No distant metastasis 4. No regional lymph node metastasis

1

After being seen in the oncology clinic, a client with severe bone marrow suppression is admitted to the hospital. The client's cancer therapy consisted of radiation and chemotherapy. When developing the care plan for this client, which nursing diagnosis takes priority? 1. Risk for infection 2. Anxiety 3. Imbalanced nutrition: Less than body requirements 4. Risk for injury

1

The nurse evaluates teaching as effective when a female client states that she will 1. Use sunscreen when outdoors. 2. Obtain a cancer history from her parents. 3. Decrease tobacco smoking from one pack/day to half a pack/day. 4. Exercise 30 minutes 3 times each week.

1

What disadvantages of chemotherapy should the patient be informed about prior to starting the regimen? 1. It targets normal body cells as well as cancer cells. 2. It causes a systemic reaction. 3. It attacks cancer cells during their vulnerable phase. 4. It functions against disseminated disease.

1

When the client complains of increased fatigue following radiotherapy, the nurse knows this is most likely to be related to which factor? 1. Radiation can result in myelosuppression. 2. The cancer cells are dying in large numbers. 3. The cancer is spreading. 4. Fighting off infection is an exhausting venture.

1

Which of the following is a term used to describe the process of programmed cell death? 1. Apoptosis 2. Mitosis 3. Angiogenesis 4. Carcinogenesis

1

Which primary cancer treatment goal is prolonged survival and containment of cancer cell growth? 1. Control 2. Prevention 3. Palliation 4. Cure

1

Which type of hematopoietic stem cell transplantation (HSCT) is characterized by cells from a donor other than the patient? 1. Allogeneic 2. Autologous 3. Homogenic 4. Syngeneic

1

A client has been receiving chemotherapy. Upon assessing the client during morning rounds, the nurse notes the client is now bleeding from intravenous and venipuncture sites. Stool is positive for occult blood. The client is requesting to sit in a chair for a meal. The nurse implements the following interventions: (Select all that apply.) 1. Assess level of consciousness. 2. Apply pressure to the bleeding sites. 3. Check intake and output records. 4. Assist the client to a chair. 5. Monitor vital signs once a shift.

1, 2, 3

When a patient takes vincristine, an antineoplastic agent that inhibits DNA and protein synthesis, the patient needs to be informed that he should report which of the following symptoms that would be an expected side-effect of motor neuropathy? Select all that apply. 1. Loss of balance and coordination 2. Cramps and spasms in the legs 3. Muscle weakness 4. Burning and tingling sensations in the extremities

1, 2, 3

A client admitted with pneumonia has a history of lung cancer and heart failure. A nurse caring for this client recognizes that he should maintain adequate fluid intake to keep secretions thin for ease in expectoration. The amount of fluid intake this client should maintain is: 1.4 L. unspecified. 3 L. 2 L.

1.4

A young female client has received chemotherapeutic medications and asks about any effects the treatments will have related to her sexual health. The most appropriate statement by the nurse is 1. "You will continue having your menses every month." 2. "You will need to practice birth control measures." 3. "You will experience menopause now." 4. "You will be unable to have children."

2

During chemotherapy, an oncology client has a nursing diagnosis of Impaired oral mucous membrane related to decreased nutrition and immunosuppression secondary to the cytotoxic effects of chemotherapy. Which nursing intervention is most likely to decrease the pain of stomatitis? 1. Recommending that the client discontinue chemotherapy 2. Providing a solution of viscous lidocaine for use as a mouth rinse 3. Checking regularly for signs and symptoms of stomatitis 4. Monitoring the client's platelet and leukocyte counts

2

Palliation refers to 1. the spread of cancer cells from the primary tumor to distant sites. 2. relief of symptoms associated with cancer. 3. the lowest point of white blood cell depression after therapy that has toxic effects on the bone marrow. 4. hair loss related to the treatment of cancer.

2

The client has finished the first round of chemotherapy. Which statement made by the client indicates a need for further teaching by the nurse? 1. "Hair loss may not occur until after the second round of therapy." 2. "I can continue taking my vitamins and herbs because they make me feel better." 3. "I will eat clear liquids for the next 24 hours." 4. "I will use birth control measures until after all treatment is completed."

2

The clinic nurse is caring for a client who has just been diagnosed with a tumor. The client says to the nurse "The doctor says my tumor is benign. What does that mean?" What is the nurse's best response? 1. "Benign tumors can spread from one place to another." 2. "Benign tumors don't usually cause death." 3. "Benign tumors grow very rapidly." 4. "Benign tumors invade surrounding tissue."

2

The nurse is caring for a patient undergoing an incisional biopsy. Which of the following statements does the nurse understand is true about an incisional biopsy? 1. It treats cancer with lymph node involvement. 2. It removes a wedge of tissue for diagnosis. 3. It removes an entire lesion and surrounding tissue. 4. It is used to remove the cancerous cells using a needle.

2

The nurse is conducting a screening for familial predisposition of cancer. Which of the following should the nurse note as a possible indication of hereditary cancer syndrome? 1. Onset of cancer after age 50 in family member 2. An aunt and uncle diagnosed with cancer 3. A second cousin diagnosed with cancer 4. A first cousin diagnosed with cancer

2

The nurse is teaching a client newly diagnosed with cancer about chemotherapy. The nurse tells the client he'll receive an antitumor antibiotic. The nurse knows that this type of medications is: 1. cell-cycle specific in the S phase. 2. cell-cycle nonspecific. 3. cell-cycle specific in the P phase. 4. cell-cycle specific in the M phase.

2

The nurse working on a bone marrow unit knows that it is a priority to monitor which of the following in a client who has just undergone a stem cell transplant? 1. Monitor the client's toilet patterns. 2. Monitor the client closely to prevent infection. 3. Monitor the client's heart rate. 4. Monitor the client's physical condition.

2

What should the nurse tell a female client who is about to begin chemotherapy and anxious about losing her hair? 1. Her hair will grow back within 2 months post therapy. 2. She should consider getting a wig or cap before she loses her hair. 3. Alopecia related to chemotherapy is relatively uncommon. 4. Her hair will grow back the same as it was before treatment.

2

When caring for a client who is receiving external beam radiation, which is the key point for the nurse to incorporate into the plan of care? 1. Avoid showering or washing over skin markings. 2. Inspect the skin frequently. 3. Time, distance, and shielding 4. The use of disposable utensils and wash cloths

2

Which of the following is a characteristic of a malignant tumor? 1. It is usually slow growing. 2. It gains access to the blood and lymphatic channels. 3. It grows by expansion. 4. It demonstrates cells that are well differentiated.

2

Which of the following occurs when there is accumulation of fluid in the pericardial space that compresses the heart? 1. DIC 2. Cardiac tamponade 3. SIADH 4. Superior Vena Cava Syndrome (SVCS)

2

Which of the following would be inconsistent as a common side effect of chemotherapy? 1. Alopecia 2. Weight gain 3. Fatigue 4. Myelosuppression

2

Which statement by a client undergoing external radiation therapy indicates the need for further teaching? 1. "I'll not use my heating pad during my treatment." 2. "I'm worried I'll expose my family members to radiation." 3. "I'll wash my skin with mild soap and water only." 4. "I'll wear protective clothing when outside."

2

Which statement by a client undergoing external radiation therapy indicates the need for further teaching? 1. "I'll wash my skin with mild soap and water only." 2. "I'm worried I'll expose my family members to radiation." 3. "I'll not use my heating pad during my treatment." 4. "I'll wear protective clothing when outside."

2

Which type of vaccine uses the patient's own cancer cells, which are prepared for injection, back into the patient? 1. Allogeneic 2. Autologous 3. Therapeutic 4. Prophylactic

2

The client is to receive cyclophosphamide (Cytoxan) 50 mg/kg intravenously in divided doses over 5 days. The client weighs 176 pounds. How many mg of cyclophosphamide will the client receive each day?

800

You are teaching clients about cancer prevention and explain that the root cause of cancer is damage to cellular deoxyribonucleic acid (DNA) as a result from multiple factors. You explain that which of the following are possible carcinogens? Select all that apply. 1. Gender 2. Dietary substances 3. Environmental factors 4. Viruses 5. Age

2, 3, 4

The client has a body surface area of 2.05 m². He is prescribed vincristine (Oncovin) 1.4 mg/m². Vincristine is available as 1 mg/1 mL. How many mL will the nurse administer? Round your answer to the nearest tenth.

2.9

The client has a body surface area of 2.05 m². He is prescribed vincristine (Oncovin) 1.4 mg/m². Vincristine is available as 1 mg/1 mL. How many mL will the nurse administer? Round your answer

2.9 The dose ordered is 1.4 mg for each 1 m² of the client's body surface area, which is 2.05. 1.4 mg/m² x 2.05 m² = 2.87 mg. The dose available is 1 mg for each 1 mL. 1 mg/1 mL x 2.87 mg = 2.87 mL. Rounding your answer to 1 decimal place would be 2.9 mL.

A client diagnosed with acute myelocytic leukemia has been receiving chemotherapy. During the last 2 cycles of chemotherapy, the client developed severe thrombocytopenia requiring multiple platelet transfusions. The client is now scheduled to receive a third cycle. How can the nurse best detect early signs and symptoms of thrombocytopenia? 1. Check the client's history for a congenital link to thrombocytopenia. 2. Perform a cardiovascular assessment every 4 hours. 3. Closely observe the client's skin for petechiae and bruising. 4. Monitor daily platelet counts.

3

A month following biopsy, a 75-year-old female client returns to the surgeon's office for a report on her diagnostic procedure to determine the cell composition of her abdominal neoplasm. Which of the following terms is significant to indicate the likelihood of the tumor spreading? 1. Neoplasm 2. Lesion 3. Benign 4. Primary site

3

A newly diagnosed cancer client is crying and states the following to the nurse: "I promised God that I will be a better person if I can just get better." What is the appropriate assessment of this comment by the nurse? 1. Anger directed toward nursing staff is not unusual in dealing with cancer clients. 2. The client is just trying to protect self from potential loss. 3. The cancer is viewed as a punishment from past actions. 4. Loss is inevitable so client is making final plans.

3

A nurse is administering daunorubicin (DaunoXome) to a patient with lung cancer. Which situation requires immediate intervention? 1. The client begins to shiver. 2. The client states he is nauseous. 3. The I.V. site is red and swollen. 4. The laboratory reports a white blood cell (WBC) count of 1,000/mm3.

3

A nurse is administering daunorubicin through a peripheral I.V. line when the client complains of burning at the insertion site. The nurse notes no blood return from the catheter and redness at the I.V. site. The client is most likely experiencing which complication? 1. Erythema 2. Flare 3. Extravasation 4. Thrombosis

3

A nurse is caring for a client who is receiving chemotherapy and has a platelet count of 30,000/mm3. Which statement by the client indicates a need for additional teaching? 1. "I take a stool softener every morning." 2. "I use an electric razor to shave." 3. "I floss my teeth every morning." 4. "I removed all the throw rugs from the house."

3

A patient is scheduled for cryosurgery for cervical cancer and tells the nurse, "I am not exactly sure what the doctor is going to do." What is the best response by the nurse? 1. "The physician is going to use medication to inject the area." 2. "The physician is going to use radiofrequency to ablate the area." 3. "The physician is going to use liquid nitrogen to freeze the area." 4. "The physician is going to use a laser to remove the area."

3

A side-effect of chemotherapy is renal damage. To prevent this, the nurse should: 1. Withhold medication when the blood urea nitrogen level exceeds 20 mg/dL. 2. Modify the diet to acidify the urine, thus preventing uric acid crystallization. 3. Encourage fluid intake, if possible, to dilute the urine. 4. Limit fluids to 1,000 mL/day to minimize stress on the renal tubules.

3

According to the TNM classification system, T0 means there is 1. distant metastasis. 2. no regional lymph node metastasis. 3. no evidence of primary tumor. 4. no distant metastasis.

3

After being seen in the oncology clinic, a client with severe bone marrow suppression is admitted to the hospital. The client's cancer therapy consisted of radiation and chemotherapy. When developing the care plan for this client, which nursing diagnosis takes priority? 1. Risk for injury 2. Imbalanced nutrition: Less than body requirements 3. Risk for infection 4. Anxiety

3

After cancer chemotherapy, a client experiences nausea and vomiting. The nurse should assign highest priority to which intervention? 1. Withholding fluids for the first 4 to 6 hours after chemotherapy administration 2. Serving small portions of bland food 3. Administering metoclopramide and dexamethasone as ordered 4. Encouraging rhythmic breathing exercises

3

An important nursing function is monitoring factors that may indicate that bleeding is occurring. One serum indicator is a (an): 1. Lymphocyte count of 30%. 2. Reticulocyte count of 1%. 3. Platelet count of 60,000/mm3. 4. Neutrophil count of 60%.

3

During which step of cellular carcinogenesis do cellular changes exhibit increased malignant behavior? 1. Initiation 2. Promotion 3. Progression 4. Prolongation

3

The nurse at the clinic explains to the patient that the surgeon will be removing a mole on the patient's back that has the potential to develop into cancer. The nurse informs the patient that this is what type of procedure? 1. Palliative 2. Diagnostic 3. Prophylactic 4. Reconstructive

3

What does the nurse understand is the rationale for administering allopurinol for a patient receiving chemotherapy?

It lowers serum and uric acid levels.

The nurse is caring for a client who is scheduled for chemotherapy. Which is the best statement the nurse can make about the client experiencing chemotherapy-induced alopecia? 1. "Clients with alopecia will have delay in grey hair." 2. "New hair growth will return without any change to color or texture." 3. "The hair loss is temporary." 4. "Wigs can be used after the chemotherapy is completed."

3

The nurse is providing client teaching for a client undergoing chemotherapy. What dietary modifications should the nurse advise? 1. Eat wholesome meals. 2. Avoid intake of fluids. 3. Avoid spicy and fatty foods. 4. Eat warm or hot foods.

3

The nurse working on a bone marrow unit knows that it is a priority to monitor which of the following in a client who has just undergone a stem cell transplant? 1. Monitor the client's heart rate. 2. Monitor the client's physical condition. 3. Monitor the client closely to prevent infection. 4. Monitor the client's toilet patterns.

3

Which of the following is a term used to describe the process of programmed cell death? 1. Mitosis 2. Carcinogenesis 3. Apoptosis 4. Angiogenesis

3

Which oncologic emergency involves the failure in the negative feedback mechanism that normally regulates the release of antidiuretic hormone (ADH)? 1. Cardiac tamponade 2. Disseminated intravascular coagulation (DIC) 3. Syndrome of inappropriate antidiuretic hormone release (SIADH) 4. Tumor lysis syndrome

3

While doing a health history, a client tells you that her mother, grandmother, and sister died of cancer. The client wants to know what she can do to keep from getting cancer. What would be your best response? 1. "With your family history, there is nothing you can do to prevent getting cancer." 2. "If you eat right, exercise, and get enough rest, you can prevent breast cancer." 3. "You can't prevent cancer, but you can have your blood analyzed for tumor markers to see what your risk level is." 4. "Cancer often skips a generation, so don't worry about it."

3

Following a BMT the patient should be monitored for at least

3 months

A bowel resection is scheduled for a client with the diagnosis of colon cancer with metastasis to the liver and bone. Which statement by the nurse best explains the purpose of the surgery? 1. "This surgery will prevent further tumor growth." 2. "Removing the tumor is a primary treatment for colon cancer." 3. "Once the tumor is removed, cell pathology can be determined." 4. "Tumor removal will promote comfort."

4

A client complains of sporadic epigastric pain, yellow skin, nausea, vomiting, weight loss, and fatigue. Suspecting gallbladder disease, the physician orders a diagnostic workup, which reveals gallbladder cancer. Which nursing diagnosis is appropriate for this client? 1. Disturbed body image 2. Chronic low self-esteem 3. Impaired swallowing 4. Anticipatory grieving

4

A client has been receiving chemotherapy to treat cancer. Which assessment finding suggests that the client has developed stomatitis? 1. Yellow tooth discoloration 2. Rust-colored sputum 3. White, cottage cheese-like patches on the tongue 4. Red, open sores on the oral mucosa

4

A decrease in circulating white blood cells is 1. neutropenia. 2. granulocytopenia. 3. thrombocytopenia. 4. leukopenia.

4

A nurse is caring for a client receiving chemotherapy. Which assessment finding places the client at the greatest risk for an infection? 1. Ate 75% of all meals during the day 2. White blood cell (WBC) count of 9,000 cells/mm3 3. Temperature of 98.3° F (36.8° C) 4. Stage 3 pressure ulcer on the left heel

4

A nurse is caring for a client receiving chemotherapy. Which nursing action is most appropriate for handling chemotherapeutic agents? 1. Disconnect I.V. tubing with gloved hands. 2. Throw I.V. tubing in the trash after the infusion is stopped. 3. Break needles after the infusion is discontinued. 4. Wear disposable gloves and protective clothing.

4

A patient is admitted for an excisional biopsy of a breast lesion. What intervention should the nurse provide for the care of this patient? 1. Clarify information provided by the physician. 2. Counsel the patient about the possibility of losing her breast. 3. Provide aseptic care to the incision postoperatively. 4. Provide time for the patient to discuss her concerns.

4

A patient with brain tumor is undergoing radiation and chemotherapy for treatment of cancer. Of late, the patient is complaining of swelling in the gums, tongue, and lips. Which of the following is the most likely cause of these symptoms? 1. Nadir 2. Extravasation 3. Neutropenia 4. Stomatitis

4

For a client newly diagnosed with radiation-induced thrombocytopenia, the nurse should include which intervention in the care plan? 1. Administering aspirin if the temperature exceeds 102° F (38.8° C) 2. Placing the client in strict isolation 3. Providing for frequent rest periods 4. Inspecting the skin for petechiae once every shift

4

For a client newly diagnosed with radiation-induced thrombocytopenia, the nurse should include which intervention in the care plan? 1. Providing for frequent rest periods 2. Administering aspirin if the temperature exceeds 102° F (38.8° C) 3. Placing the client in strict isolation 4. Inspecting the skin for petechiae once every shift

4

In which phase of the cell cycle does cell division occur? 1. G1 phase 2. S phase 3. G2 phase 4. Mitosis

4

The client is receiving a vesicant anti neo plastic for treatment of cancer. Which assessment finding would require the nurse to take immediate action? 1. Stomatitis 2. Bone pain 3. Nausea and vomiting 4. Extra vasation

4

The nurse is caring for a client who is scheduled for chemotherapy. Which is the best statement the nurse can make about the client experiencing chemotherapy-induced alopecia? 1. "New hair growth will return without any change to color or texture." 2. "Wigs can be used after the chemotherapy is completed." 3. "Clients with alopecia will have delay in grey hair." 4. "The hair loss is temporary."

4

The nurse is evaluating the client's risk for cancer and recommends changes when the client states she 1. drinks 1 glass of wine at dinner each night 2. uses the treadmill for 30 minutes on 5 days each week 3.works as a secretary at a medical radiation treatment center 4. eats red meat such as steaks or hamburgers every day

4

The physician is attending to a 72-year-old patient with a malignant brain tumor. The physician recommends immediate radiation therapy. Which of the following is a reason for the physician's recommendation? 1. To destroy marginal tissues 2. To remove the tumor from the brain 3. To analyze involved lymph nodes 4. To prevent the formation of new cancer cells

4

Which of the following does a nurse thoroughly evaluate before a bone marrow transplant (BMT) procedure? 1. Allergy history 2. Family history 3. Drug history 4. Blood studies

4

A nurse is developing a care plan for bone marrow suppression, the major dose-limiting adverse reaction to floxuridine (FUDR). How long after drug administration does bone marrow suppression become noticeable?

7 to 14 days

A nurse is developing a care plan for bone marrow suppression, the major dose-limiting adverse reaction to floxuridine (FUDR). How long after drug administration does bone marrow suppression become noticeable?

7 to 14 days Bone marrow suppression becomes noticeable 7 to 14 days after floxuridine administration. Bone marrow recovery occurs in 21 to 28 days.

A nurse is developing a care plan for bone marrow suppression, the major dose-limiting adverse reaction to floxuridine (FUDR). How long after drug administration does bone marrow suppression become noticeable?

7-14 Days

The client is to receive cyclophosphamide (Cytoxan) 50 mg/kg intravenously in divided doses over 5 days. The client weighs 176 pounds. How many mg of cyclophosphamide will the client receive each day? Enter the correct number ONLY.

800

The client is to receive cyclophosphamide (Cytoxan) 50 mg/kg intravenously in divided doses over 5 days. The client weighs 176 pounds. How many mg of cyclophosphamide will the client receive each day?

800mg

The lethal tumor dose is defined as the dose that will eradicate what percentage of the tumor yet preserve normal tissue? a) 85% b) 95% c) 65% d) 75%

95% Explanation: The radiation dosage is dependent on the sensitivity of the target tissues to radiation and on the tumor size. The lethal tumor dose is defined as the dose that will eradicate 95% of the tumor yet preserve normal tissue.

A client complains of unusual vaginal discharge and heavier, more painful periods than usual. This client's symptoms may be possible warning signs of which type of cancer? A. cervical B. kidney C. colon D. bladder

A

A client diagnosed with cancer has his tumor staged and graded based on what factors? A) How they tend to grow and the cell type B) How they spread and tend to grow C) How they differentiate the cell type D) How they spread and differentiate

A

A client is recovering from a craniotomy with tumor debulking. Which comment by the client indicates to the nurse a correct understanding of what the surgery entailed? A) "I guess the doctor could not remove the entire tumor." B) "I am so glad the doctor was able to remove the entire tumor." C) "I will be glad to finally be done with treatments for this thing." D) "Thank goodness the tumor is contained and curable."

A

A client with a recent history of GI disturbance has been schedulded for a barium study. The physician ordered this particular test for this client because it will: A. show movement of the GI tract B. show tumor "hot spots" in the GI tract C. remove a tissue sample from the GI tract D. provide a three-dimensional cross-sectional view

A

Carcinogens are factors related to the formation of various malignancies. Which factor has the greatest impact on the development of all cancers? A. chemical agents B. environmental factors C. viruses D. defective genes

A

The client is receiving a vesicant antineoplastic for treatment of cancer. Which assessment finding would require the nurse to take immediate action? A) Extravasation B) Stomatitis C) Nausea and vomiting D) Bone pain

A

The nurse is caring for a client is scheduled for chemotherapy followed by autologous stem cell transplant. Which of the following statements by the client indicates a need for further teaching? A) "I hope they find a bone marrow donor who matches." B) "The doctor will remove cells from my bone marrow before beginning chemotherapy." C) "I will receive chemotherapy until most of the cancer is gone, and then I will get my own stem cells back." D) "I will need to be in protective isolation for up to 3 months after treatment."

A

The nurse is caring for a client who is scheduled for chemotherapy. Which is the best statement the nurse can make about the client experiencing chemotherapy-induced alopecia? A) "The hair loss is temporary." B) "New hair growth will return without any change to color or texture." C) "Clients with alopecia will have delay in grey hair." D) "Wigs can be used after the chemotherapy is completed."

A

The oncologist advises a client with an extensive family history of breast cancer to consider a mastectomy. This procedure would be considered which type of surgery? A. prophylactic B. cryosurgery C. palliative D. local excision

A

When providing care for a client with stage IV cancer, the nurse knows to include which intervention in the plan of care? A) Incorporating touching and listening B) Encouraging the expression of life regrets C) Assessing signs and symptoms of impending death D) Discussing ways for the client to handle the dying process

A

Which term is a growth-based classification of tumors? A. malignant B. sarcoma C. carcinoma D. leukemia

A

While completing an admission assessment, the client reports a family history of ovarian cancer among a maternal grandmother, aunt, and sister. The nurse knows that these cancers are most likely associated with what etiology? A) Inherited gene mutation B) Smoking and tobacco use C) Exposure to chemicals and spermicides D) Increased tumor suppressor genes

A

While doing a health history, a client tells you that her mother, grandmother, and sister died of cancer. The client wants to know what she can do to keep from getting cancer. What would be your best response? A) "You can't prevent cancer, but you can have your blood analyzed for tumor markers to see what your risk level is." B) "If you eat right, exercise, and get enough rest, you can prevent breast cancer." C) "With your family history, there is nothing you can do to prevent getting cancer." D) "Cancer often skips a generation, so don't worry about it."

A

You are an oncology nurse caring for a client who is taking antineoplastic agents. What adverse symptoms must you monitor for in this client? A)Symptoms of gout B)Symptoms of hypertension C)Symptoms of diarrhea D)Symptoms of anemia

A

The oncology nurse is giving chemotherapy to a client in a short stay area. The client confides that they are very depressed. The nurse recognizes depression as which of the following?

A normal reaction to the diagnosis of cancer

The oncology nurse is giving chemotherapy to a client in a short stay area. The client confides that they are very depressed. The nurse recognizes depression as which of the following?

A normal reaction to the diagnosis of cancer.

The oncology nurse is giving chemotherapy to a client in a short stay area. The client confides that they are very depressed. The nurse recognizes depression as which of the following?

A normal reaction to the diagnosis of cancer. Clients have many reactions, ranging from anxiety, fear, and depression to feelings of guilt related to viewing cancer as a punishment for past actions or failure to practice a healthy life-style. They also may express anger related to the diagnosis and their inability to be in control. While depression is a psychiatric diagnosis not everyone has the diagnosis sometime in their life; depression is not a side effect of the neoplastic drugs nor is it an aberrant psychologic reaction to the chemotherapy.

The nurse recognizes which of the following alternative therapies as appropriate in the care of cancer clients? Select all that apply. A) Reminiscing B) Patient-controlled analgesia C) Hot and cold therapy D) Epidural stimulators E) Alternating analgesics F) Nonopioid use

A,C

The client is scheduled for a breast lump excision and sentinel node biopsy. What should the nurse know in planning care for the client with a negative biopsy report?

A wide excision of lump will be performed

The client is scheduled for a breast lump excision and sentinel node biopsy. What should the nurse know in planning care for the client with a negative biopsy report?

A wide excision of lump will be performed.

The client is scheduled for a breast lump excision and sentinel node biopsy. What should the nurse know in planning care for the client with a negative biopsy report?

A wide excision of lump will be performed. The sentinel node is the first node in which a tumor will drain; if no malignant cells are found there, additional excision or radical removal will not be necessary. Excision of the lump along with a wide margin of cancer-free tissue is standard treatment for malignant tumors.

A client is struggling emotionally with a recent diagnosis of lung cancer. Which nursing interventions would be most effective in helping the client deal with these emotions? Select all that apply. A. assess the client's immediate support systems B. encourage the client to make better choices C. encourage the client to set goals for the future D. teach the client to use relaxation techniques

A, C, D

The root cause of cancer is damage to cellular deoxyribonucleic acid (DNA) which can be caused my many factors, or carcinogens. What factors can be carcinogenic? Select all that apply. A. environmental factors B. gender C. viruses D. dietary substances E. age

A, C, D

You are a public health nurse giving a talk on the warning signals of cancer to a local community group. Which of the following are the warning signals of cancer? Select all that apply. A) Sores that don't heal B) Unusual bleeding or discharge C) Yellow discoloration of body area D) Tenderness or pain E) Persistent indigestion

A,B,E

Which nursing interventions are most important when implementing care for a client receiving temporary internal sealed radiation therapy? Select all that apply. A) Time, distance, and shielding B) Count wires, threads, or needles every shift C) Maintain indelible skin markings. D) Provide rest periods between treatments. E) Administer treatment through IV access port. F) Avoid standing in direct path of implants.

A,B,F

The nurse is providing an educational presentation on dietary recommendations for reducing the risk of cancer. Which of the following food selections would demonstrate a good understanding of the information provided in the presentation? Select all that apply. A) Egg white omelet with spinach and mushrooms B) Crispy chicken Caesar Salad C) Steamed broccoli and carrots D) Turkey breast on whole wheat bread E) Smoked salmon F) Vegetable and cheddar quiche

A,C,D

A nurse is caring for a client who is receiving chemotherapy and has a platelet count of 30,000/mm3. Which statement by the client indicates a need for additional teaching? A. "I floss my teeth every morning." B. "I use an electric razor to shave." C. "I removed all the throw rugs from the house." D. "I take a stool softener every morning."

A. "I floss my teeth every morning." A client with a platelet count of 30,000/mm3 is at risk for bleeding and shouldn't floss his teeth. Flossing may increase the risk of bleeding in a client with a platelet count less than 40,000/mm3. Using an electric razor is appropriate because doing so helps minimize the risk of cutting when shaving. Taking a stool softener helps decrease potential trauma to the GI tract that may cause bleeding. Removing throw rugs from the house helps prevent falls, which could lead to uncontrolled bleeding.

A patient will be receiving radiation for 6 weeks for the treatment of breast cancer and asks the nurse why it takes so long. What is the best response by the nurse? A. "It will allow time for the repair of healthy tissue." B. "It allows time for you to cope with the treatment." C. "It is not really understood why you have to go for 6 weeks of treatment." D. "It will decrease the incidence of leukopenia and thrombocytopenia."

A. "It will allow time for the repair of healthy tissue." In external-beam radiation therapy (EBRT), the total radiation dose is delivered over several weeks in daily doses called fractions. This allows healthy tissue to repair and achieves greater cell kill by exposing more cells to the radiation as they begin active cell division. Repeated radiation treatments over time (fractionated doses) also allow for the periphery of the tumor to be reoxygenated repeatedly, because tumors shrink from the outside inward. This increases the radiosensitivity of the tumor, thereby increasing tumor cell death (Kelvin, 2010).

Which of the following is a sign or symptoms of septic shock? A. Altered mental status B. Increased urine output C. Hypertension D. Warm, moist skin

A. Altered mental status -Signs of septic shock include altered mental status, cool and clammy skin, decreased urine output, and hypotension.

The nurse is conducting a screening for familial predisposition of cancer. Which of the following should the nurse note as a possible indication of hereditary cancer syndrome? A. An aunt and uncle diagnosed with cancer B. A first cousin diagnosed with cancer C. A second cousin diagnosed with cancer D. Onset of cancer after age 50 in family member

A. An aunt and uncle diagnosed with cancer The hallmarks of hereditary cancer syndrome include cancer in two or more first-degree or second-degree relatives, early onset of cancer in family members younger than age 50, same type of cancer in several family members, individual family members with more than one type of cancer, and a rare cancer in one or more family members.

5-Fluorouracil (5FU) is classified as which type of antineoplastic agent? A. Antimetabolite B. Alkylating C. Nitrosoureas D. Mitotic spindle poisons

A. Antimetabolite 5-FU is an antimetabolite. An example of an alkylating agent is nitrogen mustard. A nitrosourea is streptozocin. A mitotic spindle poison is vincristine (VCR).

A client receiving external radiation to the left thorax to treat lung cancer has a nursing diagnosis of Risk for impaired skin integrity. Which intervention should be part of this client's care plan? A. Avoiding using soap on the irradiated areas B. Wearing a lead apron during direct contact with the client C. Removing thoracic skin markings after each radiation treatment D. Applying talcum powder to the irradiated areas daily after bathing

A. Avoiding using soap on the irradiated areas Because external radiation commonly causes skin irritation, the nurse should wash the irradiated area with water only and leave the area open to air. No soaps, deodorants, lotions, or powders should be applied. A lead apron is unnecessary because no radiation source is present in the client's body or room. Skin in the area to be irradiated is marked to position the radiation beam as precisely as possible; skin markings must not be removed.

A client diagnosed with acute myelocytic leukemia has been receiving chemotherapy. During the last 2 cycles of chemotherapy, the client developed severe thrombocytopenia requiring multiple platelet transfusions. The client is now scheduled to receive a third cycle. How can the nurse best detect early signs and symptoms of thrombocytopenia? A. Closely observe the client's skin for petechiae and bruising. B. Perform a cardiovascular assessment every 4 hours. C. Monitor daily platelet counts. D. Check the client's history for a congenital link to thrombocytopenia.

A. Closely observe the client's skin for petechiae and bruising. The nurse should closely observe the client's skin for petechiae and bruising. Daily laboratory testing may not reflect the client's condition as quickly as subtle changes in the client's skin. Performing a cardiovascular assessment every 4 hours and checking the clients history for a congenital link to thrombocytopenia don't help detect early signs and symptoms of thrombocytopenia.

The drug interleukin-2 is an example of which type of biologic response modifier? A. Cytokine B. Retinoids C. Monoclonal antibodies D. Antimetabolites

A. Cytokine Other cytokines include interferon alfa and filgrastim. Monoclonal antibodies include rituximab, trastuzumab, and gemtuzumab. Retinoic acid is an example of a retinoid. Antimetabolites are cell cycle-specific antineoplastic agents.

According to the tumor-node-metastasis (TNM) classification system, T0 means there is which of the following? A. No evidence of primary tumor B. Distant metastasis C. No regional lymph node metastasis D. No distant metastasis

A. No evidence of primary tumor -T0 means that there is no evidence of primary tumor. N0 means that there is no regional lymph node metastasis. M0 means that there is no distant metastasis. M1 means that there is distant metastasis.

During chemotherapy, an oncology client has a nursing diagnosis of Impaired oral mucous membrane related to decreased nutrition and immunosuppression secondary to the cytotoxic effects of chemotherapy. Which nursing intervention is most likely to decrease the pain of stomatitis? A. Providing a solution of viscous lidocaine for use as a mouth rinse B. Checking regularly for signs and symptoms of stomatitis C. Recommending that the client discontinue chemotherapy D. Monitoring the client's platelet and leukocyte counts

A. Providing a solution of viscous lidocaine for use as a mouth rinse -To decrease the pain of stomatitis, the nurse should provide a solution of hydrogen viscous lidocaine for the client to use as a mouth rinse. (Commercially prepared mouthwashes contain alcohol and may cause dryness and irritation of the oral mucosa.) The nurse also may administer systemic analgesics as ordered. Stomatitis occurs 7 to 10 days after chemotherapy begins; thus, stopping chemotherapy wouldn't be helpful or practical. Instead, the nurse should stay alert for this potential problem to ensure prompt treatment. Monitoring platelet and leukocyte counts may help prevent bleeding and infection but wouldn't decrease pain in this highly susceptible client. Checking for signs and symptoms of stomatitis also wouldn't decrease the pain.

When the client complains of increased fatigue following radiotherapy, the nurse knows this is most likely to be related to which factor? A. Radiation can result in myelosuppression. B. The cancer cells are dying in large numbers. C. The cancer is spreading. D. Fighting off infection is an exhausting venture.

A. Radiation can result in myelosuppression. Fatigue results from anemia associated with myelo suppression and decreased RBC production. The spreading of cancer can cause many symptoms dependent on location and type of cancer but not a significant factor to support fatigue with radiotherapy. The production of healthy cells can increase metabolic rate, but death of cancer cells does not support fatigue in this case. Fighting infection can cause fatigue, but there is no evidence provided to support presence of infection in this client.

Which oncologic emergency involves the failure in the negative feedback mechanism that normally regulates the release of antidiuretic hormone (ADH)? A. Syndrome of inappropriate antidiuretic hormone release (SIADH) B. Tumor lysis syndrome C. Cardiac tamponade D. Disseminated intravascular coagulation (DIC) SUBMIT ANSWER

A. Syndrome of inappropriate antidiuretic hormone release (SIADH) -SIADH is a result of the failure in the negative feedback mechanism that normally regulates the release of antidiuretic hormone (ADH). Cardiac tamponade is an accumulation of fluid in the pericardial space. DIC is a complex disorder of coagulation and fibrinolysis, which results in thrombosis and bleeding. Tumor lysis syndrome is a rapidly developing oncologic emergency that results from the rapid release of intracellular contents as a result of radiation- or chemotherapy-induced cell destruction of large or rapidly growing cancers such as leukemia.

A patient diagnosed with colon cancer presents with the characteristic symptoms of a left-sided lesion. Which of the following symptoms are indicative of this disorder? Select all that apply. Abdominal distention Black, tarry stools Constipation Narrowing stools Dull abdominal pain

Abdominal distention Constipation Narrowing stools

A patient is undergoing chemotherapy treatment for prostate cancer and has lost considerable weight due to nausea and vomiting. Which of the following nursing interventions is appropriate for the patient?

Adjusting meal plan before/after chemotherapy

A client is undergoing chemotherapy treatment for prostate cancer and has lost considerable weight due to nausea and vomiting. Which nursing intervention is appropriate for the client?

Adjusting the client's meal plan before and after chemotherapy

A client is undergoing chemotherapy treatment for prostate cancer and has lost considerable weight due to nausea and vomiting. Which nursing intervention is appropriate for the client?

Adjusting the client's meal plan before and after chemotherapy The nurse should readjust the client's meal plan before and after chemotherapy administration. The nurse should take into consideration the client's likes and dislikes and avoid foods with strong odors. The nurse should ensure adequate fluid hydration before, during, and after drug administration when the client has side effects of nausea and vomiting. Fresh fruits are not recommended when the client is at risk of infection, such as during chemotherapy. Beta-blockers are not administered to control nausea and vomiting.

Following surgery for adenocarcinoma, the client learns the tumor stage is T3,N1,M0. What treatment mode will the nurse anticipate?

Adjuvant therapy

Following surgery for adenocarcinoma, the client learns the tumor stage is T3,N1,M0. What treatment mode will the nurse anticipate?

Adjuvant therapy is likely.

Following surgery for adenocarcinoma, the client learns the tumor stage is T3,N1,M0. What treatment mode will the nurse anticipate?

Adjuvant therapy is likely. T3 indicates a large tumor size with N1 indicating regional lymph node involvement. Although M0 suggest no metastasis, following with adjuvant (chemotherapy or radiation therapy) treatment is indicated to prevent the spread of cancer outside the lymph to other organs. The tumor staging of stage IV is indicative of palliative care.

Following surgery for adenocarcinoma, the client learns the tumor stage is T3,N1,M0. What treatment mode will the nurse anticipate? a) No further treatment is indicated. b) Repeat biopsy is needed before treatment begins. c) Palliative care is likely. d) Adjuvant therapy is likely.

Adjuvant therapy is likely. Explanation: T3 indicates a large tumor size with N1 indicating regional lymph node involvement. Although M0 suggest no metastasis, following with adjuvant (chemotherapy or radiation therapy) treatment is indicated to prevent the spread of cancer outside the lymph to other organs. The tumor staging of stage IV is indicative of palliative care

A patient with a diagnosis of renal cell carcinoma is being treated with chemotherapy. During a previous round of chemotherapy, the patient's tumor responded well to treatment but the chemotherapy caused intense nausea and vomiting. How should the patient's potential nausea and vomiting be addressed during this current round of treatment? Prioritize nonpharmacological treatments over medications. Administer antiemetics in anticipation of the patient's nausea. Provide the patient with antiemetics at his first complaint of nausea. Administer antiemetics if the patient vomits or believes he will soon vomit.

Administer antiemetics in anticipation of the patient's nausea.

After cancer chemotherapy, a client experiences nausea and vomiting. The nurse should assign highest priority to which intervention?

Administering metoclopramide and dexamethasone as ordered

After cancer chemotherapy, a client experiences nausea and vomiting. The nurse should assign highest priority to which intervention? Withholding fluids for the first 4 to 6 hours after chemotherapy administration Encouraging rhythmic breathing exercises Administering metoclopramide and dexamethasone as ordered Serving small portions of bland food

Administering metoclopramide and dexamethasone as ordered

After cancer chemotherapy, a client experiences nausea and vomiting. The nurse should assign highest priority to which intervention?

Administering metoclopramide and dexamethasone as ordered The nurse should assign highest priority to administering an antiemetic, such as metoclopramide, and an anti-inflammatory agent, such as dexamethasone, because it may reduce the severity of chemotherapy-induced nausea and vomiting. This intervention, in turn, helps prevent dehydration, a common complication of chemotherapy. Serving small portions of bland food, encouraging rhythmic breathing exercises, and withholding fluids for the first 4 to 6 hours are less likely to achieve this outcome.

A patient, age 67 years, is admitted for diagnostic studies to rule out cancer. The patient is Caucasian, married, has been employed as a landscaper for 40 years, and has a 36-year history of smoking a pack of cigarettes daily. What significant risk factors does the nurse recognize this patient has? (Select all that apply.) Race Age Marital status Occupation Cigarette smoking

Age Cigarette smoking Occupation Most cancer occurs in people older than 65 years. Although the overall rate of cancer deaths has declined, cancer death rates in African American men remain substantially higher than those among Caucasian men and twice those of Hispanic men. Excessive exposure to the ultraviolet rays of the sun, especially in fair-skinned people, increases the risk of skin cancers. Factors such as clothing styles (sleeveless shirts or shorts), the use of sunscreens, occupation, recreational habits, and environmental variables, including humidity, altitude, and latitude, all play a role in the amount of exposure to ultraviolet light. Tobacco smoke, thought to be the single most lethal chemical carcinogen, accounts for at least 30% of cancer deaths in humans

A patient, age 67 years, is admitted for diagnostic studies to rule out cancer. The patient is Caucasian, married, has been employed as a landscaper for 40 years, and has a 36-year history of smoking a pack of cigarettes daily. What significant risk factors does the nurse recognize this patient has? (Select all that apply.) Race Occupation Marital status Age Cigarette smoking

Age Cigarette smoking Occupation

Which type of hematopoietic stem cell transplantation (HSCT) is characterized by cells from a donor other than the patient?

Allogeneic

Which type of hematopoietic stem cell transplantation (HSCT) is characterized by cells from a donor other than the patient? Homogenic Allogeneic Autologous Syngeneic

Allogeneic If the source of donor cells is from a donor other than the patient, it is termed allogeneic. Autologous donor cells come from the patient. Syngeneic donor cells are from an identical twin. Homogenic is not a type of stem cell transplant.

Which type of hematopoietic stem cell transplantation (HSCT) is characterized by cells from a donor other than the patient?

Allogenic

Which is a sign or symptom of septic shock?

Altered mental status

Which of the following is a sign or symptoms of septic shock?

Altered mental status

Which is a sign or symptom of septic shock?

Altered mental status Signs of septic shock include altered mental status, cool and clammy skin, decreased urine output, and hypotension.

The nurse is conducting a screening for familial predisposition of cancer. Which of the following should the nurse note as a possible indication of hereditary cancer syndrome?

An aunt and uncle diagnosed with cancer

Which of the following is a term used to describe the process by which a new blood supply is formed?

Angiogenesis

Which of the following is a term used to describe the process by which a new blood supply is formed? a) Apoptosis b) Angiogenesis c) Carcinogenesis d) Mitosis

Angiogenesis Angiogenesis is the process by which a new blood supply is formed. Apoptosis is the innate cellular process of programmed cell death. Mitosis is the phase of the cell cycle in which cell division occurs. Carcinogenesis is the process by which cancer arises.

17. The clinic nurse is caring for a 42-year-old male oncology patient. He complains of extreme fatigue and weakness after his first week of radiation therapy. Which response by the nurse would best reassure this patient? A) These symptoms usually result from radiation therapy; however, we will continue to monitor your laboratory and x-ray studies. B) These symptoms are part of your disease and are an unfortunately inevitable part of living with cancer. C) Try not to be concerned about these symptoms. Every patient feels this way after having radiation therapy. D) Even though it is uncomfortable, this is a good sign. It means that only the cancer cells are dying.

Ans:A

18. A 16-year-old female patient experiences alopecia resulting from chemotherapy, prompting the nursing diagnoses of disturbed body image and situational low self- esteem. What action by the patient would best indicate that she is meeting the goal of improved body image and self-esteem? A) The patient requests that her family bring her makeup and wig. B) The patient begins to discuss the future with her family. C) The patient reports less disruption from pain and discomfort. D) The patient cries openly when discussing her disease.

Ans:A

19. A 50-year-old man diagnosed with leukemia will begin chemotherapy. What would the nurse do to combat the most common adverse effects of chemotherapy? A) Administer an antiemetic. B) Administer an antimetabolite. C) Administer a tumor antibiotic. D) Administer an anticoagulant.

Ans:A

2. A nurse who works in an oncology clinic is assessing a patient who has arrived for a 2- month follow-up appointment following chemotherapy. The nurse notes that the patient's skin appears yellow. Which blood tests should be done to further explore this clinical sign? A) Liver function tests (LFTs) B) Complete blood count (CBC) C) Platelet count D) Blood urea nitrogen and creatinine

Ans:A

30. You are caring for a patient who has just been told that her stage IV colon cancer has recurred and metastasized to the liver. The oncologist offers the patient the option of surgery to treat the progression of this disease. What type of surgery does the oncologist offer? A) Palliative B) Reconstructive C) Salvage D) Prophylactic

Ans:A

33. The home health nurse is performing a home visit for an oncology patient discharged 3 days ago after completing treatment for non-Hodgkin lymphoma. The nurse's assessment should include examination for the signs and symptoms of what complication? A) Tumor lysis syndrome (TLS) B) Syndrome of inappropriate antiduretic hormone (SIADH) C) Disseminated intravascular coagulation (DIC) D) Hypercalcemia

Ans:A

36. The hospice nurse has just admitted a new patient to the program. What principle guides hospice care? A) Care addresses the needs of the patient as well as the needs of the family. B) Care is focused on the patient centrally and the family peripherally. C) The focus of all aspects of care is solely on the patient. D) The care team prioritizes the patient's physical needs and the family is responsible for the patient's emotional needs.

Ans:A

37. A 60-year-old patient with a diagnosis of prostate cancer is scheduled to have an interstitial implant for high-dose radiation (HDR). What safety measure should the nurse include in this patient's subsequent plan of care? A) Limit the time that visitors spend at the patient's bedside. B) Teach the patient to perform all aspects of basic care independently. C) Assign male nurses to the patient's care whenever possible. D) Situate the patient in a shared room with other patients receiving brachytherapy.

Ans:A

6. The nurse is caring for a patient who is to begin receiving external radiation for a malignant tumor of the neck. While providing patient education, what potential adverse effects should the nurse discuss with the patient? A) Impaired nutritional status B) Cognitive changes C) Diarrhea D) Alopecia

Ans:A

7. While a patient is receiving IV doxorubicin hydrochloride for the treatment of cancer, the nurse observes swelling and pain at the IV site. The nurse should prioritize what action? A) Stopping the administration of the drug immediately B) Notifying the patient's physician C) Continuing the infusion but decreasing the rate D) Applying a warm compress to the infusion site

Ans:A

23. The nurse is describing some of the major characteristics of cancer to a patient who has recently received a diagnosis of malignant melanoma. When differentiating between benign and malignant cancer cells, the nurse should explain differences in which of the following aspects? Select all that apply. A) Rate of growth B) Ability to cause death C) Size of cells D) Cell contents E) Ability to spread

Ans:A, B, E

13. You are caring for an adult patient who has developed a mild oral yeast infection following chemotherapy. What actions should you encourage the patient to perform? Select all that apply. A) Use a lip lubricant. B) Scrub the tongue with a firm-bristled toothbrush. C) Use dental floss every 24 hours. D) Rinse the mouth with normal saline. E) Eat spicy food to aid in eradicating the yeast.

Ans:A, C, D

1. The public health nurse is presenting a health-promotion class to a group at a local community center. Which intervention most directly addresses the leading cause of cancer deaths in North America? A) Monthly self-breast exams B) Smoking cessation C) Annual colonoscopies D) Monthly testicular exams

Ans:B

12. A nurse is creating a plan of care for an oncology patient and one of the identified nursing diagnoses is risk for infection related to myelosuppression. What intervention addresses the leading cause of infection-related death in oncology patients? A) Encourage several small meals daily. B) Provide skin care to maintain skin integrity. C) Assist the patient with hygiene, as needed. D) Assess the integrity of the patient's oral mucosa regularly.

Ans:B

21. An oncology nurse educator is providing health education to a patient who has been diagnosed with skin cancer. The patient's wife has asked about the differences between normal cells and cancer cells. What characteristic of a cancer cell should the educator cite? A) Malignant cells contain more fibronectin than normal body cells. B) Malignant cells contain proteins called tumor-specific antigens. C) Chromosomes contained in cancer cells are more durable and stable than those of normal cells. D) The nuclei of cancer cells are unusually large, but regularly shaped.

Ans:B

22. A patient's most recent diagnostic imaging has revealed that his lung cancer has metastasized to his bones and liver. What is the most likely mechanism by which the patient's cancer cells spread? A) Hematologic spread B) Lymphatic circulation C) Invasion D) Angiogenesis

Ans:B

24. A 54-year-old has a diagnosis of breast cancer and is tearfully discussing her diagnosis with the nurse. The patient states, ìThey tell me my cancer is malignant, while my coworker's breast tumor was benign. I just don't understand at all.î When preparing a response to this patient, the nurse should be cognizant of what characteristic that distinguishes malignant cells from benign cells of the same tissue type? A) Slow rate of mitosis of cancer cells B) Different proteins in the cell membrane C) Differing size of the cells D) Different molecular structure in the cells

Ans:B

31. The nurse is caring for a patient with an advanced stage of breast cancer and the patient has recently learned that her cancer has metastasized. The nurse enters the room and finds the patient struggling to breath and the nurse's rapid assessment reveals that the patient's jugular veins are distended. The nurse should suspect the development of what oncologic emergency? A) Increased intracranial pressure B) Superior vena cava syndrome (SVCS) C) Spinal cord compression D) Metastatic tumor of the neck

Ans:B

34. The nurse is admitting an oncology patient to the unit prior to surgery. The nurse reads in the electronic health record that the patient has just finished radiation therapy. With knowledge of the consequent health risks, the nurse should prioritize assessments related to what health problem? A) Cognitive deficits B) Impaired wound healing C) Cardiac tamponade D) Tumor lysis syndrome

Ans:B

39. A patient with a diagnosis of gastric cancer has been unable to tolerate oral food and fluid intake and her tumor location precludes the use of enteral feeding. What intervention should the nurse identify as best meeting this patient's nutritional needs? A) Administration of parenteral feeds via a peripheral IV B) TPN administered via a peripherally inserted central catheter C) Insertion of an NG tube for administration of feeds D) Maintaining NPO status and IV hydration until treatment completion

Ans:B

8. A patient newly diagnosed with cancer is scheduled to begin chemotherapy treatment and the nurse is providing anticipatory guidance about potential adverse effects. When addressing the most common adverse effect, what should the nurse describe? A) Pruritis (itching) B) Nausea and vomiting C) Altered glucose metabolism D) Confusion

Ans:B

14. The nurse on a bone marrow transplant unit is caring for a patient with cancer who is preparing for HSCT. What is a priority nursing diagnosis for this patient? A) Fatigue related to altered metabolic processes B) Altered nutrition: less than body requirements related to anorexia C) Risk for infection related to altered immunologic response D) Body image disturbance related to weight loss and anorexia

Ans:C

20. A 58-year-old male patient has been hospitalized for a wedge resection of the left lower lung lobe after a routine chest x-ray shows carcinoma. The patient is anxious and asks if he can smoke. Which statement by the nurse would be most therapeutic? A) Smoking is the reason you are here. B) The doctor left orders for you not to smoke. C) You are anxious about the surgery. Do you see smoking as helping? D) Smoking is OK right now, but after your surgery it is contraindicated.

Ans:C

28. A public health nurse has formed an interdisciplinary team that is developing an educational program entitled Cancer: The Risks and What You Can Do About Them. Participants will receive information, but the major focus will be screening for relevant cancers. This program is an example of what type of health promotion activity? A) Disease prophylaxis B) Risk reduction C) Secondary prevention D) Tertiary prevention

Ans:C

3. The school nurse is teaching a nutrition class in the local high school. One student states that he has heard that certain foods can increase the incidence of cancer. The nurse responds, ìResearch has shown that certain foods indeed appear to increase the risk of cancer.î Which of the following menu selections would be the best choice for potentially reducing the risks of cancer? A) Smoked salmon and green beans B) Pork chops and fried green tomatoes C) Baked apricot chicken and steamed broccoli D) Liver, onions, and steamed peas

Ans:C

32. The hospice nurse is caring for a patient with cancer in her home. The nurse has explained to the patient and the family that the patient is at risk for hypercalcemia and has educated them on that signs and symptoms of this health problem. What else should the nurse teach this patient and family to do to reduce the patient's risk of hypercalcemia? A) Stool softeners are contraindicated. B) Laxatives should be taken daily C) Consume 2-4L of fluid daily D) Restrict calcium intake

Ans:C

35. An oncology patient has just returned from the postanesthesia care unit after an open hemicolectomy. This patient's plan of nursing care should prioritize which of the following? A) Assess the patient hourly for signs of compartment syndrome. B) Assess the patient's fine motor skills once per shift. C) Assess the patient's wound for dehiscence every 4 hours. D) Maintain the patient's head of bed at 45 degrees or more at all times.

Ans:C

4. Traditionally, nurses have been involved with tertiary cancer prevention. However, an increasing emphasis is being placed on both primary and secondary prevention. What would be an example of primary prevention? A) Yearly Pap tests B) Testicular self-examination C) Teaching patients to wear sunscreen D) Screening mammograms

Ans:C

5. The nurse is caring for a 39-year-old woman with a family history of breast cancer. She requested a breast tumor marking test and the results have come back positive. As a result, the patient is requesting a bilateral mastectomy. This surgery is an example of what type of oncologic surgery? A) Salvage surgery B) Palliative surgery C) Prophylactic surgery D) Reconstructive surgery

Ans:C

9. A patient on the oncology unit is receiving carmustine, a chemotherapy agent, and the nurse is aware that a significant side effect of this medication is thrombocytopenia. Which symptom should the nurse assess for in patients at risk for thrombocytopenia? A) Interrupted sleep pattern B) Hot flashes C) Epistaxis (nose bleed) D) Increased weight

Ans:C

26. The nurse is performing an initial assessment of an older adult resident who has just relocated to the long-term care facility. During the nurse's interview with the patient, she admits that she drinks around 20 ounces of vodka every evening. What types of cancer does this put her at risk for? Select all that apply. A) Malignant melanoma B) Brain cancer C) Breast cancer D) Esophageal cancer E) Liver cancer

Ans:C, D, E

10. The nurse is orienting a new nurse to the oncology unit. When reviewing the safe administration of antineoplastic agents, what action should the nurse emphasize? A) Adjust the dose to the patient's present symptoms. B) Wash hands with an alcohol-based cleanser following administration. C) Use gloves and a lab coat when preparing the medication. D) Dispose of the antineoplastic wastes in the hazardous waste receptacle.

Ans:D

11. A nurse provides care on a bone marrow transplant unit and is preparing a female patient for a hematopoietic stem cell transplantation (HSCT) the following day. What information should the nurse emphasize to the patient's family and friends? A) Your family should likely gather at the bedside in case there's a negative outcome. B) Make sure she doesn't eat any food in the 24 hours before the procedure. C) Wear a hospital gown when you go into the patient's room. D) Do not visit if you've had a recent infection.

Ans:D

15. An oncology nurse is caring for a patient who has developed erythema following radiation therapy. What should the nurse instruct the patient to do? A) Periodically apply ice to the area. B) Keep the area cleanly shaven. C) Apply petroleum jelly to the affected area. D) Avoid using soap on the treatment area.

Ans:D

16. The nurse is caring for a patient has just been given a 6-month prognosis following a diagnosis of extensive stage small-cell lung cancer. The patient states that he would like to die at home, but the team believes that the patient's care needs are unable to be met in a home environment. What might you suggest as an alternative? A) Discuss a referral for rehabilitation hospital. B) Panel the patient for a personal care home. C) Discuss a referral for acute care. D) Discuss a referral for hospice care.

Ans:D

25. An oncology patient will begin a course of chemotherapy and radiation therapy for the treatment of bone metastases. What is one means by which malignant disease processes transfer cells from one place to another? A) Adhering to primary tumor cells B) Inducing mutation of cells of another organ C) Phagocytizing healthy cells D) Invading healthy host tissues

Ans:D

29. A 62-year-old woman diagnosed with breast cancer is scheduled for a partial mastectomy. The oncology nurse explained that the surgeon will want to take tissue samples to ensure the disease has not spread to adjacent axillary lymph nodes. The patient has asked if she will have her lymph nodes dissected, like her mother did several years ago. What alternative to lymph node dissection will this patient most likely undergo? A) Lymphadenectomy B) Needle biopsy C) Open biopsy D) Sentinel node biopsy

Ans:D

38. An oncology patient has begun to experience skin reactions to radiation therapy, prompting the nurse to make the diagnosis Impaired Skin Integrity: erythematous reaction to radiation therapy. What intervention best addresses this nursing diagnosis? A) Apply an ice pack or heating pad PRN to relieve pain and pruritis B) Avoid skin contact with water whenever possible C) Apply phototherapy PRN D) Avoid rubbing or scratching the affected area

Ans:D

40. An oncology nurse is contributing to the care of a patient who has failed to respond appreciably to conventional cancer treatments. As a result, the care team is considering the possible use of biologic response modifiers (BRFs). The nurse should know that these achieve a therapeutic effect by what means? A) Promoting the synthesis and release of leukocytes B) Focusing the patient's immune system exclusively on the tumor C) Potentiating the effects of chemotherapeutic agents and radiation therapy D) Altering the immunologic relationship between the tumor and the patient

Ans:D

A client complains of sporadic epigastric pain, yellow skin, nausea, vomiting, weight loss, and fatigue. Suspecting gallbladder disease, the physician orders a diagnostic workup, which reveals gallbladder cancer. Which nursing diagnosis is appropriate for this client?

Anticipatory grieving

A client complains of sporadic epigastric pain, yellow skin, nausea, vomiting, weight loss, and fatigue. Suspecting gallbladder disease, the physician orders a diagnostic workup, which reveals gallbladder cancer. Which nursing diagnosis is appropriate for this client? Disturbed body image Anticipatory grieving Impaired swallowing Chronic low self-esteem

Anticipatory grieving

A client complains of sporadic epigastric pain, yellow skin, nausea, vomiting, weight loss, and fatigue. Suspecting gallbladder disease, the physician orders a diagnostic workup, which reveals gallbladder cancer. Which nursing diagnosis is appropriate for this client? a) Disturbed body image b) Chronic low self-esteem c) Anticipatory grieving d) Impaired swallowing

Anticipatory grieving

A client complains of sporadic epigastric pain, yellow skin, nausea, vomiting, weight loss, and fatigue. Suspecting gallbladder disease, the physician orders a diagnostic workup, which reveals gallbladder cancer. Which nursing diagnosis is appropriate for this client?

Anticipatory grieving Anticipatory grieving is an appropriate nursing diagnosis for this client because few clients with gallbladder cancer live more than 1 year after diagnosis. Impaired swallowing isn't associated with gallbladder cancer. Although surgery typically is done to remove the gallbladder and, possibly, a section of the liver, it isn't disfiguring and doesn't cause Disturbed body image. Chronic low self-esteem isn't an appropriate nursing diagnosis at this time because the diagnosis has just been made.

A client complains of sporadic epigastric pain, yellow skin, nausea, vomiting, weight loss, and fatigue. Suspecting gallbladder disease, the physician orders a diagnostic workup, which reveals gallbladder cancer. Which nursing diagnosis is appropriate for this client? a) Disturbed body image b) Anticipatory grieving c) Impaired swallowing d) Chronic low self-esteem

Anticipatory grieving Explanation: Anticipatory grieving is an appropriate nursing diagnosis for this client because few clients with gallbladder cancer live more than 1 year after diagnosis. Impaired swallowing isn't associated with gallbladder cancer. Although surgery typically is done to remove the gallbladder and, possibly, a section of the liver, it isn't disfiguring and doesn't cause Disturbed body image. Chronic low self-esteem isn't an appropriate nursing diagnosis at this time because the diagnosis has just been made.

To combat the most common adverse effects of chemotherapy, a nurse should administer an:

Antiemetic

5-Fluorouracil (5FU) is classified as which type of antineoplastic agent? a) Nitrosoureas b) Mitotic spindle poisons c) Antimetabolite d) Alkylating

Antimetabolite

5-Fluorouracil (5FU) is classified as which type of antineoplastic agent?

Antimetabolite -5-FU is an antimetabolite. An example of an alkylating agent is nitrogen mustard. A nitrosourea is streptozocin. A mitotic spindle poison is vincristine (VCR).

5-Fluorouracil (5FU) is classified as which type of antineoplastic agent?

Antimetabolite 5-FU is an antimetabolite. An example of an alkylating agent is nitrogen mustard. A nitrosourea is streptozocin. A mitotic spindle poison is vincristine (VCR).

Which of the following is a term used to describe the process of programmed cell death?

Apoptosis

Which of the following is a term used to describe the process of programmed cell death?

Apoptosis Apoptosis is the innate cellular process of programmed cell death. Mitosis is the phase of the cell cycle in which cell division occurs. Carcinogenesis is the process by which cancer arises. Angiogenesis is the process by which a new blood supply is formed.

Which action by the nurse is most appropriate when a client demonstrates subcutaneous emphysema along the suture line or chest dressing 2 hours after chest surgery? Report the finding to the physician immediately. Record the observation. Apply a compression dressing to the area. Measure the client's pulse oximetry.

Apply a compression dressing to the area.

A client has been receiving chemotherapy. Upon assessing the client during morning rounds, the nurse notes the client is now bleeding from intravenous and venipuncture sites. Stool is positive for occult blood. The client is requesting to sit in a chair for a meal. The nurse implements the following interventions: (Select all that apply.) Check intake and output records. Assist the client to a chair. Apply pressure to the bleeding sites. Assess level of consciousness. Monitor vital signs once a shift.

Apply pressure to the bleeding sites. Assess level of consciousness. Check intake and output records.

Which nursing intervention should be incorporated into the plan of care to manage the delayed clotting process in a client with leukemia?

Apply prolonged pressure to needle sites or other sources of external bleeding

A nurse caring for a client who has hemophilia is getting ready to take the client's vital signs. What should the nurse do before taking a blood pressure?

Ask if taking a blood pressure has ever produced bleeding under the skin or in the arm joints.

A client with cancer is receiving chemotherapy and reports to the nurse that his mouth is painful and he has difficulty ingesting food. The nurse does which of the following: Rinses the client's mouth with alcohol-based mouthwash every 2 hours Teaches the client to floss his teeth once every 24 hours Asks the client to open his mouth to facilitate inspection of the oral mucosa Consults with the healthcare provider about use of nystatin (Mycostatin) Instructs the client to brush the teeth with a soft toothbrush

Asks the client to open his mouth to facilitate inspection of the oral mucosa Consults with the healthcare provider about use of nystatin (Mycostatin) Instructs the client to brush the teeth with a soft toothbrush

A client has been receiving chemotherapy. Upon assessing the client during morning rounds, the nurse notes the client is now bleeding from intravenous and venipuncture sites. Stool is positive for occult blood. The client is requesting to sit in a chair for a meal. The nurse implements the following interventions:

Assess level of consciousness. Apply pressure to the bleeding sites. Check intake and output records.

A client has been receiving chemotherapy. Upon assessing the client during morning rounds, the nurse notes the client is now bleeding from intravenous and venipuncture sites. Stool is positive for occult blood. The client is requesting to sit in a chair for a meal. The nurse implements the following interventions: (Select all that apply.) - Assess level of consciousness. - Apply pressure to the bleeding sites. - Assist the client to a chair. - Check intake and output records.

Assess level of consciousness. Apply pressure to the bleeding sites. Check intake and output records. -The client may be experiencing disseminated intravascular coagulation (DIC) following the cancer experience and chemotherapy treatment. When the nurse notes the client is experiencing unexpected and abnormal bleeding, the nurse will assess level of consciousness (the client can be bleeding in the brain) and intake and output records (the client may experience decreased urinary output as a result of poor renal perfusion). The nurse applies pressure to venipuncture sites to decrease bleeding. The nurse will assess vital signs more frequently than once a shift. The nurse minimizes client activities to decrease risk for injury.

Which type of vaccine uses the client's own cancer cells, which are killed and prepared for injection back into the client?

Autologous

Which type of vaccine uses the patient's own cancer cells that are prepared for injection back into the patient?

Autologous

Which type of vaccine uses the patient's own cancer cells that are prepared for injection back into the patient? a) Prophylactic b) Autologous c) Therapeutic d) Allogeneic

Autologous Autologous vaccines are made from the patient's own cancer cells, which are obtained during diagnostic biopsy or surgery. Prophylactic vaccines, such as polio vaccine, are given to prevent people from developing a disease. Therapeutic vaccines are given to kill existing cancer cells and to provide long-lasting immunity against further cancer development. Allogeneic vaccines are made from cancer cells that are obtained from other people who have a specific type of cancer.

Which of the following advice does the nurse offer clients who are undergoing unsealed radiation therapy to reduce exposure? Avoid drinking plenty of fluids. Avoid eating for 3 hours after therapy. Avoid kissing and sexual contact. Avoid applying skin moisturizers.

Avoid kissing and sexual contact.

The nurse is providing client teaching for a client undergoing chemotherapy. What dietary modifications should the nurse advise?

Avoid spicy and fatty foods

The nurse is providing client teaching for a client undergoing chemotherapy. What dietary modifications should the nurse advise?

Avoid spicy and fatty foods.

The nurse is providing client teaching for a client undergoing chemotherapy. What dietary modifications should the nurse advise? Eat wholesome meals. Eat warm or hot foods. Avoid spicy and fatty foods. Avoid intake of fluids.

Avoid spicy and fatty foods.

The nurse is providing client teaching for a client undergoing chemotherapy. What dietary modifications should the nurse advise?

Avoid spicy and fatty foods. The nurse advises a client undergoing chemotherapy to avoid hot and very cold liquids and spicy and fatty foods. The nurse also encourages the client to have small meals and appropriate fluid intake.

Following surgery for adenocarcinoma, the client learns the tumor stage is T3, N1, M0. What treatment mode will the nurse anticipate? A) No further treatment is indicated. B) Adjuvant therapy is likely. C) Palliative care is likely. D) Repeat biopsy is needed before treatment begins.

B

The nurse is evaluating bloodwork results of a client with cancer who is receiving chemotherapy. The client's platelet count is 60,000/mm3. Which is an appropriate nursing action?

Avoiding the use of products containing aspirin

The nurse is evaluating bloodwork results of a client with cancer who is receiving chemotherapy. The client's platelet count is 60,000/mm3. Which is an appropriate nursing action?

Avoiding the use of products containing aspirin Clients with a platelet count of 60,000/mm3 are at mild risk for bleeding. Appropriate nursing interventions include avoiding the use of products such as aspirin that may interfere with the client's clotting systems; avoiding taking temperature rectally and administering suppositories; providing the client with an electric shaver for shaving; and avoiding commercial mouthwashes because of their potential to dry out oral mucosa, which can lead to cracking and bleeding.

The nurse is evaluating bloodwork results of a patient with cancer who is receiving chemotherapy. The patient's platelet count is 60,000/mm3. Which of the following is an appropriate nursing action?

Avoiding use of products containing aspirin

The nurse is evaluating bloodwork results of a patient with cancer who is receiving chemotherapy. The patient's platelet count is 60,000/mm3. Which of the following is an appropriate nursing action? a) Providing commercial mouthwash to patient b) Providing patient with a razor to shave c) Avoiding use of products containing aspirin d) Taking patient's temperature rectally

Avoiding use of products containing aspirin Explanation: Patients with a platelet count of 60,000/mm3 are at mild risk for bleeding. Appropriate nursing interventions include avoiding use of products such as aspirin that may interfere with the patient's clotting systems; avoiding taking temperature rectally and administering suppositories; providing patient with an electric shaver for shaving; and avoiding commercial mouthwashes due to their potential to dry out oral mucosa, which can lead to cracking and bleeding.

A client receiving external radiation to the left thorax to treat lung cancer has a nursing diagnosis of Risk for impaired skin integrity. Which intervention should be part of this client's care plan?

Avoiding using soap on the irradiated areas

A client receiving external radiation to the left thorax to treat lung cancer has a nursing diagnosis of Risk for impaired skin integrity. Which intervention should be part of this client's care plan? Avoiding using soap on the irradiated areas Wearing a lead apron during direct contact with the client Applying talcum powder to the irradiated areas daily after bathing Removing thoracic skin markings after each radiation treatment

Avoiding using soap on the irradiated areas

A client receiving external radiation to the left thorax to treat lung cancer has a nursing diagnosis of Risk for impaired skin integrity. Which intervention should be part of this client's care plan?

Avoiding using soap on the irradiated areas Because external radiation commonly causes skin irritation, the nurse should wash the irradiated area with water only and leave the area open to air. No soaps, deodorants, lotions, or powders should be applied. A lead apron is unnecessary because no radiation source is present in the client's body or room. Skin in the area to be irradiated is marked to position the radiation beam as precisely as possible; skin markings must not be removed.

A client receiving external radiation to the left thorax to treat lung cancer has a nursing diagnosis of Risk for impaired skin integrity. Which intervention should be part of this client's care plan? a) Wearing a lead apron during direct contact with the client b) Avoiding using soap on the irradiated areas c) Removing thoracic skin markings after each radiation treatment d) Applying talcum powder to the irradiated areas daily after bathing

Avoiding using soap on the irradiated areas Explanation: Because external radiation commonly causes skin irritation, the nurse should wash the irradiated area with water only and leave the area open to air. No soaps, deodorants, lotions, or powders should be applied. A lead apron is unnecessary because no radiation source is present in the client's body or room. Skin in the area to be irradiated is marked to position the radiation beam as precisely as possible; skin markings must not be removed

A client has just received stem cell transplantation as treatment for leukemia. What are the post procedural nursing interventions for clients receiving any form of stem cell transplantation? A. closely monitor the client for at least 4 weeks B. closely monitor the client for at least 3 weeks C. closely monitor the client for at least 3 days D. closely monitor the client for at least 5 months

B

A client is scheduled for a nerve-sparing prostatectomy. The emotional spouse confides in the nurse that the client will not talk about the cancer and/or upcoming surgery. Which nursing diagnosis will the nurse choose as primary diagnosis for this client? A) Sexual Dysfunction B) Fear C) Knowledge Deficit D) Ineffective Coping

B

A client who is being treated for bladder cancer expresses his concern of passing cancer to his wife during intercourse. Which is the best response by the nurse? A) "You should avoid intercourse until your cancer is cured." B) "Cancer is not transferred from person to person via direct contact." C) "I understand you are concerned about your wife, but don't worry." D) "Perhaps you should have your sperm tested for presence of cancer cells."

B

A client with a 4-cm breast mass is scheduled for biopsy with frozen section followed by lumpectomy and possible mastectomy. The client asks the nurse, "Why can't the doctor tell me specifically whether I will need to have my entire breast removed"? Which is the best response from the nurse? A) "The doctor will know which surgery is required, once the tumor is exposed." B) "The frozen section will determine presence of cancer and type of surgery required." C) "You need to trust your doctor to provide you with the best of care." D) "You seem anxious about your upcoming surgery."

B

An elderly client has been diagnosed with metastatic cancer and has a poor prognosis of survival. The family asks the nurse for advice on whether to tell the client of the diagnosis or to keep it quiet. Which is the best response from the nurse? A) "I wouldn't tell, if I were you." B) "In my experience, clients who know are more likely to be involved with their plan of care." C) "The shock of learning the diagnosis may be too much stress for an elderly person." D) "This is a private concern that should include the physician, not me."

B

Cancer is the second leading cause of death in the United States, second only to heart disease. Half of all men and one third of all women will develop cancer during their lifetimes. Which types of cancer have the highest prevalence among both men and women? A. skin and brain B. lung and colon C. colon and skin D. lung and skin

B

The client is scheduled for a breast lump excision and sentinel node biopsy. What should the nurse know in planning care for the client with a negative biopsy report? A) A lump excision is not necessary. B) A wide excision of lump will be performed. C) The lump and all axillary lymph nodes will be excised. D) The entire breast and all regional lymph nodes will be excised.

B

The nurse is caring for a thyroid cancer client following oral radioactive iodine treatment. Which teaching point is most important? A) Shield your throat area when near others. B) Flush the toilet twice after every use. C) Prepare food separately from family members. D) Use disposable utensils for the next month.

B

The nurse working on a bone marrow unit knows that it is a priority to monitor which of the following in a client who has just undergone a stem cell transplant? A) Monitor the client's toilet patterns. B) Monitor the client closely to prevent infection. C) Monitor the client's physical condition. D) Monitor the client's heart rate.

B

What should the nurse tell a client who is about to benign chemotherapy and is anxious about hair loss? A. the hair will grow back the same as it was before treatment B. the client should consider getting a wig or cap prior to beginning treatment C. alopecia related to chemotherapy is relatively uncommon D. the hair will grow back within 2 months post therapy

B

Which patient instructions are necessary for a client receiving systemic internal radiation therapy who is being discharged? Select all that apply. A. use separate hand towels B. wash hands carefully following toileting C. avoid all physical contact with family members D. drink only clear fluids and water E. was laundry separately

B

You are providing client teaching for a client undergoing chemotherapy. What dietary modifications should you advise? A) Eat wholesome meals. B) Avoid spicy and fatty foods. C) Avoid intake of fluids. D) Eat warm or hot foods.

B

A client returns for the results of diagnostic testing for metastasized brain cancer. The client has been battling this disease for more than 30 years and has become symptomatic. The latest diagnostic study, a CT scan, reveals the extent of the malignancy. What information can be gleaned from a CT scan? Select all that apply. A. cancer staging B. tumor density, shape, and size C. blood vessels feeding the tumor

B, C

During a client's examination and consultation, the physician keeps telling the client, "You have an abdominal neoplasm." Which statements accurately paraphrase the physician's statement? Select all that apply. A. "You have abdominal cancer." B. "You have an abdominal tumor." C. "You have a new growth of abnormal tissue in your abdomen." D. "You have an abdominal malignancy."

B, C

A client without symptoms or complaints receives a diagnosis of prostate cancer after a routine physical. What factors contributed to this diagnosis? Select all that apply. A. environmental factors B. client history C. tumor markers D. risk factors

B, C, D

During a routine physical examination, a client confides that he is terrified of developing prostate cancer like his father. What are the warning signs of prostate cancer? Select all that apply. A. easy bruising B. weak and interrupted urine flow C. continuous pain in the lower back D. pain in the upper thighs E. blood in the urine

B, C, D

A client is recovering from a craniotomy with tumor debulking. Which comment by the client indicates to the nurse a correct understanding of what the surgery entailed? A. "Thank goodness the tumor is contained and curable." B. "I guess the doctor could not remove the entire tumor." C. "I am so glad the doctor was able to remove the entire tumor." D. "I will be glad to finally be done with treatments for this thing."

B. "I guess the doctor could not remove the entire tumor." Debulking is a reference made when a tumor cannot be completely removed, often due to its extension far into healthy tissue. Without complete removal, this is not a cure and, the cancer cells will continue to replicate and require adjuvant therapies to prevent further invasion. The physician, not the nurse, will need to clarify the details of the surgery.

Following a BMT the patient should be monitored for at least A. 3 days. B. 3 months. C. 5 months. D. 4 weeks.

B. 3 months. After a BMT, the nurse closely monitors the patient for at least 3 months because complications related to the transplant are still possible, and infections are very common.

Which type of vaccine uses the patient's own cancer cells that are prepared for injection back into the patient? A. Prophylactic B. Autologous C. Allogeneic D. Therapeutic

B. Autologous -Autologous vaccines are made from the patient's own cancer cells, which are obtained during diagnostic biopsy or surgery. Prophylactic vaccines, such as polio vaccine, are given to prevent people from developing a disease. Therapeutic vaccines are given to kill existing cancer cells and to provide long-lasting immunity against further cancer development. Allogeneic vaccines are made from cancer cells that are obtained from other people who have a specific type of cancer.

A client undergoes a biopsy of a suspicious lesion. The biopsy report classifies the lesion according to the TNM staging system as follows: TIS, N0, M0. What does this classification mean? A. Carcinoma in situ, no demonstrable metastasis of the regional lymph nodes, and ascending degrees of distant metastasis B. Carcinoma in situ, no abnormal regional lymph nodes, and no evidence of distant metastasis C. No evidence of primary tumor, no abnormal regional lymph nodes, and no evidence of distant metastasis D. Can't assess tumor or regional lymph nodes and no evidence of metastasis

B. Carcinoma in situ, no abnormal regional lymph nodes, and no evidence of distant metastasis TIS, N0, M0 denotes carcinoma in situ, no abnormal regional lymph nodes, and no evidence of distant metastasis. No evidence of primary tumor, no abnormal regional lymph nodes, and no evidence of distant metastasis is classified as T0, N0, M0. If the tumor and regional lymph nodes can't be assessed and no evidence of metastasis exists, the lesion is classified as TX, NX, M0. A progressive increase in tumor size, no demonstrable metastasis of the regional lymph nodes, and ascending degrees of distant metastasis is classified as T1, T2, T3, or T4; N0; and M1, M2, or M3.

The nurse is conducting a community education program using the American Cancer Society's colorectal screening and prevention guidelines. The nurse determines that the participants understand the teaching when they identify that people over the age of 50 should have which of the following screening tests every 10 years? A. Papanicolaou (Pap) B. Colonoscopy C. Prostate-specific antigen (PSA) D. Fecal occult blood test

B. Colonoscopy -Recommendations for screening for colorectal cancer include screening colonoscopies every 10 years. Fecal occult blood tests should be completed annually in people over age 50. PSA tests for prostate-specific antigen is used as a screening tool for prostate cancer. A Pap test is a screening tool for cervical cancer.

A patient will be having an endoscopic procedure with a diagnostic biopsy. What type of biopsy does the nurse explain will remove an entire piece of suspicious tissue? A. Incisional biopsy B. Excisional biopsy C. Needle biopsy D. Punch biopsy

B. Excisional biopsy Excisional biopsy is most frequently used for small, easily accessible tumors of the skin, breast, and upper or lower gastrointestinal and upper respiratory tracts. In many cases, the surgeon can remove the entire tumor as well as the surrounding marginal tissues. The removal of normal tissue beyond the tumor area decreases the possibility that residual microscopic malignant cells may lead to a recurrence of the tumor. Incisional biopsy is performed if the tumor mass is too large to be removed. In this case, a wedge of tissue from the tumor is removed for analysis. Needle biopsy is performed to sample suspicious masses that are easily and safely accessible, such as some masses in the breasts, thyroid, lung, liver, and kidney. A core needle biopsy uses a specially designed needle to obtain a small core of tissue that permits histologic analysis.

The client is receiving a vesicant anti neo plastic for treatment of cancer. Which assessment finding would require the nurse to take immediate action? A. Nausea and vomiting B. Extravasation C. Bone pain D. Stomatitis

B. Extravasation The nurse needs to monitor IV administration of anti neo plastics (especially vesicants) to prevent tissue necrosis to blood vessels, skin, muscles, and nerves. Stomatitis, nausea/vomiting, and bone pain can be symptoms of the disease process or treatment mode but does not require immediate action.

When caring for a client who is receiving external beam radiation, which is the key point for the nurse to incorporate into the plan of care? A. Time, distance, and shielding B. Inspect the skin frequently. C. Avoid showering or washing over skin markings. D. The use of disposable utensils and wash cloths

B. Inspect the skin frequently. Inspecting the skin frequently will allow early identification and intervention of skin problems associated with external radiation therapy. The external markings should not be removed, but clients may shower and lightly wash over the skin. Time, distance, and shielding are key in the management of sealed, internal radiation therapy and not external beam radiation. The use of disposable utensils and care items would be important when caring for clients following systemic, unsealed, internal radiation therapy.

What does the nurse understand is the rationale for administering allopurinol for a patient receiving chemotherapy? A. It prevents alopecia. B. It lowers serum and uric acid levels. C. It stimulates the immune system against the tumor cells. D. It treats drug-related anemia.

B. It lowers serum and uric acid levels. Adequate hydration, diuresis, alkalinization of the acid crystals, and administration of allopurinol (Zyloprim) may be used to prevent renal toxicity.

Which of the following does a nurse thoroughly evaluate before a hematopoietic stem cell transplant (HSCT) procedure? a) Family history b) Blood studies c) Allergy history d) Drug history

Blood studies

A client with cancer is being evaluated for possible metastasis. What is one of the most common metastasis sites for cancer cells? A. Reproductive tract B. Liver C. Colon D. White blood cells (WBCs)

B. Liver The liver is one of the five most common cancer metastasis sites. The others are the lymph nodes, lung, bone, and brain. The colon, reproductive tract, and WBCs are occasional metastasis sites.

The client is diagnosed with a benign brain tumor. Which of the following features of a benign tumor is of most concern to the nurse? A. Random, rapid growth of the tumor B. Tumor pressure against normal tissues C. Emission of abnormal proteins D. Cells colonizing to distant body parts

B. Tumor pressure against normal tissues Benign tumors grow more slowly than malignant tumors and do not emit tumor-specific antigens or proteins. Benign tumors do not metastasize to distant sites. Benign tumors can compress tissues as it grows, which can result in impaired organ functioning.

A serum sodium level lower than 110 mEq/L is associated with A. anorexia B. seizure. C. weight gain. D. myalgia.

B. seizure. Serum sodium levels lower than 110 mEq/L is associated with seizures, abnormal reflexes, papilledema, coma, and death. Anorexia, weight gain, and myalgia are associated with serum sodium levels lower than 120 mEq/L.

When teaching a client with iron deficiency anemia about appropriate food choices, the nurse encourages the client to increase the dietary intake of which foods?

Beans, dried fruits, and leafy, green vegetables

Mrs. Unger is a 53-year-old woman who was diagnosed with breast cancer following a process that began with abnormal screen mammography results. Mrs. Unger, her oncologist, and surgeon have agreed on a mastectomy as treatment and have discussed the importance of rigorously assessing whether her cancer has metastasized. What action will best detect possible metastasis of Mrs. Unger's breast cancer?

Biopsy of the axillary lymph nodes

The most common cause of iron deficiency anemia in men and postmenopausal women is:

Bleeding

The patient diagnosed with thrombocytopenia is at risk for which of the following adverse effects:

Bleeding The patient diagnosed with thrombocytopenia is at risk for bleeding and infection until blood cell counts return to normal. Headache, diminished reflexes, and stomatitis are not adverse effects related to the diagnosis.

Which should a nurse thoroughly evaluate before a bone marrow transplant (BMT) procedure?

Blood Studies

Which does a nurse thoroughly evaluate before a hematopoietic stem cell transplant (HSCT) procedure? Family History Drug history Blood Studies Allergy history

Blood studies

Which of the following does a nurse thoroughly evaluate before a bone marrow transplant (BMT) procedure?

Blood studies

Which of the following does a nurse thoroughly evaluate before a hematopoietic stem cell transplant (HSCT) procedure?

Blood studies

Which of the following does a nurse thoroughly evaluate before a bone marrow transplant (BMT) procedure?

Blood studies -Before the BMT procedure, the nurse thoroughly evaluates the patient's physical condition; organ function; nutritional status; complete blood studies, including assessment for past antigen exposure, such as HIV, hepatitis, or cytomegalovirus; and psychosocial status. Before a BMT procedure, the nurse need not evaluate patient's family, drug, or allergy history.

Which should a nurse thoroughly evaluate before a bone marrow transplant (BMT) procedure?

Blood studies Before the BMT procedure, the nurse thoroughly evaluates the client's physical condition; organ function; nutritional status; complete blood studies, including assessment for past exposure to antigens such as HIV, hepatitis, or cytomegalovirus; and psychosocial status. Before a BMT procedure, the nurse need not evaluate the client's family, drug, or allergy history.

Which does a nurse thoroughly evaluate before a hematopoietic stem cell transplant (HSCT) procedure?

Blood studies Before the HSCT procedure, the nurse thoroughly evaluates the client's physical condition; organ function; nutritional status; complete blood studies, including assessment for past exposure to antigens such as HIV, hepatitis, or cytomegalovirus; and psychosocial status. Before an HSCT procedure, the nurse need not evaluate client's family, drug, or allergy history.

The nurse is providing education to a client with cancer radiation treatment options. The nurse determines that the client understands the teaching when the client states that which type of radiation aims to protect healthy tissue during the treatment?

Brachytherapy In internal radiation, or brachytherapy, a dose of radiation is delivered to a localized area inside the body through the use of an implant. With this type of therapy, the farther the tissue is from the radiation source, the lower the dose. This helps to protect normal tissue from the radiation therapy.

The nurse is providing education to a patient with cancer radiation treatment options. The nurse determines that the patient understands when he or she states that which of the following types of radiation is aimed at protecting healthy tissue during the treatment? a) Brachytherapy b) Teletherapy c) Proton therapy d) External

Brachytherapy Explanation: In internal radiation, or brachytherapy, a dose of radiation is delivered to a localized area inside the body by use of an implant. With this type of therapy, the further the tissue is from the radiation source, the lower the dose. This helps to protect normal tissue from the radiation therapy.

The nurse is providing education to a patient with cancer radiation treatment options. The nurse determines that the patient understands when he or she states that which of the following types of radiation is aimed at protecting healthy tissue during the treatment? a) External b) Brachytherapy c) Teletherapy d) Proton therapy

Brachytherapy Explanation: In internal radiation, or brachytherapy, a dose of radiation is delivered to a localized area inside the body by use of an implant. With this type of therapy, the further the tissue is from the radiation source, the lower the dose. This helps to protect normal tissue from the radiation therapy

A cancer client makes the following statement to the nurse: "I guess I will tell my doctor to forego the chemotherapy. I do not want to be throwing up all the time. I would rather die." Which of the following facts supports the use of chemotherapy for this client? A) Nausea and vomiting are only a factor for the first 24 hours after treatment. B) Most clients believe the discomfort is well worth the cure for cancer. C) Chemotherapy treatment can be adjusted to optimize effects while limiting adverse effects. D) Clinical trials are opening up new cancer treatments all the time.

C

A client has been diagnosed with a neoplasm and is seeking further information and possible treatment. The primary care physician described the neoplasm as "insidious." What does the word "insidious" mean? A. aggressive B. terminal C. slow-growing D. life-threatening

C

A client is receiving IV administration of the vesicant doxorubicin when the nurse detects signs of extravasation. After stopping administration, what nursing action would be taken to treat this complication? A. apply warm compress B. administer thiosulfate as ordered C. elevate and rest the extremity D. apply ice to the site

C

A client with advanced cancer makes the following comment to the nurse: "Why are you bathing me? I am going to die no matter what." What is the most appropriate response of the nurse? A) "A bath will make you feel better." B) "Do you want to skip the bath today?" C) "Would you like to talk about what you are feeling?" D) "I can give you some medicine to make you feel better."

C

A client with cancer stage T4, N3, M1 is ordered morphine sulfate 4 mg, subcutaneous every 3 to 4 hours. Two hours after the last injection, the client rings the call bell to report a pain scale of 9. Which is the appropriate action by the nurse? A) Explain to the client that the medication can only be given every 3 to 4 hours. B) Ignore the call bell and stall until it is time to administer the next dose. C) Notify the physician of the breakthrough pain in an attempt to obtain additional orders. D) Ask the family to attempt diversion activities until the next dose can be given.

C

A newly diagnosed cancer client is crying and states the following to the nurse: "I promised God that I will be a better person if I can just get better." What is the appropriate assessment of this comment by the nurse? A) The client is just trying to protect self from potential loss. B) Anger directed toward nursing staff is not unusual in dealing with cancer clients. C) The cancer is viewed as a punishment from past actions. D) Loss is inevitable so client is making final plans.

C

A young client has been diagnosed with cancer that has metastasized to the lungs. During client education, the client's mother asks about tumor staging and its relation to her child's condition. What stage would the nurse expect this client's tumor to be assigned? A. stage 0 B. stage III C. stage IV D. stage I

C

Cancer has many characteristics. What is one of the most discouraging characteristics of cancer? A) Large size B) Carcinogenesis C) Metastasis D) Slow growth

C

The client is diagnosed with a benign brain tumor. Which of the following features of a benign tumor is of most concern to the nurse? A) Random, rapid growth of the tumor B) Cells colonizing to distant body parts C) Tumor pressure against normal tissues D) Emission of abnormal proteins

C

The nurse performs a breast exam on a client and finds a firm, non-moveable lump in the upper outer quadrant of the right breast that the client reports was not there 3 weeks ago. What does this finding suggest? A) Normal finding B) Benign fibrocystic disease C) Malignant tumor D) Malignant tumor with metastasis to surrounding tissue

C

The physician recommends that you have your daughter vaccinated with HPV vaccine. What is this vaccine for? A) Help prevent lung cancer B) Help prevent breast cancer C) Help prevent cervical cancer D) Help prevent leukemia

C

Which of the following can be considered carcinogens? A) Parasites B) Medical procedures C) Dietary substances D) Infective genes

C

The nurse is caring for a client who is scheduled for chemotherapy. Which is the best statement the nurse can make about the client experiencing chemotherapy-induced alopecia? A. "New hair growth will return without any change to color or texture." B. "Wigs can be used after the chemotherapy is completed." C. "The hair loss is temporary." D. "Clients with alopecia will have delay in grey hair."

C. "The hair loss is temporary." Alopecia associated with chemotherapy is usually temporary and will return after the therapy is completed. New hair growth may return unchanged, but there is no guarantee and color, texture, and quality of hair may be changed. There is no correlation between chemotherapy and delay in greying of hair. Use of wigs, scarves, and head coverings can be used by clients at any time during treatment plan.

The oncology nurse is giving chemotherapy to a client in a short stay area. The client confides that they are very depressed. The nurse recognizes depression as which of the following? A. A psychiatric diagnosis everyone has at one time or another. B. An aberrant psychologic reaction to the chemotherapy. C. A normal reaction to the diagnosis of cancer. D. A side effect of the neoplastic drugs.

C. A normal reaction to the diagnosis of cancer. Clients have many reactions, ranging from anxiety, fear, and depression to feelings of guilt related to viewing cancer as a punishment for past actions or failure to practice a healthy life-style. They also may express anger related to the diagnosis and their inability to be in control. While depression is a psychiatric diagnosis not everyone has the diagnosis sometime in their life; depression is not a side effect of the neoplastic drugs nor is it an aberrant psychologic reaction to the chemotherapy.

After cancer chemotherapy, a client experiences nausea and vomiting. The nurse should assign highest priority to which intervention? A. Withholding fluids for the first 4 to 6 hours after chemotherapy administration B. Encouraging rhythmic breathing exercises C. Administering metoclopramide and dexamethasone as ordered D. Serving small portions of bland food

C. Administering metoclopramide and dexamethasone as ordered The nurse should assign highest priority to administering an antiemetic, such as metoclopramide, and an anti-inflammatory agent, such as dexamethasone, because it may reduce the severity of chemotherapy-induced nausea and vomiting. This intervention, in turn, helps prevent dehydration, a common complication of chemotherapy. Serving small portions of bland food, encouraging rhythmic breathing exercises, and withholding fluids for the first 4 to 6 hours are less likely to achieve this outcome.

Which type of hematopoietic stem cell transplantation (HSCT) is characterized by cells from a donor other than the patient? A. Autologous B. Homogenic C. Allogeneic D. Syngeneic

C. Allogeneic If the source of donor cells is from a donor other than the patient, it is termed allogeneic. Autologous donor cells come from the patient. Syngeneic donor cells are from an identical twin. Homogenic is not a type of stem cell transplant.

Which oncologic emergency involves the accumulation of fluid in the pericardial space? A. Disseminated intravascular coagulation (DIC) B. Syndrome of inappropriate antidiuretic hormone release (SIADH) C. Cardiac tamponade D. Tumor lysis syndrome

C. Cardiac tamponade -Cardiac tamponade is an accumulation of fluid in the pericardial space. DIC is a complex disorder of coagulation and fibrinolysis that results in thrombosis and bleeding. SIADH is a result of the failure in the negative feedback mechanism that normally regulates the release of antidiuretic hormone (ADH). Tumor lysis syndrome is a rapidly developing oncologic emergency that results from the rapid release of intracellular contents as a result of radiation- or chemotherapy-induced cell destruction of large or rapidly growing cancers such as leukemia.

The nurse is caring for a client newly diagnosed with cancer. Which of the following therapies is used to treat something other than cancer? A. Chemotherapy B. Radiation therapy C. Electroconvulsive therapy D. Surgery

C. Electroconvulsive therapy Cancer is frequently treated with a combination of therapies using standardized protocols. Three basic methods used to treat cancer are surgery, radiation therapy, and chemotherapy. Electroconvulsive therapy (ECT) is a method of treatment for mental distress or illness.

What intervention should the nurse provide to reduce the incidence of renal damage when a patient is taking a chemotherapy regimen? A. Withhold medication when the blood urea nitrogen level exceeds 20 mg/dL. B. Take measures to acidify the urine and prevent uric acid crystallization. C. Encourage fluid intake to dilute the urine. D. Limit fluids to 1,000 mL daily to prevent accumulation of the drug's end products after cell lysis.

C. Encourage fluid intake to dilute the urine. -The nurse should ensure adequate fluid hydration before, during, and after drug administration and assess intake and output. Adequate fluid volume dilutes drug levels, which can help prevent renal damage.

A nurse is administering daunorubicin through a peripheral I.V. line when the client complains of burning at the insertion site. The nurse notes no blood return from the catheter and redness at the I.V. site. The client is most likely experiencing which complication? A. Flare B. Erythema C. Extravasation D. Thrombosis

C. Extravasation The client is exhibiting signs of extravasation, which occurs when the medication leaks into the surrounding tissues and causes swelling, burning, or pain at the injection site. Erythema is redness of the skin that results from skin irritation. Flare is a spreading of redness that occurs as a result of drawing a pointed instrument across the skin. Thrombosis is the formation of clot within the vascular system.

The nurse is working with a patient who has had an allohematopoietic stem cell transplant (HSCT) and notices a diffuse rash and diarrhea. The nurse contacts the physician to report that the patient has symptoms of which of the following? A. Acute leukopenia B. Nadir C. Graft-versus-host disease D. Metastasis

C. Graft-versus-host disease Graft-versus-host disease is a major cause of morbidity and mortality in patients who have had allogeneic transplant. Clinical manifestation of the disease include diffuse rash that progresses to blistering and desquamation, and mucosal inflammation of the eyes and the entire GI tract with subsequent diarrhea, abdominal pain, and hepatomegaly.

Which of the following is a characteristic of a malignant tumor? A. It grows by expansion. B. It demonstrates cells that are well differentiated. C. It gains access to the blood and lymphatic channels. D. It is usually slow growing.

C. It gains access to the blood and lymphatic channels. By this mechanism, the tumor metastasizes to other areas of the body. Cells of malignant tumors are undifferentiated. Malignant tumors demonstrate variable rate of growth; however, the more anaplastic the tumor, the faster its growth. A malignant tumor grows at the periphery and sends out processes that infiltrate and destroy surrounding tissues.

The nurse is caring for a patient undergoing an incisional biopsy. Which of the following statements does the nurse understand is true about an incisional biopsy? A. It is used to remove the cancerous cells using a needle. B. It removes an entire lesion and surrounding tissue. C. It removes a wedge of tissue for diagnosis. D. It treats cancer with lymph node involvement.

C. It removes a wedge of tissue for diagnosis. The three most common biopsy methods are excisional, incisional, and needle. In an incisional biopsy, a wedge of tissue is removed from the tumor and analyzed. In an excisional biopsy, the surgeon removes the tumor and the surrounding marginal tissues. Needle aspiration biopsy involves aspirating tissue fragments through a needle guided into the cancer cells.

The nurse performs a breast exam on a client and finds a firm, non-moveable lump in the upper outer quadrant of the right breast that the client reports was not there 3 weeks ago. What does this finding suggest? A. Benign fibrocystic disease B. Malignant tumor with metastasis to surrounding tissue C. Malignant tumor D. Normal finding

C. Malignant tumor A fast-growing lump is suggestive of a malignant tumor. Metastasis can only be determined by cytology, not by palpation.

Which type of surgery is utilized in an attempt to relieve complications of cancer? A. Reconstructive B. Prophylactic C. Palliative D. Salvage

C. Palliative Palliative surgery is performed to relieve complications of cancer. Prophylactic surgery involves removing non-vital tissues or organs that are likely to develop cancer. Reconstructive surgery may follow curative or radical surgery and is carried out in an attempt to improve function or obtain a more desirable cosmetic effect. Salvage surgery is an additional treatment option that uses an extensive surgical approach to treat the local recurrence of a cancer after the use of a less extensive primary approach.

An important nursing function is monitoring factors that may indicate that bleeding is occurring. One serum indicator is a (an): A. Neutrophil count of 60%. B. Reticulocyte count of 1%. C. Platelet count of 60,000/mm3. D. Lymphocyte count of 30%.

C. Platelet count of 60,000/mm3. Thrombocytopenia, a decrease in the circulating platelet count, is the most common cause of bleeding in patients with cancer and is usually defined as a count less than 100,000/mm3. The risk of bleeding increases as the count drops lower. The risk of spontaneous bleeding occurs with a count of less than 20,000/mm3.

A patient is admitted for an excisional biopsy of a breast lesion. What intervention should the nurse provide for the care of this patient? A. Clarify information provided by the physician. B. Provide aseptic care to the incision postoperatively. C. Provide time for the patient to discuss her concerns. D. Counsel the patient about the possibility of losing her breast.

C. Provide time for the patient to discuss her concerns. Patients who are undergoing surgery for the diagnosis or treatment of cancer may be anxious about the surgical procedure, possible findings, postoperative limitations, changes in normal body functions, and prognosis. The patient and family require time and assistance to process the possible changes and outcomes resulting from the surgery (Chart 15-4). The nurse serves as the patient advocate and liaison and encourages the patient and family to take an active role in decision making when possible. If the patient or family asks about the results of diagnostic testing and surgical procedures, the nurse's response is guided by the information that was conveyed previously. The nurse may be asked to explain and clarify information for patients and families that was provided initially but was not grasped because of anxiety and overwhelming feelings. It is important that the nurse, as well as other members of the health care team, provide information that is consistent from one clinician to another.

A nurse is caring for a client receiving chemotherapy. Which assessment finding places the client at the greatest risk for an infection? A. Ate 75% of all meals during the day B. Temperature of 98.3° F (36.8° C) C. Stage 3 pressure ulcer on the left heel D. White blood cell (WBC) count of 9,000 cells/mm3

C. Stage 3 pressure ulcer on the left heel -A stage 3 pressure ulcer is a break in the skin's protective barrier, which could lead to infection in a client who is receiving chemotherapy. The WBC count and temperature are within normal limits. Eating 75% of meals is normal and doesn't increase the client's risk for infection. A client who is malnourished is at a greater risk for infection.

A cancer client makes the following statement to the nurse: "I guess I will tell my doctor to forego the chemotherapy. I do not want to be throwing up all the time. I would rather die."Which of the following facts supports the use of chemotherapy for this client?

Chemotherapy treatment can be adjusted to optimize effects while limiting adverse effects.

A nurse is administering daunorubicin (DaunoXome) to a patient with lung cancer. Which situation requires immediate intervention? A. The client states he is nauseous. B. The client begins to shiver. C. The I.V. site is red and swollen. D. The laboratory reports a white blood cell (WBC) count of 1,000/mm3.

C. The I.V. site is red and swollen. A red, swollen I.V. site indicates possible infiltration. Daunorubicin is a vesicant chemotherapeutic agent and can be very damaging to tissue if it infiltrates. The nurse should immediately stop the medication, apply ice to the site, and notify the physician. Although nausea, WBC count of 1,000/mm3, and shivering require interventions, these findings aren't a high priority at this time.

A client reports feeling tired, cold, and short of breath at times. Assessment reveals tachycardia and reduced energy. What would the nurse expect the physician to order?

CBC

A client undergoes a biopsy of a suspicious lesion. The biopsy report classifies the lesion according to the TNM staging system as follows: TIS, N0, M0. What does this classification mean?

Carcinoma in situ, no abnormal regional lymph nodes, and no evidence of distant metastasis

A client undergoes a biopsy of a suspicious lesion. The biopsy report classifies the lesion according to the TNM staging system as follows: Tis, N0, M0. What does this classification mean?

Carcinoma in situ, no abnormal regional lymph nodes, and no evidence of distant metastasis

The nurse is conducting a community education program using the American Cancer Society's colorectal screening and prevention guidelines. The nurse determines that the participants understand the teaching when they identify that people over the age of 50 should have which screening test every 10 years?

Colonoscopy

A client undergoes a biopsy of a suspicious lesion. The biopsy report classifies the lesion according to the TNM staging system as follows: TIS, N0, M0. What does this classification mean?

Carcinoma in situ, no abnormal regional lymph nodes, and no evidence of distant metastasis TIS, N0, M0 denotes carcinoma in situ, no abnormal regional lymph nodes, and no evidence of distant metastasis. No evidence of primary tumor, no abnormal regional lymph nodes, and no evidence of distant metastasis is classified as T0, N0, M0. If the tumor and regional lymph nodes can't be assessed and no evidence of metastasis exists, the lesion is classified as TX, NX, M0. A progressive increase in tumor size, no demonstrable metastasis of the regional lymph nodes, and ascending degrees of distant metastasis is classified as T1, T2, T3, or T4; N0; and M1, M2, or M3.

Which occurs when fluid accumulates in the pericardial space and compresses the heart?

Cardiac tamponade

Which of the following occurs when there is accumulation of fluid in the pericardial space that compresses the heart?

Cardiac tamponade

Which of the following occurs when there is accumulation of fluid in the pericardial space that compresses the heart? a) Cardiac tamponade b) SIADH c) DIC d) Superior Vena Cava Syndrome (SVCS)

Cardiac tamponade

Which oncologic emergency involves the accumulation of fluid in the pericardial space?

Cardiac tamponade

A client returns to the surgeon's office for a report on a diagnostic procedure to determine the cell composition of the client's abdominal neoplasm. Which term is significant to indicate the likelihood of the tumor spreading? A. neoplasm B. lesion C. primary site D. benign

D

Which oncologic emergency involves the accumulation of fluid in the pericardial space?

Cardiac tamponade Cardiac tamponade is an accumulation of fluid in the pericardial space. DIC is a complex disorder of coagulation and fibrinolysis that results in thrombosis and bleeding. SIADH is a result of the failure in the negative feedback mechanism that normally regulates the release of antidiuretic hormone (ADH). Tumor lysis syndrome is a rapidly developing oncologic emergency that results from the rapid release of intracellular contents as a result of radiation- or chemotherapy-induced cell destruction of large or rapidly growing cancers such as leukemia.

The nurse is talking with a group of clients who are older than age 50 years about the recognition of colon cancer to access early intervention. What should the nurse inform the clients to report immediately to their primary care provider? Change in bowel habits Excess gas Daily bowel movements Abdominal cramping when having a bowel movement

Change in bowel habits

A cancer client makes the following statement to the nurse: "I guess I will tell my doctor to forego the chemotherapy. I do not want to be throwing up all the time. I would rather die."Which of the following facts supports the use of chemotherapy for this client?

Chemotherapy treatment can be adjusted to optimize effects while limiting adverse effects

A cancer client makes the following statement to the nurse: "I guess I will tell my doctor to forego the chemotherapy. I do not want to be throwing up all the time. I would rather die."Which of the following facts supports the use of chemotherapy for this client?

Chemotherapy treatment can be adjusted to optimize effects while limiting adverse effects. Chemotherapy is not one drug for all clients. The therapy can be specifically designed to optimize effects while limiting adverse effects with supplemental anti emetics to control the nausea and vomiting. It is true that nausea and vomiting are most prevalent in the first 24 hours after each chemotherapy treatment but does not eliminate the fears expressed by this client. No one can state the worth of any treatment, and a cure is never promised. Clinical trials open up new options for treatment, but the process is lengthy and is not a certainty for a client in need of immediate treatment.

The nurse is irrigating a client's colostomy when the client begins to report cramping. What is the appropriate action by the nurse? Discontinue the irrigation immediately. Change irrigation fluid to normal saline. Clamp the tubing and allow client to rest. Increase the rate of administration.

Clamp the tubing and allow client to rest.

The nurse should teach the patient who is being radiated about protecting his skin and oral mucosa. An important teaching point would be to tell the patient to:

Cleanse the skin with a mild soap, using his fingertips, not a rough wash cloth.

The nurse should teach the patient who is being radiated about protecting his skin and oral mucosa. An important teaching point would be to tell the patient to: Cleanse the skin with a mild soap, using his fingertips, not a rough wash cloth. Apply a small ice compress to the treated area afterward to decrease localized redness, post-radiation. Use an approved emollient 2 hours before the radiation to give the skin time to absorb the medication and provide a shield for damage. Use an ointment, after treatment, to decrease the feeling of burning, which may last for several hours.

Cleanse the skin with a mild soap, using his fingertips, not a rough wash cloth.

A client has just received stem cell transplantation as treatment for leukemia. What are the post procedural nursing interventions for clients receiving any form of stem cell transplantation? Closely monitor the client for atleast 3 months Closely monitor the client for atleast 3 days Closely monitor the client for atleast 4 weeks Closely monitor the client for atleast 5 months

Closely monitor the client for at least 3 months. - After stem cell transplantation, the nurse closely monitors the client for at least 3 months because complications related to the transplant are still possible and infections are very common.

A client diagnosed with acute myelocytic leukemia has been receiving chemotherapy. During the last 2 cycles of chemotherapy, the client developed severe thrombocytopenia requiring multiple platelet transfusions. The client is now scheduled to receive a third cycle. How can the nurse best detect early signs and symptoms of thrombocytopenia?

Closely observe the client's skin for petechiae and bruising.

A client diagnosed with acute myelocytic leukemia has been receiving chemotherapy. During the last 2 cycles of chemotherapy, the client developed severe thrombocytopenia requiring multiple platelet transfusions. The client is now scheduled to receive a third cycle. How can the nurse best detect early signs and symptoms of thrombocytopenia? Monitor daily platelet counts. Perform a cardiovascular assessment every 4 hours. Closely observe the client's skin for petechiae and bruising. Check the client's history for a congenital link to thrombocytopenia.

Closely observe the client's skin for petechiae and bruising.

The nurse is conducting a community education program using the American Cancer Society's colorectal screening and prevention guidelines. The nurse determines that the participants understand the teaching when they identify that people over the age of 50 should have which screening test every 10 years?

Colonoscopy Recommendations for screening for colorectal cancer include a screening colonoscopy every 10 years. Fecal occult blood tests should be completed annually in people over age 50. The test for PSA is used as a screening tool for prostate cancer. A Pap test is a screening tool for cervical cancer.

Which primary cancer treatment goal is prolonged survival and containment of cancer cell growth?

Control

Which primary cancer treatment goal is prolonged survival and containment of cancer cell growth? Control Cure Palliation Prevention

Control

Which primary cancer treatment goal is prolonged survival and containment of cancer cell growth? A. Control B. Palliation C. Cure D. Prevention

Control -The range of possible treatment goals may include complete eradication of malignant disease (cure), prolonged survival and containment of cancer cell growth (control), or relief of symptoms associated with the disease (palliation). Prevention is not a treatment goal when the patient has already been diagnosed with cancer. Prevention of metastasis to a secondary site may be a goal.

Which primary cancer treatment goal is prolonged survival and containment of cancer cell growth?

Control The range of possible treatment goals may include complete eradication of malignant disease (cure), prolonged survival and containment of cancer cell growth (control), or relief of symptoms associated with the disease (palliation). Prevention is not a treatment goal when the patient has already been diagnosed with cancer. Prevention of metastasis to a secondary site may be a goal.

Which primary cancer treatment goal is prolonged survival and containment of cancer cell growth? a) Palliation b) Control c) Cure d) Prevention

Control The range of possible treatment goals may include complete eradication of malignant disease (cure), prolonged survival and containment of cancer cell growth (control), or relief of symptoms associated with the disease (palliation). Prevention is not a treatment goal when the patient has already been diagnosed with cancer. Prevention of metastasis to a secondary site may be a goal.

A client who is receiving chemotherapy for esophageal cancer complains of "feeling sick to my stomach all the time." What is the best suggestion the nurse can make to help alleviate this client's nausea? A. drink more soft drinks B. consume more salty foods C. consume warm or hot foods D. eat low-fat foods

D

Which of the following is a type of procedure that uses liquid nitrogen to freeze tissue that causes cell destruction?

Cryoablation

Which procedure uses liquid nitrogen to freeze tissue, thereby destroying cells?

Cryoablation Cryoablation uses liquid nitrogen or a very cold probe to freeze tissue, causing cell destruction. Electrosurgery, chemosurgery, and laser surgery do not use liquid nitrogen to freeze tissue.

Which of the following is a type of procedure that uses liquid nitrogen to freeze tissue and cause cell destruction?

Cryosurgery

Which of the following is a type of procedure that uses liquid nitrogen to freeze tissue and cause cell destruction?

Cryosurgery Cryosurgery uses liquid nitrogen or a very cold probe to freeze tissue to cause cell destruction. Electrosurgery, chemosurgery, and laser surgery do not use liquid nitrogen to freeze tissue.

The drug interleukin-2 is an example of which type of biologic response modifier?

Cytokine

A bowel resection is scheduled for a client with the diagnosis of colon cancer with metastasis to the liver and bone. Which statement by the nurse best explains the purpose of the surgery? A) "Removing the tumor is a primary treatment for colon cancer." B) "This surgery will prevent further tumor growth." C) "Once the tumor is removed, cell pathology can be determined." D) "Tumor removal will promote comfort."

D

A client receives a diagnosis of chondrosarcoma. This means that the client has a : A. tumor that originated in the smooth muscle B. benign tumor of the cartilage C. tumor that originated in the bone D. malignant tumor

D

A side-effect of chemotherapy is renal damage. To prevent this, the nurse should:

Encourage fluid intake, if possible, to dilute the urine. To prevent renal damage, it is helpful to dilute the urine by increasing fluids as tolerated.

Based on the understanding of the effects of chemotherapy, the nurse would anticipate which of the following clinical findings in a client 2 weeks posttherapy? A) Change in hair color B) Elevated temperature C) Elevated white blood cells count D) Ease of bruising

D

Chemotherapy has been used for the past 3 months to treat a client with pancreatic cancer. The CA 19-9 levels are rising. Which explanation would the nurse attribute as the most likely cause? A) It is normal for this antigen to rise for up to 6 months. B) The client is having an adverse response to the chemotherapy. C) The chemotherapy is effectively destroying the cancer cells. D) The cancer is growing despite the chemotherapy treatment.

D

The client has finished the first round of chemotherapy. Which statement made by the client indicates a need for further teaching by the nurse? A) "I will eat clear liquids for the next 24 hours." B) "Hair loss may not occur until after the second round of therapy." C) "I will use birth control measures until after all treatment is completed." D) "I can continue taking my vitamins and herbs because they make me feel better."

D

The nurse is caring for a client with the diagnosis of colon cancer with metastasis to the liver. Which statement made by the client indicates an understanding of the diagnosis? A) "Once the colon tumor is removed, I will be fine." B) "I will be happy once all the cancer is cut out." C) "How could I be so unlucky to get cancer twice?" D) "My cancer has now spread to my liver."

D

The nurse is invited to present a teaching program to parents of school-age children. Which topic would be of greatest value for decreasing cancer risks? A) Pool and water safety B) Breast and testicular self-exams C) Handwashing and infection prevention D) Sun safety and use of sunscreen

D

The nurse knows that interferon agents are used in association with chemotherapy to produce which effects in the client? A) Suppression of the bone marrow B) Enhance action of the chemotherapy C) Decrease the need for additional adjuvant therapies D) Shorten the period of neutropenia

D

When caring for a client who is receiving external beam radiation, which is the key point for the nurse to incorporate into the plan of care? A) Time, distance, and shielding B) The use of disposable utensils and wash cloths C) Avoid showering or washing over skin markings. D) Inspect the skin frequently.

D

When the client complains of increased fatigue following radiotherapy, the nurse knows this is most likely to be related to which factor? A) The cancer is spreading. B) The cancer cells are dying in large numbers. C) Fighting off infection is an exhausting venture. D) Radiation can result in myelosuppression.

D

Which of the following advice does the nurse offer clients who are undergoing unsealed radiation therapy to reduce exposure? A) Avoid drinking plenty of fluids. B) Avoid eating for 3 hours after therapy. C) Avoid applying skin moisturizers. D) Avoid kissing and sexual contact.

D

Which of the following laboratory findings, would be identified by the nurse as the greatest risk for a cancer client scheduled for implantable port? A) White blood cell count 10,800/mm3 B) Hemoglobin 10 g/dL C) Hematocrit 36.0% D) Platelet count 98,000/mm3

D

The nurse is assessing a 75-year-old woman who had a total hysterectomy when she was 30 years old and normal Pap test results for the past 10 years. The client asks about continuing the Pap test. The nurse states: A. "You will need to continue for the rest of your life." B. "You need to continue obtaining a Pap test for only the next 5 years." C. "You could have stopped immediately after your hysterectomy." D. "You may choose to discontinue this test."

D. "You may choose to discontinue this test." The American Cancer Society recommendations for women 70 years or older, who have had normal Pap tests for 10 years, and who have had a total hysterectomy may choose to stop cervical cancer screening as in a Pap test.

What disadvantages of chemotherapy should the patient be informed about prior to starting the regimen? A. It attacks cancer cells during their vulnerable phase. B. It functions against disseminated disease. C. It causes a systemic reaction. D. It targets normal body cells as well as cancer cells.

D. It targets normal body cells as well as cancer cells. -Chemotherapy agents affect both normal and malignant cells; therefore, their effects are often widespread, affecting many body systems.

Following surgery for adenocarcinoma, the client learns the tumor stage is T3,N1,M0. What treatment mode will the nurse anticipate? A. No further treatment is indicated. B. Repeat biopsy is needed before treatment begins. C. Palliative care is likely. D. Adjuvant therapy is likely.

D. Adjuvant therapy is likely. T3 indicates a large tumor size with N1 indicating regional lymph node involvement. Although M0 suggest no metastasis, following with adjuvant (chemotherapy or radiation therapy) treatment is indicated to prevent the spread of cancer outside the lymph to other organs. The tumor staging of stage IV is indicative of palliative care.

A patient is to receive Bacille Calmette-Guerin (BCG), a nonspecific biologic response modifier. Why would the patient receive this form of treatment? A. For skin cancer B. For cancer of the lungs C. For cancer of the breast D. For cancer of the bladder

D. For cancer of the bladder -Early investigations of the stimulation of the immune system involved nonspecific agents such as bacille Calmette-Guérin (BCG) and Corynebacterium parvum. When injected into the patient, these agents serve as antigens that stimulate an immune response. The hope is that the stimulated immune system will then eradicate malignant cells. Extensive animal and human investigations with BCG have shown promising results, especially in treating localized malignant melanoma. In addition, BCG bladder instillation is a standard form of treatment for localized bladder cancer (Polovich et al, 2009).

What foods should the nurse suggest that the patient consume less of in order to reduce nitrate intake because of the possibility of carcinogenic action? A. Eggs and milk B. Green, leafy vegetables C. Fish and poultry D. Ham and bacon

D. Ham and bacon Dietary substances that appear to increase the risk of cancer include fats, alcohol, salt-cured or smoked meats, nitrate and nitrite-containing foods, and red and processed meats. Nitrates are added to cured meats, such as ham and bacon.

The physician recommends that you have your daughter vaccinated with HPV vaccine. What is this vaccine for? A. Help prevent lung cancer B. Help prevent breast cancer C. Help prevent leukemia D. Help prevent cervical cancer

D. Help prevent cervical cancer The vaccines that are approved for use in the United States include the human papilloma virus (HPV), which may help prevent women from getting cervical cancer. There are no vaccines for the prevention of lung cancer, breast cancer, or leukemia.

For a client newly diagnosed with radiation-induced thrombocytopenia, the nurse should include which intervention in the care plan? A. Administering aspirin if the temperature exceeds 102° F (38.8° C) B. Placing the client in strict isolation C. Providing for frequent rest periods D. Inspecting the skin for petechiae once every shift

D. Inspecting the skin for petechiae once every shift Because thrombocytopenia impairs blood clotting, the nurse should inspect the client regularly for signs of bleeding, such as petechiae, purpura, epistaxis, and bleeding gums. The nurse should avoid administering aspirin because it may increase the risk of bleeding. Frequent rest periods are indicated for clients with anemia, not thrombocytopenia. Strict isolation is indicated only for clients who have highly contagious or virulent infections that are spread by air or physical contact.

The nurse at the clinic explains to the patient that the surgeon will be removing a mole on the patient's back that has the potential to develop into cancer. The nurse informs the patient that this is what type of procedure? A. Palliative B. Reconstructive C. Diagnostic D. Prophylactic

D. Prophylactic -Prophylactic surgery involves removing nonvital tissues or organs that are at increased risk of developing cancer. When surgical cure is not possible, the goals of surgical interventions are to relieve symptoms, make the patient as comfortable as possible, and promote quality of life as defined by the patient and family. Palliative surgery and other interventions are performed in an attempt to relieve complications of cancer, such as ulceration, obstruction, hemorrhage, pain, and malignant effusions (Table 15-6). Reconstructive surgery may follow curative or radical surgery in an attempt to improve function or obtain a more desirable cosmetic effect. Diagnostic surgery, or biopsy, is performed to obtain a tissue sample for histologic analysis of cells suspected to be malignant.

A patient is admitted for an excisional biopsy of a breast lesion. What intervention should the nurse provide for the care of this patient? A. Counsel the patient about the possibility of losing her breast. B. Clarify information provided by the physician. C. Provide aseptic care to the incision postoperatively. D. Provide time for the patient to discuss her concerns.

D. Provide time for the patient to discuss her concerns. Patients who are undergoing surgery for the diagnosis or treatment of cancer may be anxious about the surgical procedure, possible findings, postoperative limitations, changes in normal body functions, and prognosis. The patient and family require time and assistance to process the possible changes and outcomes resulting from the surgery (Chart 15-4). The nurse serves as the patient advocate and liaison and encourages the patient and family to take an active role in decision making when possible. If the patient or family asks about the results of diagnostic testing and surgical procedures, the nurse's response is guided by the information that was conveyed previously. The nurse may be asked to explain and clarify information for patients and families that was provided initially but was not grasped because of anxiety and overwhelming feelings. It is important that the nurse, as well as other members of the health care team, provide information that is consistent from one clinician to another.

What should the nurse tell a female client who is about to begin chemotherapy and anxious about losing her hair? A. Her hair will grow back within 2 months post therapy. B. Her hair will grow back the same as it was before treatment. C. Alopecia related to chemotherapy is relatively uncommon. D. She should consider getting a wig or cap before she loses her hair.

D. She should consider getting a wig or cap before she loses her hair. If hair loss is anticipated, purchase a wig, cap, or scarf before therapy begins. Alopecia develops because chemotherapy affects rapidly growing cells of the hair follicles. Hair usually begins to grow again within 4 to 6 months after therapy. Clients should know that new growth may have a slightly different color and textures

The physician is attending to a 72-year-old patient with a malignant brain tumor. The physician recommends immediate radiation therapy. Which of the following is a reason for the physician's recommendation? A. To remove the tumor from the brain B. To destroy marginal tissues C. To analyze involved lymph nodes D. To prevent the formation of new cancer cells

D. To prevent the formation of new cancer cells -Radiation therapy helps in preventing cellular growth. It may be used to cure the cancer or to control malignancy when the tumor cannot be removed or when lymph node involvement is present; also, it can be used prophylactically to prevent spread. Biopsy is used for analyzing the lymph nodes or for destroying the surrounding tissues around the tumor.

Your patient has recently completed her first round of chemotherapy in the treatment of lung cancer. When reviewing this morning's blood work, what findings would be suggestive of myelosuppression?

Decreased platelets and red blood cells

Your patient has recently completed her first round of chemotherapy in the treatment of lung cancer. When reviewing this morning's blood work, what findings would be suggestive of myelosuppression? Increased creatinine and blood urea nitrogen (BUN) Decreased platelets and red blood cells Increased white blood cells and c-reactive protein (CRP) Decreased sodium levels and decreased potassium levels

Decreased platelets and red blood cells

What are considered carcinogens?

Dietary substances

The nurse is caring for a client with external bleeding. What is the nurse's priority intervention?

Direct pressure

The nurse assesses that extravasation of a chemotherapy agent has occurred. What should the initial action of the nurse be?

Discontinue the infusion.

The nurse assesses that extravasation of a chemotherapy agent has occurred. What should the initial action of the nurse be?

Discontinue the infusion. If extravasation is suspected, the medication administration is stopped immediately, and depending on the drug, the nurse may attempt to aspirate any remaining drug from the extravasation site. The other actions listed may be appropriate to perform, but should occur after discontinuing the infusion.

While administering cisplatin (Platinol-AQ) to a client, the nurse assesses swelling at the insertion site. The first action of the nurse is to

Discontinue the intravenous medication

While administering cisplatin (Platinol-AQ) to a client, the nurse assesses swelling at the insertion site. The first action of the nurse is to

Discontinue the intravenous medication.

While administering cisplatin (Platinol-AQ) to a client, the nurse assesses swelling at the insertion site. The first action of the nurse is to Administer a neutralizing solution. Aspirate as much of the fluid as possible. Discontinue the intravenous medication. Apply a warm compress.

Discontinue the intravenous medication.

While administering cisplatin (Platinol-AQ) to a client, the nurse assesses swelling at the insertion site. The first action of the nurse is to

Discontinue the intravenous medication. -If extravasation of a chemotherapeutic medication is suspected, the nurse immediately stops the medication. Depending on the drug, the nurse may then attempt to aspirate any remaining drug, apply a warm or cold compress, administer a neutralizing solution, or all these measures.

While administering cisplatin (Platinol-AQ) to a client, the nurse assesses swelling at the insertion site. The first action of the nurse is to

Discontinue the intravenous medication. If extravasation of a chemotherapeutic medication is suspected, the nurse immediately stops the medication. Depending on the drug, the nurse may then attempt to aspirate any remaining drug, apply a warm or cold compress, administer a neutralizing solution, or all these measures.

A nurse is assessing a patient's stoma on postoperative day 3. The nurse notes that the stoma has a shiny appearance and a bright red color. How should the nurse best respond to this assessment finding? Irrigate the ostomy to clear a possible obstruction. Document that the stoma appears healthy and well perfused. Contact the primary care provider to report this finding. Document a nursing diagnosis of Impaired Skin Integrity.

Document that the stoma appears healthy and well perfused.

A client with anemia has been admitted to the medical-surgical unit. Which assessment findings are characteristic of iron deficiency anemia?

Dyspnea, tachycardia, and pallor

Based on the understanding of the effects of chemotherapy, the nurse would anticipate which of the following clinical findings in a client 2 weeks post therapy? Change in hair color Elevated white blood cells count Ease of bruising Elevated temperature

Ease of bruising

Based on the understanding of the effects of chemotherapy, the nurse would anticipate which of the following clinical findings in a client 2 weeks post therapy? Elevated temperature Ease of bruising Change in hair color Elevated white blood cells count

Ease of bruising

Based on the understanding of the effects of chemotherapy, the nurse would anticipate which of the following clinical findings in a client 2 weeks post therapy?

Ease of bruising The effects of chemotherapy can include myelosuppression, resulting in anemia or bleeding tendencies, as exhibited in ease in bruising. Elevated temperature and WBCs are signs of infection and are anticipated findings after chemotherapy treatment. Re growth of hair after alopecia can result in change of hair color but not anticipated 2 weeks post treatment.

The nurse is evaluating the client's risk for cancer and recommends changes when the client states she?

Eats red meat such as steaks or burgers every day

What intervention should the nurse provide to reduce the incidence of renal damage when a patient is taking a chemotherapy regimen?

Encourage fluid intake to dilute the urine. The nurse should ensure adequate fluid hydration before, during, and after drug administration and assess intake and output. Adequate fluid volume dilutes drug levels, which can help prevent renal damage.

The nurse is providing an educational presentation on dietary recommendations for reducing the risk of cancer. Which of the following food selections would demonstrate a good understanding of the information provided in the presentation? Select all that apply.

Egg white omelet with spinach and mushrooms Steamed broccoli and carrots Turkey breast on whole wheat bread Foods high in fat and those that are smoked or preserved with salt or nitrates are associated with increased cancer risks. An omelet made of egg whites and vegetables is a healthy low fat selection as are steamed broccoli/carrots and turkey breast on whole grain bread. A salad can be a healthy selection but Caesar salads contain much fat from the dressing and addition of cheeses and fried chicken. Salmon that is not smoked would be a good selection. Quiche usually contains high-fat milk, crème, eggs, and cheese.

The nurse is caring for a client newly diagnosed with cancer. Which of the following therapies is used to treat something other than cancer?

Electroconvulsive therapy

You are an oncology nurse caring for a client who tells you that their tastes have changed. They go on to say that "meat tastes bad". What is a nursing intervention to increase protein intake for a client with taste changes?

Encourage cheese and sandwiches

You are an oncology nurse caring for a client who tells you that their tastes have changed. They go on to say that "meat tastes bad". What is a nursing intervention to increase protein intake for a client with taste changes?

Encourage cheese and sandwiches.

You are an oncology nurse caring for a client who tells you that their tastes have changed. They go on to say that "meat tastes bad". What is a nursing intervention to increase protein intake for a client with taste changes?

Encourage cheese and sandwiches. -The nurse encourages the clients with taste changes to eat cheese and sandwiches. Encouraging the client to take in the maximum amount of fluids does not increase protein intake. The nurse advises the client to drink protein beverages. Sucking on hard candies during treatment does not increase protein intake.

You are an oncology nurse caring for a client who tells you that their tastes have changed. They go on to say that "meat tastes bad". What is a nursing intervention to increase protein intake for a client with taste changes?

Encourage cheese and sandwiches. The nurse encourages the clients with taste changes to eat cheese and sandwiches. Encouraging the client to take in the maximum amount of fluids does not increase protein intake. The nurse advises the client to drink protein beverages. Sucking on hard candies during treatment does not increase protein intake.

What intervention should the nurse provide to reduce the incidence of renal damage when a patient is taking a chemotherapy regimen?

Encourage fluid intake to dilute the urine.

A side-effect of chemotherapy is renal damage. To prevent this, the nurse should

Encourage fluid intake, if possible, to dilute the urine.

A nurse is providing care for a patient whose recent colostomy has contributed to a nursing diagnosis of Disturbed Body Image Related to Colostomy. What intervention best addresses this diagnosis? Encourage the patient to conduct online research into colostomies. Emphasize the fact that the colostomy was needed to alleviate a much more serious health problem. Emphasize the fact that the colostomy is temporary measure and is not permanent. Engage the patient in the care of the ostomy to the extent that the patient is willing.

Engage the patient in the care of the ostomy to the extent that the patient is willing.

A patient will be having an endoscopic procedure with a diagnostic biopsy. What type of biopsy does the nurse explain will remove an entire piece of suspicious tissue? Incisional biopsy Punch biopsy Needle biopsy Excisional biopsy

Excisional biopsy Excisional biopsy is most frequently used for small, easily accessible tumors of the skin, breast, and upper or lower gastrointestinal and upper respiratory tracts. In many cases, the surgeon can remove the entire tumor as well as the surrounding marginal tissues. The removal of normal tissue beyond the tumor area decreases the possibility that residual microscopic malignant cells may lead to a recurrence of the tumor. Incisional biopsy is performed if the tumor mass is too large to be removed. In this case, a wedge of tissue from the tumor is removed for analysis. Needle biopsy is performed to sample suspicious masses that are easily and safely accessible, such as some masses in the breasts, thyroid, lung, liver, and kidney. A core needle biopsy uses a specially designed needle to obtain a small core of tissue that permits histologic analysis.

A patient with uterine cancer is being treated with internal radiation therapy. What would the nurse's priority responsibility be for this patient?

Explain to the patient that she will continue to emit radiation while the implant is in place.

A patient with uterine cancer is being treated with internal radiation therapy. What would the nurse's priority responsibility be for this patient? Maintain as much distance as possible from the patient while in the room. Wear a lead apron when providing direct patient care. Explain to the patient that she will continue to emit radiation while the implant is in place. Alert family members that they should restrict their visiting to 5 minutes at any one time.

Explain to the patient that she will continue to emit radiation while the implant is in place.

The client is receiving a vesicant anti neo plastic for treatment of cancer. Which assessment finding would require the nurse to take immediate action?

Extra vasation

A nurse is administering daunorubicin through a peripheral I.V. line when the client complains of burning at the insertion site. The nurse notes no blood return from the catheter and redness at the I.V. site. The client is most likely experiencing which complication?

Extravasation

The client is receiving a vesicant antineoplastic for treatment of cancer. Which assessment finding would require the nurse to take immediate action?

Extravasation

A nurse is administering daunorubicin through a peripheral I.V. line when the client complains of burning at the insertion site. The nurse notes no blood return from the catheter and redness at the I.V. site. The client is most likely experiencing which complication?

Extravasation The client is exhibiting signs of extravasation, which occurs when the medication leaks into the surrounding tissues and causes swelling, burning, or pain at the injection site. Erythema is redness of the skin that results from skin irritation. Flare is a spreading of redness that occurs as a result of drawing a pointed instrument across the skin. Thrombosis is the formation of clot within the vascular system.

The client is receiving a vesicant antineoplastic for treatment of cancer. Which assessment finding would require the nurse to take immediate action? Extravasation Stomatitis Nausea and vomiting Bone pain

Extravasation The nurse needs to monitor IV administration of antineoplastics (especially vesicants) to prevent tissue necrosis to blood vessels, skin, muscles, and nerves. Stomatitis, nausea/vomiting, and bone pain can be symptoms of the disease process or treatment mode but does not require immediate action.

The nurse is performing a community screening for colorectal cancer. Which characteristic should the nurse include in the screening? Low-fat, low-protein, high-fiber diet Age younger than 40 years Familial polyposis History of skin cancer

Familial polyposis

A patient with uterine cancer is being treated with intracavitary radiation. The patient will emit radiation while the implant is in place. The nurse is aware of the precautions necessary for the provider of care and visitors. Which of the following are appropriate guidelines to follow? Select all that apply.

Family members should stand about 6 feet from the patient. Lead aprons should be worn to buffer the exposure. Visitors may stay for 30 minutes or less.

You are the nurse caring for a client with cancer. The client complains of pain and nausea. When assessed, you note that the client appears fearful. What other factor must you consider when a client with cancer indicates signs of pain, nausea, and fear? a) High cholesterol levels b) Ulceration c) Fatigue d) Infection

Fatigue Explanation: Clients with cancer experience fatigue, which is a side effect of cancer treatments that rest fails to relieve. The nurse must assess the client for other stressors that contribute to fatigue such as pain, nausea, fear, and lack of adequate support. The nurse works with other healthcare team members to treat the client's fatigue. The above indications do not contribute to infections, ulcerations, or high cholesterol levels.

A client diagnosed with cancer has their tumor staged and graded based on what?

How they tend to grow and the cell type

The nurse is caring for a thyroid cancer client following oral radioactive iodine treatment. Which teaching point is most important?

Flush the toilet twice after every use.

The nurse is caring for a thyroid cancer client following oral radioactive iodine treatment. Which teaching point is most important? Use disposable utensils for the next month. Flush the toilet twice after every use. Prepare food separately from family members. Shield your throat area when near others.

Flush the toilet twice after every use.

The nurse is caring for a thyroid cancer client following oral radioactive iodine treatment. Which teaching point is most important? a) Use disposable utensils for the next month. b) Prepare food separately from family members. c) Shield your throat area when near others. d) Flush the toilet twice after every use.

Flush the toilet twice after every use.

A patient is to receive Bacille Calmette-Guerin (BCG), a nonspecific biologic response modifier. Why would the patient receive this form of treatment? a) For cancer of the breast b) For cancer of the bladder c) For cancer of the lungs d) For skin cancer

For cancer of the bladder Explanation: Early investigations of the stimulation of the immune system involved nonspecific agents such as bacille Calmette-Guérin (BCG) and Corynebacterium parvum. When injected into the patient, these agents serve as antigens that stimulate an immune response. The hope is that the stimulated immune system will then eradicate malignant cells. Extensive animal and human investigations with BCG have shown promising results, especially in treating localized malignant melanoma. In addition, BCG bladder instillation is a standard form of treatment for localized bladder cancer (Polovich et al, 2009).

Which of the following is true about a malignant tumor?

Gains access to the blood and lymphatic channels

Following surgery for adenocarcinoma, the client learns the tumor stage is T3,N1,M0. What treatment mode will the nurse anticipate? Adjuvant therapy is likely. Palliative care is likely. No further treatment is indicated. Repeat biopsy is needed before treatment begins.

Grade 1 Grade I tumors, also known as well-differentiated tumors, closely resemble the tissue of origin in structure and function. In grade II, the tumor is moderately differentiated. Tumors in grade III are poorly differentiated (little resemblance to tissue of origin). Grade IV tumors is undifferentiated (unable to tell tissue of origin).

Which grade of tumor is also known as a well-differentiated tumor? Grade I Grade II Grade III Grade IV

Grade I

The nurse is working with a patient who has had an allohematopoietic stem cell transplant (HSCT) and notices a diffuse rash and diarrhea. The nurse contacts the physician to report that the patient has symptoms of which of the following?

Graft vs. host disease

The nurse is working with a patient who has had an allohematopoietic stem cell transplant (HSCT) and notices a diffuse rash and diarrhea. The nurse contacts the physician to report that the patient has symptoms of which of the following?

Graft-versus-host disease

The nurse is working with a patient who has had an allohematopoietic stem cell transplant (HSCT) and notices a diffuse rash and diarrhea. The nurse contacts the physician to report that the patient has symptoms of which of the following? a) Nadir b) Graft-versus-host disease c) Acute leukopenia d) Metastasis

Graft-versus-host disease Graft-versus-host disease is a major cause of morbidity and mortality in patients who have had allogeneic transplant. Clinical manifestation of the disease include diffuse rash that progresses to blistering and desquamation, and mucosal inflammation of the eyes and the entire GI tract with subsequent diarrhea, abdominal pain, and hepatomegaly.

What foods should the nurse suggest that the patient consume less of in order to reduce nitrate intake because of the possibility of carcinogenic action?

Ham and bacon

What foods should the nurse suggest that the patient consume less of in order to reduce nitrate intake because of the possibility of carcinogenic action?

Ham and bacon Dietary substances that appear to increase the risk of cancer include fats, alcohol, salt-cured or smoked meats, nitrate and nitrite-containing foods, and red and processed meats. Nitrates are added to cured meats, such as ham and bacon.

The nurse understands that the client should be with human papilloma virus (HPV) vaccine per the health care provider's orders. What is this vaccine for?

Help prevent cervical cancer

The physician recommends that you have your daughter vaccinated with HPV vaccine. What is this vaccine for?

Help prevent cervical cancer

A benign tumor of the blood vessels is a(n)

Hemangioma

A patient with ESRD is taking recombinant erythropoietin for the treatment of anemia. What laboratory study does the nurse understand will have to be assessed at least monthly related to this medication?

Hemoglobin level

A client diagnosed with cancer has their tumor staged and graded based on what?

How they tend to grow and the cell type Tumors are staged and graded based upon how they tend to grow and the cell type before a client is treated for cancer.

When malignant cells are killed (tumor lysis syndrome), intracellular contents are released into the bloodstream. This leads to which of the following? Select all that apply. Hyperphosphatemia Hyperuricemia Hyperkalemia Hypercalcemia

Hyperphosphatemia Hyperuricemia Hyperkalemia

What can the nurse do to meet the challenges in caring for a patient with cancer?

Identify own perception of cancer and set realistic goals.

What can the nurse do to meet the challenges in caring for a patient with cancer?

Identify own perception of cancer and set realistic goals. Nurses need to identify their own perception of cancer and set realistic goals to meet the challenges inherent in caring for patients with cancer. In addition, nurses caring for patients with cancer must be prepared to support patients and families through a wide range of physical, emotional, social, cultural, financial, and spiritual challenges. Cancer is a diverse set of diseases, so the nurse would not make the same goals for all patients with cancer. The causes of many types of cancer are still unknown, so the nurse should not attempt to tell the patient what he or she has done to cause the cancer. The nurse need not ensure that the patient has the financial means to afford the care.

The nurse is caring for a patient who is to begin receiving external radiation for a malignant tumor of the neck. While providing patient education, what potential adverse effects should the nurse discuss with the patient? Impaired nutritional status Cognitive changes Alopecia Diarrhea

Impaired nutritional status

Chemotherapeutic agents have which effect associated with the renal system?

Increased uric acid excretion

A client with a nagging cough makes an appointment to see the physician after reading that this symptom is one of the seven warning signs of cancer. What is another warning sign of cancer?

Indigestion

What does the nurse understand is the rationale for administering allopurinol for a patient receiving chemotherapy?

It lowers serum and uric acid levels

While completing an admission assessment, the client reports a family history of ovarian cancer among a maternal grandmother, aunt, and sister. The nurse knows that these cancers are most likely associated with what etiology?

Inherited gene mutation Tumor suppressor genes assist the body in normal cell production and death. Tobacco use and chemical carcinogens can contribute to the development of cancer, but there is not enough information provided to suggest a common link. Ontogenesis are genes that have mutated and activates out of control cell growth. Inherited gene mutation occurs when the DNA is passed to the next generation.

When caring for a client who is receiving external beam radiation, which is the key point for the nurse to incorporate into the plan of care?

Inspect skin frequently

When caring for a client who is receiving external beam radiation, which is the key point for the nurse to incorporate into the plan of care?

Inspect the skin frequently.

When caring for an older client who is receiving external beam radiation, which is the key point for the nurse to incorporate into the plan of care? Time, distance, and shielding Avoid showering or washing over skin markings. The use of disposable utensils and wash cloths Inspect the skin frequently.

Inspect the skin frequently.

"For a client newly diagnosed with radiation-induced thrombocytopenia, the nurse should include which intervention in the plan of care? Administering aspirin if the temperature exceeds 102° F (38.8° C) Inspecting the skin for petechiae once every shift Providing for frequent rest periods Placing the client in strict isolation

Inspecting the skin for petechiae once every shift

For a client newly diagnosed with radiation-induced thrombocytopenia, the nurse should include which intervention in the care plan?

Inspecting the skin for petechiae once every shift

For a client newly diagnosed with radiation-induced thrombocytopenia, the nurse should include which intervention in the care plan? Providing for frequent rest periods Administering aspirin if the temperature exceeds 102° F (38.8° C) Placing the client in strict isolation Inspecting the skin for petechiae once every shift

Inspecting the skin for petechiae once every shift

The nurse is caring for a client with cancer who is treating her cancer with deep-tissue massage in addition to radiation therapy. The nurse documents the use of which therapy on the client's chart? Alternative therapy Integrative medicine Global medicine Compliant medicine

Integrative medicine

The nurse is caring for a patient with cancer who is treating her cancer with deep tissue massage in addition to radiation therapy. The nurse documents the use of which of the following on the patient's chart?

Integrative medicine

Characteristic of a malignant tumor

It gains access to the blood and lymphatic channels.

Which of the following is a characteristic of a malignant tumor?

It gains access to the blood and lymphatic channels.

Which of the following is a characteristic of a malignant tumor? It gains access to the blood and lymphatic channels. It demonstrates cells that are well differentiated. It is usually slow growing. It grows by expansion.

It gains access to the blood and lymphatic channels.

A client is receiving the cell cycle-nonspecific alkylating agent thiotepa (Thioplex), 60 mg weekly for 4 weeks by bladder instillation as part of chemotherapy regimen to treat bladder cancer. The client asks the nurse how the drug works. How does thiotepa exert its therapeutic effects? a) It interferes with deoxyribonucleic acid (DNA) replication only. b) It destroys the cell membrane, causing lysis. c) It interferes with ribonucleic acid (RNA) transcription only. d) It interferes with DNA replication and RNA transcription.

It interferes with DNA replication and RNA transcription.

What does the nurse understand is the rationale for administering allopurinol for a patient receiving chemotherapy? It treats drug-related anemia. It stimulates the immune system against the tumor cells. It lowers serum and uric acid levels. It prevents alopecia.

It lowers serum and uric acid levels.

What does the nurse understand is the rationale for administering allopurinol for a patient receiving chemotherapy?

It lowers serum and uric acid levels. Adequate hydration, diuresis, alkalinization of the acid crystals, and administration of allopurinol (Zyloprim) may be used to prevent renal toxicity.

A nurse is teaching a client about the rationale for administering allopurinol with chemotherapy. Which example would be the best teaching by the nurse? It stimulates the immune system against the tumor cells it treats drug related anemia it prevents alopecia it lowers serum and uric acid levels

It lowers serum and uric acid levels. - The use of allopurinol with chemotherapy is to prevent renal toxicity. Tumor lysis syndrome occurrence can be reduced with allopurinol's action of reducing the conversion of nucleic acid byproducts to uric acid, in this way preventing urate nephropathy and subsequent oliguric renal failure. Allopurinol does not stimulate the immune system, treat anemia, or prevent alopecia.

The nurse is caring for a client undergoing an incisional biopsy. Which statement does the nurse understand to be true about an incisional biopsy?

It removes a wedge of tissue for diagnosis.

Which of the following is a growth-based classification of tumors? a) Leukemia b) Carcinoma c) Sarcoma d) Malignant

Malignant Explanation: Tumors that are classified on the basis of the cell or tissue of origin are carcinomas, sarcomas, lymphomas, and leukemias. Tumors classified on the basis of growth are described as benign or malignant.

What disadvantages of chemotherapy should the patient be informed about prior to starting the regimen?

It targets normal body cells as well as cancer cells.

What disadvantages of chemotherapy should the patient be informed about prior to starting the regimen?

It targets normal body cells as well as cancer cells. Chemotherapy agents affect both normal and malignant cells; therefore, their effects are often widespread, affecting many body systems.

A patient with uterine cancer is being treated with intracavitary radiation. The patient will emit radiation while the implant is in place. The nurse is aware of the precautions necessary for the provider of care and visitors. Which of the following are appropriate guidelines to follow? Select all that apply. The nurse can provide direct care for up to 60 minutes per 8-hour shift. Lead aprons should be worn to buffer the exposure. Family members should stand about 6 feet from the patient. Visitors may stay for 30 minutes or less.

Lead aprons should be worn to buffer the exposure. Family members should stand about 6 feet from the patient. Visitors may stay for 30 minutes or less.

A decrease in circulating white blood cells (WBC) is referred to as which of the following?

Leukopenia

A decrease in circulating white blood cells (WBCs) is referred to as

Leukopenia

A decrease in circulating white blood cells is

Leukopenia

A decrease in circulating white blood cells (WBC) is referred to as which of the following? A. leukopenia. B. neutropenia. C. granulocytopenia. D. thrombocytopenia.

Leukopenia -A decrease in circulating WBCs is referred to as leukopenia. Granulocytopenia is a decrease in neutrophils. Thrombocytopenia is a decrease in the number of platelets. Neutropenia is an abnormally low absolute neutrophil count (ANC).

A decrease in circulating white blood cells (WBCs) is referred to as

Leukopenia A decrease in circulating WBCs is referred to as leukopenia. Granulocytopenia is a decrease in neutrophils. Thrombocytopenia is a decrease in the number of platelets. Neutropenia is an abnormally low absolute neutrophil count.

A decrease in circulating white blood cells (WBC) is referred to as which of the following? a) Neutropenia b) Thrombocytopenia c) Granulocytopenia d) Leukopenia

Leukopenia Explanation: A decrease in circulating WBCs is referred to as leukopenia. Granulocytopenia is a decrease in neutrophils. Thrombocytopenia is a decrease in the number of platelets. Neutropenia is an abnormally low absolute neutrophil count (ANC)

The nurse is assessing the diet of a female client. To decrease the risk of cancer in general, the nurse instructs the client to Decrease cigarette smoking from one pack/day to 1/2 pack/day. Include at least 6 ounces of meat in meals every day. Limit alcohol ingestion to one drink per day. Ingest two to three servings of fruits and vegetables each day.

Limit alcohol ingestion to one drink per day.

A client with cancer is being evaluated for possible metastasis. What is one of the most common metastasis sites for cancer cells?

Liver

A client with sickle cell anemia has a:

Low hematocrit

A client has received several treatments of bleomycin. It is now important for the nurse to assess

Lung sounds

A client has received several treatments of bleomycin. It is now important for the nurse to assess

Lung sounds Bleomycin has cumulative toxic effects on lung function. Thus, it will be important to assess lung sounds.

You are presenting a class on cancer for a local community group. You inform the attendees that chemical agents in the environment are believed to account for 75% of all cancers. Which organs are most susceptible to cancer caused by these chemical agents?

Lungs, liver, and kidneys

A nurse is talking with a patient who is scheduled to have a hemicolectomy with the creation of a colostomy. The patient admits to being anxious, and has many questions concerning the surgery, the care of a stoma, and necessary lifestyle changes. Which of the following nursing actions is most appropriate? Provide the patient with educational materials that match the patient's learning style. Reassure the patient that the procedure is relatively low risk and that patients are usually successful in adjusting to an ostomy. Encourage the patient to write down these concerns and questions to bring forward to the surgeon. Maintain an open dialogue with the patient and facilitate a referral to the wound-ostomy-continence (WOC) nurse.

Maintain an open dialogue with the patient and facilitate a referral to the wound-ostomy-continence (WOC) nurse.

Which is a growth-based classification of tumors?

Malignacy

Which is a growth-based classification of tumors?

Malignancy

Which is a growth-based classification of tumors?

Malignancy Tumors classified on the basis of growth are described as benign or malignant. Tumors that are classified on the basis of the cell or tissue of origin are carcinomas, sarcomas, lymphomas, and leukemias.

Which of the following is a growth-based classification of tumors?

Malignant

The nurse performs a breast exam on a client and finds a firm, non-moveable lump in the upper outer quadrant of the right breast that the client reports was not there 3 weeks ago. What does this finding suggest?

Malignant Tumor

The nurse performs a breast exam on a client and finds a firm, non-moveable lump in the upper outer quadrant of the right breast that the client reports was not there 3 weeks ago. What does this finding suggest?

Malignant tumor

The root cause of cancer is damage to cellular deoxyribonucleic acid (DNA). Such damage results from multiple factors. Which of the following is a carcinogen? Medically prescribed interventions Chemical agents Viruses Environmental factors Defective genes Dietary substances

Medically prescribed interventions Chemical agents Viruses Environmental factors Defective genes Dietary substances

Cancer has many characteristics. What is one of the most discouraging characteristics of cancer?

Metastasis Metastasis is one of cancer's most discouraging characteristics because even one malignant cell can give rise to a metastatic lesion in a distant part of the body. Not all cancerous tumors are large in size. Carcinogenesis is the process of malignant transformation and it is not a characteristic of cancer. Cancer grows rapidly, not slowly.

In which phase of the cell cycle does cell division occur?

Mitosis

In which phase of the cell cycle does cell division occur?

Mitosis Cell division occurs in mitosis. RNA and protein synthesis occurs in the G1 phase. DNA synthesis occurs during the S phase. DNA synthesis is complete, and the mitotic spindle forms in the G2 phase.

In which phase of the cell cycle does cell division occur? a) G1 phase b) S phase c) Mitosis d) G2 phase

Mitosis Cell division occurs in mitosis. RNA and protein synthesis occurs in the G1 phase. DNA synthesis occurs during the S phase. DNA synthesis is complete, and the mitotic spindle forms in the G2 phase.

The nurse working on a bone marrow unit knows that it is a priority to monitor which of the following in a client who has just undergone a bone marrow transplant?

Monitor the client closely to prevent infection.

The nurse working on a bone marrow unit knows that it is a priority to monitor which of the following in a client who has just undergone a stem cell transplant?

Monitor the client closely to prevent infection.

The nurse working on a bone marrow unit knows that it is a priority to monitor which of the following in a client who has just undergone a bone marrow transplant?

Monitor the client closely to prevent infection. -Until transplanted bone marrow begins to produce blood cells, these clients have no physiologic means to fight infection, which makes them very prone to infection. They are at high risk for dying from sepsis and bleeding before engraftment. Therefore, a nurse must closely monitor clients and take measures to prevent infection. Monitoring client's toilet patterns, physical condition, and heart rate does not prevent the possibility of the client getting an infection.

The nurse working on a bone marrow unit knows that it is a priority to monitor which of the following in a client who has just undergone a bone marrow transplant? Monitor the client's toilet patterns. Monitor the client's physical condition. Monitor the client's heart rate. Monitor the client closely to prevent infection.

Monitor the client closely to prevent infection. Until transplanted bone marrow begins to produce blood cells, these clients have no physiologic means to fight infection, which makes them very prone to infection. They are at high risk for dying from sepsis and bleeding before engraftment. Therefore, a nurse must closely monitor clients and take measures to prevent infection.

The nurse working on a bone marrow unit knows that it is a priority to monitor which of the following in a client who has just undergone a bone marrow transplant?

Monitor the client closely to prevent infection. Until transplanted bone marrow begins to produce blood cells, these clients have no physiologic means to fight infection, which makes them very prone to infection. They are at high risk for dying from sepsis and bleeding before engraftment. Therefore, a nurse must closely monitor clients and take measures to prevent infection. Monitoring client's toilet patterns, physical condition, and heart rate does not prevent the possibility of the client getting an infection.

A nurse is caring for a client admitted with pernicious anemia. Which set of findings should the nurse expect when assessing the client?

Pallor, tachycardia, and a sore tongue

The nurse working on a bone marrow unit knows that it is a priority to monitor which of the following in a client who has just undergone a stem cell transplant?

Monitor the client closely to prevent infection. Until transplanted stem cells begin to produce blood cells, these clients have no physiologic means to fight infection, which makes them very prone to infection. They are at high risk for dying from sepsis and bleeding before engraftment. Therefore, a nurse must closely monitor clients and take measures to prevent infection. Monitoring client's toilet patterns, physical condition, and heart rate does not prevent the possibility of the client getting an infection.

The nurse working on a bone marrow unit knows that it is a priority to monitor which of the following in a client who has just undergone a stem cell transplant? a) Monitor the client closely to prevent infection. b) Monitor the client's heart rate. c) Monitor the client's physical condition. d) Monitor the client's toilet patterns.

Monitor the client closely to prevent infection. Explanation: Until transplanted stem cells begin to produce blood cells, these clients have no physiologic means to fight infection, which makes them very prone to infection. They are at high risk for dying from sepsis and bleeding before engraftment. Therefore, a nurse must closely monitor clients and take measures to prevent infection. Monitoring client's toilet patterns, physical condition, and heart rate does not prevent the possibility of the client getting an infection.

A patient is taking vincristine, a plant alkaloid for the treatment of cancer. What system should the nurse be sure to assess for symptoms of toxicity?

Nervous system

When the client complains of increased fatigue following radiotherapy, the nurse knows this is most likely to be related to which factor?

Radiation can result in myelosuppression.

A patient is taking vincristine, a plant alkaloid for the treatment of cancer. What system should the nurse be sure to assess for symptoms of toxicity?

Nervous system With repeated doses, the taxanes and plant alkaloids, especially vincristine, can cause cumulative peripheral nervous system damage with sensory alterations in the feet and hands.

According to the tumor-node-metastasis (TNM) classification system, T0 means there is which of the following?

No evidence of a primary tumor

According to the TNM classification system, T0 means there is a) no distant metastasis. b) no regional lymph node metastasis. c) distant metastasis. d) no evidence of primary tumor.

No evidence of primary tumor

According to the tumor-node-metastasis (TNM) classification system, T0 means there is

No evidence of primary tumor

According to the tumor-node-metastasis (TNM) classification system, T0 means there is which of the following?

No evidence of primary tumor

According to the TNM classification system, T0 means there is

No evidence of primary tumor.

Your patient is receiving carmustine, a chemotherapy agent. A significant side effect of this medication is thrombocytopenia. What symptom would the nurse assess for in a patient at risk for thrombocytopenia?

Nose bleed

A client with cancer stage T4,N3,M1 is ordered morphine sulfate 4 mg, subcutaneous every 3 to 4 hours. Two hours after the last injection, the client rings the call bell to report a pain scale of 9. Which is the appropriate action by the nurse?

Notify the physician of the breakthrough pain in an attempt to obtain additional orders.

A client with multiple myeloma is complaining of severe pain when the nurse comes in to give a bath and change position. What is the priority intervention by the nurse?

Obtain the pain medication and delay the bath and position change until the medication reaches its peak.

A nurse is teaching a community class about how to decrease the risk of cancer. Which food should the nurse recommend?

Oranges A diet high in vitamin C and citrus may help reduce the risk of certain cancers, such as stomach and esophageal cancers. Hot dogs and smoked and cured foods are high in nitrates, which may be linked to esophageal and gastric cancers. Steak is a high-fat food that may increase the risk of breast, colon, and prostate cancers.

Which type of surgery is used in an attempt to relieve complications of cancer?

Palliative

Which type of surgery is utilized in an attempt to relieve complications of cancer?

Palliative

which type of surgery is used in an attempt to relieve complications of cancer? Palliative Prophylactic reconstructive Salvage

Palliative

Which type of surgery is used in an attempt to relieve complications of cancer?

Palliative Palliative surgery is performed to relieve complications of cancer. Prophylactic surgery involves removing nonvital tissues or organs that are likely to develop cancer. Reconstructive surgery may follow curative or radical surgery and is carried out in an attempt to improve function or to obtain a more desirable cosmetic effect. Salvage surgery is an additional treatment option that uses an extensive surgical approach to treat the local recurrence of a cancer after the use of a less extensive primary approach.

You are admitting a client to your unit. While assessing the client, you find out that the client has had hoarseness of approximately 4 months duration. Why would you report this to the attending physician?

Persistent hoarseness is a warning sign of cancer

The nurse is receiving a client with a radioactive implant for the treatment of cervical cancer. The nurse

Places the client in a private room Safety precautions are used for the client with a radioactive implant. They include assigning the client to a private room, seeing that visitors maintain a 6-foot distance from the radiation source, prohibiting visits by children, and preventing exposure to those who may be or are pregnant, as in the case of the nursing assistant.

An important nursing function is monitoring factors that may indicate that bleeding is occurring. One serum indicator is a (an):

Platelet count of 60,000/mm3.

An important nursing function is monitoring factors that may indicate that bleeding is occurring. One serum indicator is a (an):

Platelet count of 60,000/mm3. Thrombocytopenia, a decrease in the circulating platelet count, is the most common cause of bleeding in patients with cancer and is usually defined as a count less than 100,000/mm3. The risk of bleeding increases as the count drops lower. The risk of spontaneous bleeding occurs with a count of less than 20,000/mm3.

A nurse is interviewing a client about his past medical history. Which preexisting condition may lead the nurse to suspect that a client has colorectal cancer? Hemorrhoids Weight gain Duodenal ulcers Polyps

Polyps

During which step of cellular carcinogenesis do cellular changes exhibit increased malignant behavior?

Progression

During which step of cellular carcinogenesis do cellular changes exhibit increased malignant behavior?

Progression -Progression is the third step of carcinogenesis, in which cells show a propensity to invade adjacent tissues and metastasize. During promotion, repeated exposure to promoting agents causes the expression of abnormal genetic information even after long latency periods. During initiation, initiators such as chemicals, physical factors, and biologic agents, escape normal enzymatic mechanisms and alter the genetic structure of cellular DNA. No stage of cellular carcinogenesis is termed prolongation.

A client is receiving chemotherapy to treat breast cancer. Which assessment finding indicates a chemotherapy induced complication? Urine output of 400 ml in 8 hours Serum potassium level of 2.6 mEq/L Blood pressure of 120/64 to 130/72 mm Hg Sodium level of 142 mEq/L

Serum potassium level of 2.6 mEq/L

During which step of cellular carcinogenesis do cellular changes exhibit increased malignant behavior?

Progression Progression is the third step of carcinogenesis, in which cells show a propensity to invade adjacent tissues and metastasize. During promotion, repeated exposure to promoting agents causes the expression of abnormal genetic information, even after long latency periods. During initiation, initiators such as chemicals, physical factors, and biologic agents escape normal enzymatic mechanisms and alter the genetic structure of cellular DNA. No stage of cellular carcinogenesis is termed prolongation.

The nurse at the clinic explains to the patient that the surgeon will be removing a mole on the patient's back that has the potential to develop into cancer. The nurse informs the patient that this is what type of procedure?

Prophylactic

The nurse at the clinic explains to the patient that the surgeon will be removing a mole on the patient's back that has the potential to develop into cancer. The nurse informs the patient that this is what type of procedure?

Prophylactic Prophylactic surgery involves removing nonvital tissues or organs that are at increased risk of developing cancer. When surgical cure is not possible, the goals of surgical interventions are to relieve symptoms, make the patient as comfortable as possible, and promote quality of life as defined by the patient and family. Palliative surgery and other interventions are performed in an attempt to relieve complications of cancer, such as ulceration, obstruction, hemorrhage, pain, and malignant effusions (Table 15-6). Reconstructive surgery may follow curative or radical surgery in an attempt to improve function or obtain a more desirable cosmetic effect. Diagnostic surgery, or biopsy, is performed to obtain a tissue sample for histologic analysis of cells suspected to be malignant.

You are a clinic nurse. One of your clients has found she is at high risk for breast cancer. She asks you what can be done to reduce her risk. What is a means of reducing the risk for breast cancer?

Prophylactic surgery

You are a clinic nurse. One of your clients has found she is at high risk for breast cancer. She asks you what can be done to reduce her risk. What is a means of reducing the risk for breast cancer?

Prophylactic surgery Prophylactic or preventive surgery may be done if the client is at considerable risk for cancer. Palliative surgery is done when no curative treatment is available. Curative surgery is performed to cure the disease process. Reduction surgery is a distractor.

A patient is admitted for an excisional biopsy of a breast lesion. What intervention should the nurse provide for the care of this patient?

Provide time for the patient to discuss her concerns

A client is receiving chemotherapy to treat breast cancer. Which assessment finding indicates a chemotherapy-induced complication?

Serum potassium level of 2.6 mEq/L

During chemotherapy, an oncology client has a nursing diagnosis of Impaired oral mucous membrane related to decreased nutrition and immunosuppression secondary to the cytotoxic effects of chemotherapy. Which nursing intervention is most likely to decrease the pain of stomatitis?

Providing a solution of viscous lidocaine for use as a mouth rinse

During chemotherapy, an oncology client has a nursing diagnosis of Impaired oral mucous membrane related to decreased nutrition and immunosuppression secondary to the cytotoxic effects of chemotherapy. Which nursing intervention is most likely to decrease the pain of stomatitis?

Providing a solution of viscous lidocaine for use as a mouth rinse To decrease the pain of stomatitis, the nurse should provide a solution of hydrogen viscous lidocaine for the client to use as a mouth rinse. (Commercially prepared mouthwashes contain alcohol and may cause dryness and irritation of the oral mucosa.) The nurse also may administer systemic analgesics as ordered. Stomatitis occurs 7 to 10 days after chemotherapy begins; thus, stopping chemotherapy wouldn't be helpful or practical. Instead, the nurse should stay alert for this potential problem to ensure prompt treatment. Monitoring platelet and leukocyte counts may help prevent bleeding and infection but wouldn't decrease pain in this highly susceptible client. Checking for signs and symptoms of stomatitis also wouldn't decrease the pain.

When the client complains of increased fatigue following radiotherapy, the nurse knows this is most likely to be related to which factor?

Radiation can result in myelosuppression

When the client complains of increased fatigue following radiotherapy, the nurse knows this is most likely to be related to which factor?

Radiation can result in myelosuppression. Fatigue results from anemia associated with myelosuppression and decreased RBC production. The spreading of cancer can cause many symptoms dependent on location and type of cancer but not a significant factor to support fatigue with radiotherapy. The production of healthy cells can increase metabolic rate, but death of cancer cells does not support fatigue in this case. Fighting infection can cause fatigue, but there is no evidence provided to support presence of infection in this client.

A client has been receiving chemotherapy to treat cancer. Which assessment finding suggests that the client has developed stomatitis?

Red, open sores on the oral mucosa

A client has been receiving chemotherapy to treat cancer. Which assessment finding suggests that the client has developed stomatitis?

Red, open sores on the oral mucosa The tissue-destructive effects of cancer chemotherapy typically cause stomatitis, resulting in ulcers on the oral mucosa that appear as red, open sores. White, cottage cheese-like patches on the tongue suggest a candidal infection, another common adverse effect of chemotherapy. Yellow tooth discoloration may result from antibiotic therapy, not cancer chemotherapy. Rust-colored sputum suggests a respiratory disorder, such as pneumonia.

A client has been receiving chemotherapy to treat cancer. Which assessment finding suggests that the client has developed stomatitis? a) White, cottage cheese-like patches on the tongue b) Rust-colored sputum c) Red, open sores on the oral mucosa d) Yellow tooth discoloration

Red, open sores on the oral mucosa Explanation: The tissue-destructive effects of cancer chemotherapy typically cause stomatitis, resulting in ulcers on the oral mucosa that appear as red, open sores. White, cottage cheese-like patches on the tongue suggest a candidal infection, another common adverse effect of chemotherapy. Yellow tooth discoloration may result from antibiotic therapy, not cancer chemotherapy. Rust-colored sputum suggests a respiratory disorder, such as pneumonia

After being seen in the oncology clinic, a client with severe bone marrow suppression is admitted to the hospital. The client's cancer therapy consisted of radiation and chemotherapy. When developing the care plan for this client, which nursing diagnosis takes priority?

Risk for infection Risk for infection takes highest priority in clients with severe bone marrow depression because they have a decrease in the number of white blood cells, the cells that fight infection. Making clients aware that they are at risk for injuries can help prevent such injuries as falls. The nurse should institute the facility's falls prevention protocol and supply assistive devices, such as a walker, cane, or wheelchair, when needed. Imbalanced nutrition: Less than body requirements is also of concern but doesn't take priority over preventing infection. Anxiety is likely present in clients with severe bone marrow depression; however, anxiety doesn't take priority over preventing infection.

A male client has been unable to return to work for 10 days following chemotherapy as the result of ongoing fatigue and inability to perform usual activities. Laboratory test results are WBCs 2000/mm³, RBCs 3.2 x 10¹²/L, and platelets 85,000/mm³. The nurse notes that the client is anxious. Which of the following is the priority nursing diagnosis? Fatigue related to deficient blood cells Anxiety related to change in role function Risk for infection related to inadequate defenses Activity intolerance related to side effects of chemotherapy

Risk for infection related to inadequate defenses

A client suspected of having colorectal cancer requires which diagnostic study to confirm the diagnosis? Sigmoidoscopy Stool Hematest Abdominal computed tomography (CT) scan Carcinoembryonic antigen (CEA)

Sigmoidoscopy

A client is receiving chemotherapy to treat breast cancer. Which assessment finding indicates a chemotherapy-induced complication?

Serum potassium level of 2.6 mEq/L Chemotherapy commonly causes nausea and vomiting, which may lead to fluid and electrolyte imbalances. Signs of fluid loss include a serum potassium level below 3.5 mEq/L, decreased urine output (less than 40 ml/hour), and abnormally low blood pressure. Urine output of 400 ml in 8 hours, serum sodium level of 142 mEq/L, and a blood pressure of 120/64 to 130/72 mm Hg aren't abnormal findings.

A client is receiving chemotherapy to treat breast cancer. Which assessment finding indicates a chemotherapy-induced complication? a) Sodium level of 142 mEq/L b) Blood pressure of 120/64 to 130/72 mm Hg c) Urine output of 400 ml in 8 hours d) Serum potassium level of 2.6 mEq/L

Serum potassium level of 2.6 mEq/L Explanation: Chemotherapy commonly causes nausea and vomiting, which may lead to fluid and electrolyte imbalances. Signs of fluid loss include a serum potassium level below 3.5 mEq/L, decreased urine output (less than 40 ml/hour), and abnormally low blood pressure. Urine output of 400 ml in 8 hours, serum sodium level of 142 mEq/L, and a blood pressure of 120/64 to 130/72 mm Hg aren't abnormal findings

What should the nurse tell a female client who is about to begin chemotherapy and anxious about losing her hair?

She should consider getting a wig or cap before she loses her hair.

The nurse knows that interferon agents are used in association with chemotherapy to produce which effects in the client? Suppression of the bone marrow Enhance action of the chemotherapy Decrease the need for addition adjuvant therapies Shorten the period of neutropenia

Shorten the period of neutropenia Interferon agents are a type of biologic response modifiers (BRMs) used in conjunction with chemotherapy to reduce the risk of infection by shortening the period of neutropenia through bone marrow stimulation

The nurse is preparing to assess a client whose chart documents that the client experienced extravasation when receiving the vesicant vincristine during the previous shift. The documentation also notes that an antidote was administered immediately. The nurse prepares to assess for which conditions? Select all that apply.

Sloughing tissue Tissue necrosis Effectiveness of the antidote

The nurse is caring for a client with a benign breast tumor. The tumor may have which characteristic?

Slow rate of growth

A nurse is caring for a client receiving chemotherapy. Which assessment finding places the client at the greatest risk for an infection?

Stage 3 pressure ulcer on the left heel

A nurse is caring for a client receiving chemotherapy. Which assessment finding places the client at the greatest risk for an infection? Temperature of 98.3° F (36.8° C) Stage 3 pressure ulcer on the left heel Ate 75% of all meals during the day White blood cell (WBC) count of 9,000 cells/mm3

Stage 3 pressure ulcer on the left heel

A nurse is caring for a client receiving chemotherapy. Which assessment finding places the client at the greatest risk for an infection? a) Temperature of 98.3° F (36.8° C) b) White blood cell (WBC) count of 9,000 cells/mm3 c) Ate 75% of all meals during the day d) Stage 3 pressure ulcer on the left heel

Stage 3 pressure ulcer on the left heel Explanation: A stage 3 pressure ulcer is a break in the skin's protective barrier, which could lead to infection in a client who is receiving chemotherapy. The WBC count and temperature are within normal limits. Eating 75% of meals is normal and doesn't increase the client's risk for infection. A client who is malnourished is at a greater risk for infection.

A patient with sickle cell disease comes to the emergency department complaining of severe pain in the back, right hip, and right arm. What intervention is important for the nurse to provide?

Start an intravenous line with dextrose 5% in 0.25 normal saline

The nurse is providing an educational presentation on dietary recommendations for reducing the risk of cancer. Which of the following food selections would demonstrate a good understanding of the information provided in the presentation? Select all that apply. Steamed broccoli and carrots Egg white omelet with spinach and mushrooms Smoked salmon Vegetable and cheddar quiche Turkey breast on whole wheat bread Crispy chicken Caesar Salad

Steamed broccoli and carrots Egg white omelet with spinach and mushrooms Turkey breast on whole wheat bread

A client with a brain tumor is undergoing radiation and chemotherapy for treatment of cancer. The client has recently reported swelling in gums, tongue, and lips. Which is the most likely cause of these symptoms? neutropenia extravasion nadir stomatitis

Stomatitis

A patient with brain tumor is undergoing radiation and chemotherapy for treatment of cancer. Of late, the patient is complaining of swelling in the gums, tongue, and lips. Which of the following is the most likely cause of these symptoms?

Stomatitis

A patient with brain tumor is undergoing radiation and chemotherapy for treatment of cancer. Of late, the patient is complaining of swelling in the gums, tongue, and lips. Which of the following is the most likely cause of these symptoms? a) Neutropenia b) Extravasation c) Stomatitis d) Nadir

Stomatitis Explanation: The symptoms of swelling in gums, tongue, and lips indicate stomatitis. This usually occurs 5 to10 days after the administration of certain chemotherapeutic agents or radiation therapy to the head and neck. Chemotherapy and radiation produce chemical toxins that lead to the breakdown of cells in the mucosa of the epithelium, connective tissue, and blood vessels in the oral cavity.

A nurse is administering a peripheral chemotherapeutic agent. What nursing actions are used for extravasation of a chemotherapeutic agent? Select all that apply. Stop the medication infusion at the first sign of extravasation Aspirate any residual drug from the IV line Administer an antidote, if indicated Apply warm compresses to the irritated site to encourage healing Schedule the client for implanted device

Stop the medication infusion at the first sign of extravasation Aspirate any residual drug from the IV line Administer an antidote, if indicated

Nursing action for extravasation of a chemotherapeutic agent would include which of the following nursing actions? Select all that apply.

Stop the medication infusion at the first sign of extravasation. Aspirate any residual drug from the IV line. Administer an antidote, if indicated. All of the answers except D are appropriate nursing actions. The application of warmth would be contraindicated because it would cause vasodilation, which would increase the absorption of irritant into the local tissues.

Nursing action for extravasation of a chemotherapeutic agent would include which of the following nursing actions? Select all that apply. Stop the medication infusion at the first sign of extravasation. Administer an antidote, if indicated. Aspirate any residual drug from the IV line. Apply warm compresses to the irritated site to encourage healing.

Stop the medication infusion at the first sign of extravasation. Administer an antidote, if indicated. Aspirate any residual drug from the IV line.

A nurse is administering a chemotherapeutic medication to a client, who reports generalized itching and then chest tightness and shortness of breath. The nurse immediately

Stops the chemotherapeutic infusion

A nurse is administering a chemotherapeutic medication to a client, who reports generalized itching and then chest tightness and shortness of breath. The nurse immediately

Stops the chemotherapeutic infusion The client may be experiencing a type I hypersensitivity reaction, which may progress to systemic anaphylaxis. The most immediate action of the nurse is to discontinue the medication followed by initiating emergency protocols.

The nurse is invited to present a teaching program to parents of school-age children. Which topic would be of greatest value for decreasing cancer risks?

Sun safety and use of sunscreen

The nurse is invited to present a teaching program to parents of school-age children. Which topic would be of greatest value for decreasing cancer risks?

Sun safety and use of sunscreen Pool and water safety as well as infection prevention are important teaching topics but will not decrease cancer risk. While performing breast and testicular self-exams may identify cancers in the early stage, this teaching is not usually initiated in school-age children. Severe sunburns that occur in young children can place the child at risk for skin cancers later in life. Because children spend much time out of doors, the use of sunscreen and protective clothing/hats can protect the skin and decrease the risk.

You are an oncology nurse caring for a client who is taking antineoplastic agents. What symptoms must you consider when monitoring this client?

Symptoms of gout

You are an oncology nurse caring for a client who is taking antineoplastic agents. What symptoms must you consider when monitoring this client?

Symptoms of gout The nurse monitors the client being administered an antineoplastic agent for symptoms of gout, which include increased uric acid levels, joint pain, and edema. Administering antineoplastic agents does not cause hypertension, diarrhea, and anemia.

The nursing instructor is talking with the junior class of nursing students about lung cancer. What would be the best rationale the instructor could give for the difficulty of early diagnosis of lung cancer? Symptoms are often minimized by clients. There are no early symptoms of lung cancer. Symptoms often mimic other infectious diseases. Symptoms often do not appear until the disease is well established.

Symptoms often mimic other infectious diseases.

Which oncologic emergency involves the failure in the negative feedback mechanism that normally regulates the release of antidiuretic hormone (ADH)?

Syndrome of inappropriate antidiuretic hormone release (SIADH)

Which oncologic emergency involves the failure in the negative feedback mechanism that normally regulates the release of antidiuretic hormone (ADH)?

Syndrome of inappropriate antidiuretic hormone release (SIADH) SIADH is a result of the failure in the negative feedback mechanism that normally regulates the release of antidiuretic hormone (ADH). Cardiac tamponade is an accumulation of fluid in the pericardial space. DIC is a complex disorder of coagulation and fibrinolysis, which results in thrombosis and bleeding. Tumor lysis syndrome is a rapidly developing oncologic emergency that results from the rapid release of intracellular contents as a result of radiation- or chemotherapy-induced cell destruction of large or rapidly growing cancers such as leukemia.

A client is prescribed 325 mg/day of oral ferrous sulfate. What does the nurse include in client teaching?

Take 1 hour before breakfast

The client has received chemotherapy and 1 week later is at home experiencing nausea and vomiting. The first action of the nurse is to recommend Obtaining acupressure treatments Taking prescribed ondansetron (Zofran) Using imagery techniques Practicing relaxation techniques

Taking prescribed ondansetron (Zofran)

The client has received chemotherapy and 1 week later is at home experiencing nausea and vomiting. The first action of the nurse is to recommend

Taking prescribed ondansetron (Zofran) Serotonin blockers, such as ondansetron, may decrease nausea and vomiting. Once these symptoms are relieved, the client can use other strategies, such as relaxation, imagery, and acupressure. These strategies, when used with serotonin blockers, provide improved anti-emetic protection.

A client is admitted to the hospital for diagnostic testing to rule out colorectal cancer. Which intervention should the nurse include on the plan of care? Test all stools for occult blood. Prepare the client for a gastrostomy tube placement. Administer topical ointment to the rectal area to decrease bleeding. Administer morphine (Duramorph PF) routinely, as ordered.

Test all stools for occult blood.

A nurse is administering daunorubicin (DaunoXome) to a patient with lung cancer. Which situation requires immediate intervention?

The I.V. site is red and swollen

A nurse is administering daunorubicin (DaunoXome) to a patient with lung cancer. Which situation requires immediate intervention?

The I.V. site is red and swollen.

Which of the following would be inconsistent as a common side effect of chemotherapy?

Weight gain Common side effects seen with chemotherapy include myelosuppression, alopecia, nausea and vomiting, anorexia, and fatigue.

A newly diagnosed cancer client is crying and states the following to the nurse: "I promised God that I will be a better person if I can just get better." What is the appropriate assessment of this comment by the nurse?

The cancer is viewed as a punishment from past actions. The comment made by the client is reflective of the bargaining stage of grief in which the client is bargaining with God to gain time. Denial is the first stage of grief in which the client uses to protect self, which is not reflective of the comment made. Anger is the second stage of grief and is not reflective of the statement made. Acceptance of inevitable loss is the final stage of grief, which is not reflective in the comment made.

What should the nurse tell a client who is about to begin chemotherapy and is anxious about hair loss?

The client should consider getting a wig or cap prior to beginning treatment

What should the nurse tell a client who is about to begin chemotherapy and is anxious about hair loss?

The client should consider getting a wig or cap prior to beginning treatment.

What should the nurse tell a client who is about to begin chemotherapy and is anxious about hair loss?

The client should consider getting a wig or cap prior to beginning treatment. If hair loss is anticipated and causing the client anxiety, a wig, cap, or scarf should be purchased before therapy begins. Alopecia develops because chemotherapy affects the rapidly growing cells of the hair follicles. Hair usually begins to grow again within 4 to 6 months after therapy. Clients should know that new growth may have a slightly different color and texture.

A patient is taking prednisone 60 mg per day for the treatment of an acute exacerbation of Crohn's disease. The patient has developed lymphopenia with a lymphocyte count of less than 1,500 mm3. What should the nurse monitor the client for?

The onset of a bacterial infection

A 16-year-old female patient experiences alopecia resulting from chemotherapy, prompting the nursing diagnoses of disturbed body image and situational low self-esteem. What action by the patient would best indicate that she is meeting the goal of improved body image and self-esteem?

The patient requests that her family bring her makeup and wig.

A 36-year-old man is receiving three different chemotherapeutic agents for Hodgkin's disease. The nurse explains to the client that the three drugs are given over an extended period because:

The three drugs have a synergistic effect and act on the cancer cells with different mechanisms.

A 36-year-old man is receiving three different chemotherapeutic agents for Hodgkin's disease. The nurse explains to the client that the three drugs are given over an extended period because:

The three drugs have a synergistic effect and act on the cancer cells with different mechanisms. -Multiple drug regimens are used because the drugs have a synergistic effect. The drugs have different cell-cycle lysis effects, different mechanisms of action, and different toxic adverse effects. They are usually given in combination to enhance therapy. Dosage is not affected by giving the drugs in combination. The second and third drugs do not increase the effectiveness of the first. It is not true that the first two drugs are toxic to cancer cells while the third drug promotes cell growth.

A 36-year-old man is receiving three different chemotherapeutic agents for Hodgkin's disease. The nurse explains to the client that the three drugs are given over an extended period because:

The three drugs have a synergistic effect and act on the cancer cells with different mechanisms. Multiple drug regimens are used because the drugs have a synergistic effect. The drugs have different cell-cycle lysis effects, different mechanisms of action, and different toxic adverse effects. They are usually given in combination to enhance therapy. Dosage is not affected by giving the drugs in combination. The second and third drugs do not increase the effectiveness of the first. It is not true that the first two drugs are toxic to cancer cells while the third drug promotes cell growth.

The Pediatric Nurse Practitioner is doing a physical examination of a client with sickle cell anemia. Why would the nurse practitioner auscultate the lungs and heart?

To detect the abnormal sounds suggestive of acute chest syndrome and heart failure

A client with metastatic pancreatic cancer underwent surgery to remove a malignant tumor in the pancreas. Despite the tumor being removed, the physician informs the client that chemotherapy must be started. Why might the physician opt for chemotherapy?

To prevent metastasis

A client with metastatic pancreatic cancer underwent surgery to remove a malignant tumor in the pancreas. Despite the tumor being removed, the physician informs the client that chemotherapy must be started. Why might the physician opt for chemotherapy?

To prevent metastasis Chemotherapy treats systemic and metastatic cancer. It can also be used to reduce tumor size preoperatively, or to destroy any remaining tumor cells postoperatively. Angiogenesis is the growth of new capillaries from the tissue of origin. This process helps malignant cells obtain needed nutrients and oxygen to promote growth. Fatigue and stomatitis are side effects of radiation and chemotherapy.

The physician is attending to a 72-year-old client with a malignant brain tumor. The physician recommends immediate radiation therapy. What is a reason for the physician's recommendation?

To prevent the formation of new cancer cells

The physician is attending to a 72-year-old patient with a malignant brain tumor. The physician recommends immediate radiation therapy. Which of the following is a reason for the physician's recommendation?

To prevent the formation of new cancer cells

The physician is attending to a 72-year-old client with a malignant brain tumor. The physician recommends immediate radiation therapy. What is a reason for the physician's recommendation?

To prevent the formation of new cancer cells Radiation therapy helps prevent cellular growth. It may be used to cure the cancer or to control malignancy when the tumor cannot be removed or when lymph node involvement is present; also, it can be used prophylactically to prevent spread. Biopsy is used to analyze lymph nodes or to destroy the surrounding tissues around the tumor.

Which of the following is the single largest preventable cause of cancer?

Tobacco

The client is diagnosed with a benign brain tumor. Which of the following features of a benign tumor is of most concern to the nurse?

Tumor pressure against normal tissues

The client is diagnosed with a benign brain tumor. Which of the following features of a benign tumor is of most concern to the nurse?

Tumor pressure against normal tissues Benign tumors grow more slowly than malignant tumors and do not emit tumor-specific antigens or proteins. Benign tumors do not metastasize to distant sites. Benign tumors can compress tissues as it grows, which can result in impaired organ functioning.

The client is diagnosed with a benign brain tumor. Which of the following features of a benign tumor is of most concern to the nurse? Cells colonizing to distant body parts Emission of abnormal proteins Random, rapid growth of the tumor Tumor pressure against normal tissues

Tumor pressure against normal tissues Benign tumors grow more slowly than malignant tumors and do not emit tumor-specific antigens or proteins. Benign tumors do not metastasize to distant sites. Benign tumors can compress tissues as it grows, which can result in impaired organ functioning.

Your client is receiving radiation therapy. The client asks you about oral hygiene. What advice regarding oral hygiene should you offer?

Use a soft toothbrush and avoid an electronic toothbrush

Your client is receiving radiation therapy. The client asks you about oral hygiene. What advice regarding oral hygiene should you offer?

Use a soft toothbrush and avoid an electronic toothbrush.

The nurse should advise a client with iron deficiency anemia to take which action in order to prevent staining of the teeth?

Use a straw or place a spoon at the back of the mouth to take the liquid supplement.

The nurse evaluates teaching as effective when a female client states that she will:

Use sunscreen when outdoors

The nurse evaluates teaching as effective when a female client states that she will

Use sunscreen when outdoors.

The nurse evaluates teaching as effective when a female client states that she will a) Use sunscreen when outdoors. b) Decrease tobacco smoking from one pack/day to half a pack/day. c) Exercise 30 minutes 3 times each week. d) Obtain a cancer history from her parents.

Use sunscreen when outdoors.

The nurse evaluates teaching as effective when a female client states that she will

Use sunscreen when outdoors. -Use of sunscreens play a role in the amount of exposure to ultraviolet light. Even decreasing the use of tobacco still exposes a person to risk of cancer. The American Cancer Society recommends adults to engage in at least 30 minutes of moderate to vigorous physical activity on 5 or more days each week. It is recommended to obtain a cancer history from at least three generations.

The nurse evaluates teaching as effective when a female client states that she will

Use sunscreen when outdoors. Use of sunscreens play a role in the amount of exposure to ultraviolet light. Even decreasing the use of tobacco still exposes a person to risk of cancer. The American Cancer Society recommends adults to engage in at least 30 minutes of moderate to vigorous physical activity on 5 or more days each week. It is recommended to obtain a cancer history from at least three generations.

Which of the following would be consistent with a benign neoplasm? Grows by invasion Usually progressive and slow Gains access to the blood and lymph channels to metastasize Cells are undifferentiated

Usually progressive and slow

Which medication is the antidote to warfarin?

Vitamin K

A nurse is caring for a client receiving chemotherapy. Which nursing action is most appropriate for handling chemotherapeutic agents?

Wear disposable gloves and protective clothing.

Which of the following would be inconsistent as a common side effect of chemotherapy?

Weight gain

A nurse is caring for a client receiving chemotherapy. Which nursing action is most appropriate for handling chemotherapeutic agents?

Wear disposable gloves and protective clothing. A nurse must wear disposable gloves and protective clothing to prevent skin contact with chemotherapeutic agents. The nurse shouldn't recap or break needles. The nurse should use a sterile gauze pad when priming I.V. tubing, connecting and disconnecting tubing, inserting syringes into vials, breaking glass ampules, or other procedures in which chemotherapeutic agents are being handled. Contaminated needles, syringes, I.V. tubes, and other equipment must be disposed of in a leak-proof, puncture-resistant container.

A client received chemotherapy 24 hours ago. Which precautions are necessary when caring for the client?

Wear personal protective equipment when handling blood, body fluids, and feces. Chemotherapy drugs are present in the waste and body fluids of clients for 48 hours after administration. The nurse should wear personal protective equipment when handling blood, body fluids, or feces. Gloves offer minimal protection against exposure. The nurse should wear a face shield, gown, and gloves when exposure to blood or body fluid is likely. Placing incontinence pads in the regular trash container and providing a urinal or bedpan don't protect the nurse caring for the client.

Which of the following would be inconsistent as a common side effect of chemotherapy?

Weight Gain

The nurse is to administer a vesicant chemotherapeutic drug to a client who had a right mastectomy and inserts the intravenous line

With a soft, plastic catheter

The nurse is to administer a vesicant chemotherapeutic drug to a client who had a right mastectomy and inserts the intravenous line With a butterfly needle With a soft, plastic catheter In the client's right forearm In the client's left hand

With a soft, plastic catheter

While doing a health history, a client tells you that her mother, her grandmother, and her sister died of breast cancer. The client wants to know what she can do to keep from getting cancer. What would be your best response?

You can't prevent cancer, but you can have your blood analyzed for tumor markers to see what your risk level is." Specialized tests have been developed for tumor markers, specific proteins, antigens, hormones, genes, or enzymes that cancer cells release. Options B and C are incorrect and giving the client these responses would be giving inaccurate information. Options D is incorrect because it minimizes and negates the client's concern.

A Young female client has received chemotherapeutic medications and asks about any effects the treatments will have related to her sexual health. The most appropriate statement by the nurse is.. You will need to practice birth control measures You will continue having your menses every month You will experience menopause now You will be unable to have children

You will need to practice birth control measures

A client is recovering from a craniotomy with tumor debulking. Which comment by the client indicates to the nurse a correct understanding of what the surgery entailed? a) "I guess the doctor could not remove the entire tumor." b) "I will be glad to finally be done with treatments for this thing." c) "I am so glad the doctor was able to remove the entire tumor." d) "Thank goodness the tumor is contained and curable."

a) "I guess the doctor could not remove the entire tumor." Debulking is a reference made when a tumor cannot be completely removed, often due to its extension far into healthy tissue. Without complete removal, this is not a cure and, the cancer cells will continue to replicate and require adjuvant therapies to prevent further invasion. The physician, not the nurse, will need to clarify the details of the surgery.

Following surgery for adenocarcinoma, the client learns the tumor stage is T3,N1,M0. What treatment mode will the nurse anticipate? a) Adjuvant therapy is likely. b) No further treatment is indicated. c) Palliative care is likely. d) Repeat biopsy is needed before treatment begins.

a) Adjuvant therapy is likely. T3 indicates a large tumor size with N1 indicating regional lymph node involvement. Although M0 suggest no metastasis, following with adjuvant (chemotherapy or radiation therapy) treatment is indicated to prevent the spread of cancer outside the lymph to other organs. The tumor staging of stage IV is indicative of palliative care.

Nursing action for extravasation of a chemotherapeutic agent would include which of the following nursing actions? Select all that apply. a) Administer an antidote, if indicated. b) Aspirate any residual drug from the IV line. c) Stop the medication infusion at the first sign of extravasation. d) Apply warm compresses to the irritated site to encourage healing.

a) Administer an antidote, if indicated. b) Aspirate any residual drug from the IV line. c) Stop the medication infusion at the first sign of extravasation. All of the answers except D are appropriate nursing actions. The application of warmth would be contraindicated because it would cause vasodilation, which would increase the absorption of irritant into the local tissues.

The nurse is conducting a screening for familial predisposition of cancer. Which of the following should the nurse note as a possible indication of hereditary cancer syndrome? a) An aunt and uncle diagnosed with cancer b) Onset of cancer after age 50 in family member c) A second cousin diagnosed with cancer d) A first cousin diagnosed with cancer

a) An aunt and uncle diagnosed with cancer The hallmarks of hereditary cancer syndrome include cancer in two or more first-degree or second-degree relatives, early onset of cancer in family members younger than age 50, same type of cancer in several family members, individual family members with more than one type of cancer, and a rare cancer in one or more family members.

5-Fluorouracil (5FU) is classified as which type of antineoplastic agent? a) Antimetabolite b) Alkylating c) Mitotic spindle poisons d) Nitrosoureas

a) Antimetabolite 5-FU is an antimetabolite. An example of an alkylating agent is nitrogen mustard. A nitrosourea is streptozocin. A mitotic spindle poison is vincristine (VCR).

Which of the following classes of antineoplastic agents is cell cycle-specific? a) Antimetabolites (5-FU) b) Alkylating agents (cisplatin) c) Nitrosoureas (carmustine) d) Antitumor antibiotics (bleomycin)

a) Antimetabolites (5-FU) Antimetabolites are cell cycle-specific (S phase). Antitumor antibiotics are cell cycle-nonspecific. Alkylating agents are cell-cycle nonspecific. Nitrosoureas are cell cycle-nonspecific.

A 28-year-old female client with a recent history of GI disturbance has been scheduled for a barium study. Why did the physician order this particular test for this client? a) Because it will show movement of the GI tract b) Because it will remove a tissue sample from the GI tract c) Because it will show tumor "hot spots" in the GI tract d) Because it provides a three-dimensional cross-sectional view

a) Because it will show movement of the GI tract Fluoroscopy is used to show continuous x-ray images on a monitor, allowing the movement of a body structure to be viewed. A barium study involves fluoroscopy, not the biopsy of tissue samples. Nuclear imaging, not a barium study, uses IV tracers to reveal tumor hot spots. CT scans provide three-dimensional cross-sectional views of tissues to determine tumor density, shape, size, volume, and location as well as highlighting blood vessels that feed the tumor.

The root cause of cancer is damage to cellular deoxyribonucleic acid (DNA). Such damage results from multiple factors. Which of the following is a carcinogen? a) Dietary substances b) Environmental factors c) Defective genes d) Medically prescribed interventions e) Viruses f) Chemical agents

a) Dietary substances b) Environmental factors c) Defective genes d) Medically prescribed interventions e) Viruses f) Chemical agents

A patient with uterine cancer is being treated with internal radiation therapy. What would the nurse's priority responsibility be for this patient? a) Explain to the patient that she will continue to emit radiation while the implant is in place. b) Alert family members that they should restrict their visiting to 5 minutes at any one time. c) Maintain as much distance as possible from the patient while in the room. d) Wear a lead apron when providing direct patient care.

a) Explain to the patient that she will continue to emit radiation while the implant is in place. When the patient has a radioactive implant in place, the nurse and other health care providers need to protect themselves, as well as the patient, from the effects of radiation. Patients receiving internal radiation emit radiation while the implant is in place; therefore, contact with the health care team is guided by principles of time, distance, and shielding to minimize exposure of personnel to radiation. Safety precautions used in caring for a patient receiving brachytherapy include assigning the patient to a private room, posting appropriate notices about radiation safety precautions, having staff members wear dosimeter badges, making sure that pregnant staff members are not assigned to the patient's care, prohibiting visits by children or pregnant visitors, limiting visits from others to 30 minutes daily, and seeing that visitors maintain a 6-foot distance from the radiation source.

Chemotherapeutic agents have which effect associated with the renal system? a) Increased uric acid excretion b) Hypophosphatemia c) Hypokalemia d) Hypercalcemia

a) Increased uric acid excretion Chemotherapeutic agents can damage the kidneys because of their direct effects during excretion and the accumulation of end products after cell lysis. There is increased urinary excretion of uric acid from chemotherapeutic agents. Hyperkalemia, hyperphosphatemia, and hypocalcemia can occur from the use of chemotherapeutic agents.

What does the nurse understand is the rationale for administering allopurinol for a patient receiving chemotherapy? a) It lowers serum and uric acid levels. b) It treats drug-related anemia. c) It prevents alopecia. d) It stimulates the immune system against the tumor cells.

a) It lowers serum and uric acid levels. Adequate hydration, diuresis, alkalinization of the acid crystals, and administration of allopurinol (Zyloprim) may be used to prevent renal toxicity.

A patient with uterine cancer is being treated with intracavitary radiation. The patient will emit radiation while the implant is in place. The nurse is aware of the precautions necessary for the provider of care and visitors. Which of the following are appropriate guidelines to follow? Select all that apply. a) Lead aprons should be worn to buffer the exposure. b) Family members should stand about 6 feet from the patient. c) The nurse can provide direct care for up to 60 minutes per 8-hour shift. d) Visitors may stay for 30 minutes or less.

a) Lead aprons should be worn to buffer the exposure. b) Family members should stand about 6 feet from the patient. d) Visitors may stay for 30 minutes or less. Exposure for the nurse, health care provider or visitors should be limited to 30 minutes/8-hour shift. As time increases, exposure to radiation increases. The goal is to deliver safe, efficient care in the shortest amount of time.

As a nurse, you understand that cancer is the second leading cause of death in the United States, second only to heart disease. One half of all men and one third of all women will develop cancer during their lifetimes. You inform your clients that which of the following types of cancer have the highest prevalence among both men and women? a) Lung and colon b) Colon and skin c) Lung and skin d) Skin and brain

a) Lung and colon Common cancers in men include prostate, lung, and colon. Breast, lung, and colon cancer most commonly affect women.

The oncologist advises a 32-year-old female client who had a lumpectomy 2 years previously to consider a mastectomy when her cancer reappears. This procedure would be considered which type of surgery? a) Salvage b) Prophylactic c) Local excision d) Palliative

a) Salvage Salvage surgeryis done when there is a local recurrence of cancer. Also called preventative surgery, prophylactic surgery may be done when there is a family history or genetic predisposition, ability to detect surgery at an early stage, and client acceptance of the postoperative outcome. Local excision is done when the tumor is removed along with a small margin of healthy tissue. The client's first lumpectomy is an example of local excision. This type of surgery helps to relieve uncomfortable symptoms or prolong life.

A client has cancer of the neck and is receiving external beam radiation therapy to the site. The client is experiencing trauma to the irradiated skin. The nurse does all of the following. (Select all that apply.) a) assesses the client for any sun exposure b) avoids shaving the irradiated skin c) uses cool water to wash the neck area d) applies an over-the-counter ointment to the skin e) inspects for skin damage of the chest area

a) assesses the client for any sun exposure b) avoids shaving the irradiated skin The client receiving external beam radiation therapy may experience trauma to the irradiated skin. To prevent further skin damage, the client is to avoid sun exposure and shaving the irradiated skin area. Other skin areas are not damaged, only the irradiated skin. Lukewarm water is to be used to bathe the area. Water of extreme temperature should be avoided. Many over-the-counter ointments contain metals and may cause additional skin damage.

A nurse caring for a client who has just received chemotherapy infusion is wearing a disposable gown, gloves, and goggles for protection. The nurse knows that accidental exposure to chemotherapy agents can occur through: a) inhalation of aerosols. b) absorption through the goggles. c) absorption through the gown. d) absorption through the gloves.

a) inhalation of aerosols. Aerosol inhalation or absorption through the skin can cause accidental chemotherapy exposure. A nurse must wear a disposable gown and gloves when preparing and administering chemotherapy. She won't absorb chemicals through an intact gown, protective gloves, or goggles.

The clinic nurse is caring for a client who has just been diagnosed with a tumor. The client says to the nurse "The doctor says my tumor is benign. What does that mean?" What is the nurse's best response? a. "Benign tumors don't usually cause death." b. "Benign tumors grow very rapidly." c. "Benign tumors can spread from one place to another." d. "Benign tumors invade surrounding tissue."

a. "Benign tumors don't usually cause death."

An oncology clinic nurse is reinforcing prevention measures for oropharyngeal infections to a client receiving chemotherapy. Which statement by the client indicates that teaching was successful? a. "I clean my teeth gently several times per day." b. "I replace my toothbrush every month." c. "I lubricate my lips with petroleum jelly." d. "I use an alcohol-based mouthwash every morning."

a. "I clean my teeth gently several times per day."

The nurse is caring for a client is scheduled for chemotherapy followed by autologous stem cell transplant. Which of the following statements by the client indicates a need for further teaching? a. "I hope they find a bone marrow donor who matches." b. "The doctor will remove cells from my bone marrow before beginning chemotherapy." c. "I will receive chemotherapy until most of the cancer is gone, and then I will get my own stem cells back." d. "I will need to be in protective isolation for up to 3 months after treatment."

a. "I hope they find a bone marrow donor who matches."

The nurse is caring for a client who has had a nuclear scan to aid in the diagnosis of possible cancer. The scan showed a "hot spot". What does this mean? a. An area of increased concentrations of the tracer used in the scan. b. Distinguishes areas of tissue that are normal. c. An area of decreased concentrations of the tracer used in the scan. d. Distinguishes abnormal areas of tumor

a. An area of increased concentrations of the tracer used in the scan.

The nurse is conducting a screening for familial predisposition to cancer. Which element should the nurse note as a possible indication of hereditary cancer syndrome? a. An aunt and uncle diagnosed with cancer b. A second cousin diagnosed with cancer c. A first cousin diagnosed with cancer d. Onset of cancer after age 50 in family member

a. An aunt and uncle diagnosed with cancer

5-Fluorouracil (5FU) is classified as which type of antineoplastic agent? a. Antimetabolite b. Alkylating c. Nitrosoureas d. Mitotic spindle poisons

a. Antimetabolite

A client with cancer is receiving chemotherapy and reports to the nurse that his mouth is painful and he has difficulty ingesting food. The nurse does which of the following: (Select all that apply) a. Asks the client to open his mouth to facilitate inspection of the oral mucosa b. Rinses the client's mouth with alcohol-based mouthwash every 2 hours c. Instructs the client to brush the teeth with a soft toothbrush d. Consults with the healthcare provider about use of nystatin (Mycostatin) e. Teaches the client to floss his teeth once every 24 hours

a. Asks the client to open his mouth to facilitate inspection of the oral mucosa c. Instructs the client to brush the teeth with a soft toothbrush d. Consults with the healthcare provider about use of nystatin (Mycostatin)

A client has just received stem cell transplantation as treatment for leukemia. What are the post procedural nursing interventions for clients receiving any form of stem cell transplantation? a. Closely monitor the client for at least 3 months. b. Closely monitor the client for at least 5 months. c. Closely monitor the client for at least 3 days. d. Closely monitor the client for at least 4 weeks.

a. Closely monitor the client for at least 3 months.

A patient with uterine cancer is being treated with internal radiation therapy. What would the nurse's priority responsibility be for this patient? a. Explain to the patient that she will continue to emit radiation while the implant is in place. b. Maintain as much distance as possible from the patient while in the room. c. Alert family members that they should restrict their visiting to 5 minutes at any one time. d. Wear a lead apron when providing direct patient care.

a. Explain to the patient that she will continue to emit radiation while the implant is in place.

Which grade of tumor is also known as a well-differentiated tumor? a. Grade I b. Grade II c. Grade III d. Grade IV

a. Grade I

A male client has been unable to return to work for 10 days following chemotherapy as the result of ongoing fatigue and inability to perform usual activities. Laboratory test results are WBCs 2000/mm³, RBCs 3.2 x 10¹²/L, and platelets 85,000/mm³. The nurse notes that the client is anxious. Which of the following is the priority nursing diagnosis? a. Risk for infection related to inadequate defenses b. Fatigue related to deficient blood cells c. Activity intolerance related to side effects of chemotherapy d. Anxiety related to change in role function

a. Risk for infection related to inadequate defenses

a patient, age 67 years, is admitted for diagnostic studies to rule out cancer. the patient is caucasian, has been employed as a landscaper for 40 years, and has a 36 year history of smoking a pack of cigarettes a day. what significant risk factors does the nurse recognize this patient has? (select all that apply) a. age b. cigarette smoking c. occupation d. race e. martial status

a. age b. cigarette smoking c. occupation

The drug interleukin-2 is an example of which type of biologic response modifier? a. cytokine b. monoclonal antibodies c. retinoids d. antimetabolites

a. cytokine

The root cause of cancer is damage to cellular deoxyribonucleic acid (DNA) which can be caused by many factors, or carcinogens. What factors can be carcinogenic? Select all that apply. a. dietary substances b. environmental factors c. gender d. age e. viruses

a. dietary substances b. environmental factors e. viruses

what intervention should the nurse provide to reduce the incidence of renal damage when a patient is taking a chemotherapy regimen? a. encourage fluid intake to dilute the urine b. take measures to acidify the urine and prevent uric acid crystallization c. withhold medication when the blood urea nitrogen level exceeds 20mg/dL d. limit fluids to 1,000 mL daily to prevent accumulation of the drug's end products after cell lysis

a. encourage fluid intake to dilute the urine

a patient will be having an endoscopic procedure with a diagnostic biopsy. what type of biopsy does the nurse explain will remove and entire piece of suspicious tissue? a. excisional biopsy b. incisional biopsy c. needle biopsy d. punch biopsy

a. excisional biopsy

a patient with uterine cancer is being treated with internal radiation therapy. what would the nurse's priority responsibility be for this patient? a. explain to the patient that she will continue to emit radiation for approximately 1 week after the implant is removed b. maintain as much distance as possible from the patient while in the room c. alert family members that she should restrict their visiting to 5 minutes at one time d. wear a lead apron when providing direct patient care

a. explain to the patient that she will continue to emit radiation for approximately 1 week after the implant is removed

a patient is to receive Bacille Calmette-Guerin (BCG), a nonspecific biologic response modifier. why would the patient receive this form of treatment? a. for cancer of the bladder b. for cancer of the breast c. for cancer of the lungs d. for skin cancer

a. for cancer of the bladder

what can the nurse do to meet the challenges in caring for a patient with cancer? a. identify own perception of cancer and set realistic goals b. set the same goals for all patients with cancer c. tell the patient about the things the patient has done to cause cancer d. ensure that the patient has the financial means to afford their care

a. identify own perception of cancer and set realistic goals

A nurse caring for a client who has just received chemotherapy infusion is wearing a disposable gown, gloves, and goggles for protection. The nurse knows that accidental exposure to chemotherapy agents can occur through: a. inhalation of aerosols. b. absorption through the gown. c. absorption through the gloves. d. absorption through the goggles.

a. inhalation of aerosols.

what is the best way for the nurse to assess the nutritional status of a patient with cancer? a. weigh the patient daily b. monitor daily caloric intake c. observe for proper wound healing d. assess BUN and Cr levels

a. weigh the patient daily

To combat the most common adverse effects of chemotherapy, a nurse should administer an:

antiemetic

To combat the most common adverse effects of chemotherapy, a nurse should administer an:

antiemetic. Antiemetics, antihistamines, and certain steroids treat nausea and vomiting. Antimetabolites and tumor antibiotics are classes of chemotherapeutic medications. Anticoagulants slow blood clotting time, thereby helping to prevent thrombi and emboli.

An oncology clinic nurse is reinforcing prevention measures for oropharyngeal infections to a client receiving chemotherapy. Which statement by the client indicates that teaching was successful? a) "I replace my toothbrush every month." b) "I clean my teeth gently several times per day." c) "I use an alcohol-based mouthwash every morning." d) "I lubricate my lips with petroleum jelly."

b) "I clean my teeth gently several times per day." The client demonstrates understanding when he states that he'll clean his teeth gently several times per day. Frequent gentle cleaning of the mouth or rinsing reduces bacteria build-up in the oral cavity, thus reducing the risk for oropharyngeal infection. Changing the toothbrush each month reduces the bacteria in the mouth for the first few uses only. Petroleum jelly moistens the lips, but doesn't prevent breakdown of the mucous membranes or reduce the risk for oropharyngeal infection. Alcohol-based products cause drying of the mucous membrane, increasing the likelihood of oropharyngeal infection.

a patient is taking vincristine, a plant alkaloid for the treatment of cancer. what system should the nurse be sure to assess for symptoms of toxicity? a. gastrointestinal system b. nervous system c. pulmonary system d. urinary system

b. nervous system

A nurse is caring for a client who is receiving chemotherapy and has a platelet count of 30,000/mm3. Which statement by the client indicates a need for additional teaching? a) "I take a stool softener every morning." b) "I floss my teeth every morning." c) "I use an electric razor to shave." d) "I removed all the throw rugs from the house."

b) "I floss my teeth every morning." A client with a platelet count of 30,000/mm3 is at risk for bleeding and shouldn't floss his teeth. Flossing may increase the risk of bleeding in a client with a platelet count less than 40,000/mm3. Using an electric razor is appropriate because doing so helps minimize the risk of cutting when shaving. Taking a stool softener helps decrease potential trauma to the GI tract that may cause bleeding. Removing throw rugs from the house helps prevent falls, which could lead to uncontrolled bleeding.

The nurse is caring for a client who is scheduled for chemotherapy. Which is the best statement the nurse can make about the client experiencing chemotherapy-induced alopecia? a) "Wigs can be used after the chemotherapy is completed." b) "The hair loss is temporary." c) "Clients with alopecia will have delay in grey hair." d) "New hair growth will return without any change to color or texture."

b) "The hair loss is temporary." Alopecia associated with chemotherapy is usually temporary and will return after the therapy is completed. New hair growth may return unchanged, but there is no guarantee and color, texture, and quality of hair may be changed. There is no correlation between chemotherapy and delay in greying of hair. Use of wigs, scarves, and head coverings can be used by clients at any time during treatment plan.

The nurse is assessing a 75-year-old woman who had a total hysterectomy when she was 30 years old and normal Pap test results for the past 10 years. The client asks about continuing the Pap test. The nurse states: a) "You will need to continue for the rest of your life." b) "You may choose to discontinue this test." c) "You could have stopped immediately after your hysterectomy." d) You need to continue obtaining a Pap test for only the next 5 years."

b) "You may choose to discontinue this test." The American Cancer Society recommendations for women 70 years or older, who have had normal Pap tests for 10 years, and who have had a total hysterectomy may choose to stop cervical cancer screening as in a Pap test.

A client with cancer is receiving chemotherapy and reports to the nurse that his mouth is painful and he has difficulty ingesting food. The nurse does which of the following: a) Teaches the client to floss his teeth once every 24 hours b) Asks the client to open his mouth to facilitate inspection of the oral mucosa c) Rinses the client's mouth with alcohol-based mouthwash every 2 hours d) Consults with the healthcare provider about use of nystatin (Mycostatin) e) Instructs the client to brush the teeth with a soft toothbrush

b) Asks the client to open his mouth to facilitate inspection of the oral mucosa d) Consults with the healthcare provider about use of nystatin (Mycostatin) e) Instructs the client to brush the teeth with a soft toothbrush The description of the client's report is stomatitis following chemotherapy treatment. The nurse should assess the oral mucosa based on the client's report of pain and difficulty eating. The client is to use a soft toothbrush to minimize trauma to the mouth. Nystatin is a topical medication that may provide healing for the client's mouth. The client avoids alcohol-based mouthwashes as these are irritants. Flossing the teeth may cause additional trauma to the mouth.

The nurse is preparing to assess a patient whose chart documents that the patient experienced extravasation when receiving the vesicant Vincristine on the previous shift. The documentation also notes that an antidote was administered immediately. The nurse prepares to assess for which of the following? Select all that apply. a) Active bleeding b) Effectiveness of antidote c) Sloughing tissue d) Tissue necrosis

b) Effectiveness of antidote c) Sloughing tissue d) Tissue necrosis Extravasation of vesicant chemotherapeutic agents can lead to erythema, sloughing, and necrosis of surrounding tissue, muscle and tendons. To reduce the likelihood/severity of symptoms due to extravasation of a vesicant, antidotes matched to the vesicant are administered. Nurses caring for a patient who experienced extravasation of a vesicant should assess for sloughing tissue, tissue necrosis, erythema, and effectiveness of the antidote.

A patient will be having an endoscopic procedure with a diagnostic biopsy. What type of biopsy does the nurse explain will remove an entire piece of suspicious tissue? a) Punch biopsy b) Excisional biopsy c) Incisional biopsy d) Needle biopsy

b) Excisional biopsy Excisional biopsy is most frequently used for small, easily accessible tumors of the skin, breast, and upper or lower gastrointestinal and upper respiratory tracts. In many cases, the surgeon can remove the entire tumor as well as the surrounding marginal tissues. The removal of normal tissue beyond the tumor area decreases the possibility that residual microscopic malignant cells may lead to a recurrence of the tumor. Incisional biopsy is performed if the tumor mass is too large to be removed. In this case, a wedge of tissue from the tumor is removed for analysis. Needle biopsy is performed to sample suspicious masses that are easily and safely accessible, such as some masses in the breasts, thyroid, lung, liver, and kidney. A core needle biopsy uses a specially designed needle to obtain a small core of tissue that permits histologic analysis.

You are the nurse caring for a client with cancer. The client complains of pain and nausea. When assessed, you note that the client appears fearful. What other factor must you consider when a client with cancer indicates signs of pain, nausea, and fear? a) Infection b) Fatigue c) Ulceration d) High cholesterol levels

b) Fatigue Clients with cancer experience fatigue, which is a side effect of cancer treatments that rest fails to relieve. The nurse must assess the client for other stressors that contribute to fatigue such as pain, nausea, fear, and lack of adequate support. The nurse works with other healthcare team members to treat the client's fatigue. The above indications do not contribute to infections, ulcerations, or high cholesterol levels.

A client with cancer is being evaluated for possible metastasis. What is one of the most common metastasis sites for cancer cells? a) Reproductive tract b) Liver c) White blood cells (WBCs) d) Colon

b) Liver The liver is one of the five most common cancer metastasis sites. The others are the lymph nodes, lung, bone, and brain. The colon, reproductive tract, and WBCs are occasional metastasis sites.

Which of the following would be consistent with a benign neoplasm? a. grows by invasion b. usually progressive and slow c. gains access to the blood and lymph channels to metastasize d. cells are undifferentiated

b. usually progressive and slow

a patient will be receiving radiation for 6 weeks for the treatment of breast cancer and asks the nurse why it takes so long. what is the best response by the nurse? a. "it allows time for you to cope with the treatment" b. "it will allow time for the repair of healthy tissue" c. "it will decrease the incidence of leukopenia and thrombocytopenia" d. "it is not really understood why you have to go for 6 weeks of treatment?

b. "it will allow time for the repair of healthy tissue"

a patient is scheduled for cyrosurgery for cervical cancer and tells the nurse "i am not exactly sure what the doctor is going to do." what is the best response by the nurse? a. "the physician is going to use medication to inject the area" b. "the physician is going to use liquid nitrogen to freeze the area" c. "the physician is going to use a laser to remove the area" d. "the physician is going to use radiofrequency to ablate the area"

b. "the physician is going to use liquid nitrogen to freeze the area"

Following surgery for adenocarcinoma, the client learns the tumor stage is T3,N1,M0. What treatment mode will the nurse anticipate? a. No further treatment is indicated. b. Adjuvant therapy is likely. c. Palliative care is likely. d. Repeat biopsy is needed before treatment begins.

b. Adjuvant therapy is likely.

A patient will be having an endoscopic procedure with a diagnostic biopsy. What type of biopsy does the nurse explain will remove an entire piece of suspicious tissue? a. Incisional biopsy b. Excisional biopsy c. Needle biopsy d. Punch biopsy

b. Excisional biopsy

A client is scheduled for a nerve-sparing prostatectomy. The emotional spouse confides in the nurse that the client will not talk about the cancer and/or upcoming surgery. Which nursing diagnosis will the nurse choose as primary diagnosis for this client? a. Sexual Dysfunction b. Fear c. Knowledge Deficit d. Ineffective Coping

b. Fear

A nurse has agreed to draft a medication teaching plan for a patient who is taking the hormonal agent, Aromasin, an aromatase inhibitor for postmenopausal women with breast cancer. The nurse knows that a major teaching point is to tell the patient to: a. Report the unusual sign of nausea. b. Increase her intake of calcium-rich foods. c. Be alarmed if she notices fluid retention d. Report the unexpected sign of increased appetite and weight gain.

b. Increase her intake of calcium-rich foods.

Chemotherapeutic agents have which effect associated with the renal system? a. Hypokalemia b. Increased uric acid excretion c. Hypophosphatemia d. Hypercalcemia

b. Increased uric acid excretion

When caring for an older client who is receiving external beam radiation, which is the key point for the nurse to incorporate into the plan of care? a. Time, distance, and shielding b. Inspect the skin frequently. c. The use of disposable utensils and wash cloths d. Avoid showering or washing over skin markings.

b. Inspect the skin frequently.

Which of the following is a characteristic of a malignant tumor? a. It demonstrates cells that are well differentiated. b. It gains access to the blood and lymphatic channels. c. t is usually slow growing. d. It grows by expansion.

b. It gains access to the blood and lymphatic channels.

the nurse assesses that extravasation of a chemotherapy agent has occurred. what should the initial action of the nurse be? a. apply a warm compress to the area b. discontinue the infusion c. inject an antidote, if required d. place ice over the site of infiltration

b. discontinue the infusion

A client with cancer is being evaluated for possible metastasis. What is one of the most common metastasis sites for cancer cells? a. colon b. liver c. reproductive tract d. white blood cells (WBCs)

b. liver

The nurse is completing an admission assessment for a client receiving interstitial implants for prostate cancer. The nurse documents this as:

brachytherapy.

The client has finished the first round of chemotherapy. Which statement made by the client indicates a need for further teaching by the nurse? a) "I will use birth control measures until after all treatment is completed." b) "Hair loss may not occur until after the second round of therapy." c) "I can continue taking my vitamins and herbs because they make me feel better." d) "I will eat clear liquids for the next 24 hours."

c) "I can continue taking my vitamins and herbs because they make me feel better." Herbal products are not regulated by the U.S. Food and Drug Administration (FDA);although some can decrease the risk of cancer, others can have serious side effects and liver toxicity. Use of vitamins and herbals should be reviewed with the oncologist. Use of clear liquids is recommended for the client experiencing nausea and vomiting. Because hair follicles are sensitive to chemotherapy drugs, it is likely for alopecia to occur especially with consecutive treatments. Chemotherapy includes cytotoxic drugs that are harmful to rapid dividing cells such as cell development in the fetus. To prevent damage to the fetus, birth control is recommended during treatment.

Which statement by a client undergoing external radiation therapy indicates the need for further teaching? a) "I'll not use my heating pad during my treatment." b) "I'll wear protective clothing when outside." c) "I'm worried I'll expose my family members to radiation." d) "I'll wash my skin with mild soap and water only."

c) "I'm worried I'll expose my family members to radiation." The client undergoing external radiation therapy requires further teaching when he voices a concern that he might expose his family to radiation. Internal radiation, not external radiation, poses a risk to the client's family. The client requires no further teaching if he states that he should wash his skin with mild soap and water, wear protective clothing when outside, and avoid using a heating pad.

A cancer client makes the following statement to the nurse: "I guess I will tell my doctor to forego the chemotherapy. I do not want to be throwing up all the time. I would rather die."Which of the following facts supports the use of chemotherapy for this client? a) Most clients believe the discomfort is well worth the cure for cancer. b) Nausea and vomiting are only a factor for the first 24 hours after treatment. c) Chemotherapy treatment can be adjusted to optimize effects while limiting adverse effects. d) Clinical trials are opening up new cancer treatments all the time.

c) Chemotherapy treatment can be adjusted to optimize effects while limiting adverse effects. Chemotherapy is not one drug for all clients. The therapy can be specifically designed to optimize effects while limiting adverse effects with supplemental anti emetics to control the nausea and vomiting. It is true that nausea and vomiting are most prevalent in the first 24 hours after each chemotherapy treatment but does not eliminate the fears expressed by this client. No one can state the worth of any treatment, and a cure is never promised. Clinical trials open up new options for treatment, but the process is lengthy and is not a certainty for a client in need of immediate treatment.

The nurse is caring for a client newly diagnosed with cancer. Which of the following therapies is used to treat something other than cancer? a) Chemotherapy b) Radiation therapy c) Electroconvulsive therapy d) Surgery

c) Electroconvulsive therapy Cancer is frequently treated with a combination of therapies using standardized protocols. Three basic methods used to treat cancer are surgery, radiation therapy, and chemotherapy. Electroconvulsive therapy (ECT) is a method of treatment for mental distress or illness.

A side-effect of chemotherapy is renal damage. To prevent this, the nurse should: a) Limit fluids to 1,000 mL/day to minimize stress on the renal tubules. b) Withhold medication when the blood urea nitrogen level exceeds 20 mg/dL. c) Encourage fluid intake, if possible, to dilute the urine. d) Modify the diet to acidify the urine, thus preventing uric acid crystallization.

c) Encourage fluid intake, if possible, to dilute the urine. To prevent renal damage, it is helpful to dilute the urine by increasing fluids as tolerated.

A client is scheduled for a nerve-sparing prostatectomy. The emotional spouse confides in the nurse that the client will not talk about the cancer and/or upcoming surgery. Which nursing diagnosis will the nurse choose as primary diagnosis for this client? a) Ineffective Coping b) Sexual Dysfunction c) Fear d) Knowledge Deficit

c) Fear Fear of the unknown is probably the major concern for this client. Fear of the diagnosis of cancer, fear of the effects of the surgery, and fear of loss of control and functioning. Sexual dysfunction may be one of the fears but not primary at this stage. Knowledge Deficit is unclear at this time. Ineffective Coping can be illustrated by the client's refusal to talk about the problem, but no excess or abnormal behavior has been identified at this time.

A patient is to receive Bacille Calmette-Guerin (BCG), a nonspecific biologic response modifier. Why would the patient receive this form of treatment? a) For cancer of the lungs b) For cancer of the breast c) For cancer of the bladder d) For skin cancer

c) For cancer of the bladder Early investigations of the stimulation of the immune system involved nonspecific agents such as bacille Calmette-Guérin (BCG) and Corynebacterium parvum. When injected into the patient, these agents serve as antigens that stimulate an immune response. The hope is that the stimulated immune system will then eradicate malignant cells. Extensive animal and human investigations with BCG have shown promising results, especially in treating localized malignant melanoma. In addition, BCG bladder instillation is a standard form of treatment for localized bladder cancer.

Which grade of tumor is also known as a well-differentiated tumor? a) Grade IV b) Grade III c) Grade I d) Grade II

c) Grade I Grade I tumors, also known as well-differentiated tumors, closely resemble the tissue of origin in structure and function. In grade II, the tumor is moderately differentiated. Tumors in grade III are poorly differentiated (little resemblance to tissue of origin). Grade IV tumors is undifferentiated (unable to tell tissue of origin).

During which step of cellular carcinogenesis do cellular changes exhibit increased malignant behavior? a) Initiation b) Promotion c) Progression d) Prolongation

c) Progression Progression is the third step of carcinogenesis, in which cells show a propensity to invade adjacent tissues and metastasize. During promotion, repeated exposure to promoting agents causes the expression of abnormal genetic information even after long latency periods. During initiation, initiators such as chemicals, physical factors, and biologic agents, escape normal enzymatic mechanisms and alter the genetic structure of cellular DNA. No stage of cellular carcinogenesis is termed prolongation.

What should the nurse tell a female client who is about to begin chemotherapy and anxious about losing her hair? a) Her hair will grow back within 2 months post therapy. b) Alopecia related to chemotherapy is relatively uncommon. c) She should consider getting a wig or cap before she loses her hair. d) Her hair will grow back the same as it was before treatment.

c) She should consider getting a wig or cap before she loses her hair. If hair loss is anticipated, purchase a wig, cap, or scarf before therapy begins. Alopecia develops because chemotherapy affects rapidly growing cells of the hair follicles. Hair usually begins to grow again within 4 to 6 months after therapy. Clients should know that new growth may have a slightly different color and textures.

The client has finished the first round of chemotherapy. Which statement made by the client indicates a need for further teaching by the nurse? a. "I will eat clear liquids for the next 24 hours." b. "Hair loss may not occur until after the second round of therapy." c. "I can continue taking my vitamins and herbs because they make me feel better." d. "I will use birth control measures until after all treatment is completed."

c. "I can continue taking my vitamins and herbs because they make me feel better."

After a bone marrow transplant (BMT), the client should be monitored for at least a. 30 days b. 14 days c. 100 days d. 60 days

c. 100 days

The oncology nurse is giving chemotherapy to a client in a short stay area. The client confides that they are very depressed. The nurse recognizes depression as which of the following? a. A psychiatric diagnosis everyone has at one time or another. b. A side effect of the neoplastic drugs. c. A normal reaction to the diagnosis of cancer. d. An aberrant psychologic reaction to the chemotherapy.

c. A normal reaction to the diagnosis of cancer.

The nurse is caring for a client with cancer who is treating her cancer with deep-tissue massage in addition to radiation therapy. The nurse documents the use of which therapy on the client's chart? a. Alternative therapy b. Global medicine c. Integrative medicine d. Compliant medicine

c. Integrative medicine

A client is receiving the cell cycle-nonspecific alkylating agent thiotepa (Thioplex), 60 mg weekly for 4 weeks by bladder instillation as part of chemotherapy regimen to treat bladder cancer. The client asks the nurse how the drug works. How does thiotepa exert its therapeutic effects? a. It interferes with deoxyribonucleic acid (DNA) replication only. b. It interferes with ribonucleic acid (RNA) transcription only. c. It interferes with DNA replication and RNA transcription. d. It destroys the cell membrane, causing lysis.

c. It interferes with DNA replication and RNA transcription.

A patient is admitted for an excisional biopsy of a breast lesion. What intervention should the nurse provide for the care of this patient? a. Clarify information provided by the physician. b. Provide aseptic care to the incision postoperatively. c. Provide time for the patient to discuss her concerns. d. Counsel the patient about the possibility of losing her breast.

c. Provide time for the patient to discuss her concerns.

What are considered carcinogens? a. parasites b. medical procedures c. dietary substances d. inefective genes

c. dietary substances

The nurse is caring for a client newly diagnosed with cancer. Which of the following therapies is used to treat something other than cancer? a. surgery b. radiation therapy c. electroconvulsive therapy d. chemotherapy

c. electroconvulsive therapy

what foods should the nurse suggest that the patient consume less of in order to reduce nitrate intake because of the possibility of carcinogenic action? a. eggs and milk b. fish and poultry c. ham and bacon d. green, leafy vegetables

c. ham and bacon

the nurse at the clinic explains to the patient that the surgeon will be removing a mole on the patient's back that has the potential to develop into cancer. the nurse informs the patient that this is what type of procedure? a. diagnostic b. palliative c. prophylactic d. reconstructive

c. prophylactic

a patient is admitted for an excisional biopsy of a breast lesion. what intervention should the nurse provide for care of this patient? a. clarify information provided by the physician b. provide aseptic care to the incision postoperatively c. provide time for the patient to discuss their concerns d. counsel the patient about the possibility of losing her breast

c. provide time for the patient to discuss their concerns

The nurse is caring for a client with a benign breast tumor. The tumor may have which characteristic? a. ability to invade other tissues b. causes generalized symptoms c. slow rate of growth d. undifferentiated cells

c. slow rate of growth

A nurse is performing a home visit for a client who received chemotherapy within the past 24 hours. The nurse observes a small child playing in the bathroom, where the toilet lid has been left up. Based on these observations, the nurse modifies the client's teaching plan to include:

chemotherapy exposure and risk factors

A nurse is performing a home visit for a client who received chemotherapy within the past 24 hours. The nurse observes a small child playing in the bathroom, where the toilet lid has been left up. Based on these observations, the nurse modifies the client's teaching plan to include:

chemotherapy exposure and risk factors.

A nurse is caring for a recently married, 29-year-old female client, who was diagnosed with acute lymphocytic leukemia. The client is preparing for an allogeneic bone marrow transplant. Which statement by the client demonstrates she understands the informed consent she gave about the diagnosis and treatment? a) "I always had a good appetite. Even with chemo I shouldn't have to make any changes to my diet." b) "I'll have to remain in the hospital for about 3 months after my transplant." c) "I should be able to finally start a family after I'm finished with the chemo." d) "I'll only need chemotherapy treatment before receiving my bone marrow transplant."

d) "I'll only need chemotherapy treatment before receiving my bone marrow transplant." This client demonstrates understanding about treatment when she states that she'll need chemotherapy before receiving a bone marrow transplant. Most clients receive chemotherapy before undergoing bone marrow transplantation. Most women older than age 26 can't bear children after undergoing treatment because they experience the early onset of menopause. Clients who undergo chemotherapy or radiation must avoid all fresh fruits and vegetables, and all foods should be cooked to avoid bacterial contamination. Clients who undergo bone marrow transplantation typically remain hospitalized for 20 to 25 days.

Following a BMT the patient should be monitored for at least a) 3 days. b) 4 weeks. c) 5 months. d) 3 months.

d) 3 months. After a BMT, the nurse closely monitors the patient for at least 3 months because complications related to the transplant are still possible, and infections are very common.

A nurse is developing a care plan for bone marrow suppression, the major dose-limiting adverse reaction to floxuridine (FUDR). How long after drug administration does bone marrow suppression become noticeable? a) 21 to 28 days b) 24 hours c) 2 to 4 days d) 7 to 14 days

d) 7 to 14 days Bone marrow suppression becomes noticeable 7 to 14 days after floxuridine administration. Bone marrow recovery occurs in 21 to 28 days.

After cancer chemotherapy, a client experiences nausea and vomiting. The nurse should assign highest priority to which intervention? a) Withholding fluids for the first 4 to 6 hours after chemotherapy administration b) Serving small portions of bland food c) Encouraging rhythmic breathing exercises d) Administering metoclopramide and dexamethasone as ordered

d) Administering metoclopramide and dexamethasone as ordered The nurse should assign highest priority to administering an antiemetic, such as metoclopramide, and an anti-inflammatory agent, such as dexamethasone, because it may reduce the severity of chemotherapy-induced nausea and vomiting. This intervention, in turn, helps prevent dehydration, a common complication of chemotherapy. Serving small portions of bland food, encouraging rhythmic breathing exercises, and withholding fluids for the first 4 to 6 hours are less likely to achieve this outcome.

The nurse should teach the patient who is being radiated about protecting his skin and oral mucosa. An important teaching point would be to tell the patient to: a) Use an ointment, after treatment, to decrease the feeling of burning, which may last for several hours. b) Use an approved emollient 2 hours before the radiation to give the skin time to absorb the medication and provide a shield for damage. c) Apply a small ice compress to the treated area afterward to decrease localized redness, post-radiation. d) Cleanse the skin with a mild soap, using his fingertips, not a rough wash cloth.

d) Cleanse the skin with a mild soap, using his fingertips, not a rough wash cloth. The patient should cleanse himself with a mild soap using his fingertips rather than a wash cloth. All the other choices will irritate the skin and fail to protect it from additional injury.

A client diagnosed with acute myelocytic leukemia has been receiving chemotherapy. During the last 2 cycles of chemotherapy, the client developed severe thrombocytopenia requiring multiple platelet transfusions. The client is now scheduled to receive a third cycle. How can the nurse best detect early signs and symptoms of thrombocytopenia? a) Perform a cardiovascular assessment every 4 hours. b) Monitor daily platelet counts. c) Check the client's history for a congenital link to thrombocytopenia. d) Closely observe the client's skin for petechiae and bruising.

d) Closely observe the client's skin for petechiae and bruising. The nurse should closely observe the client's skin for petechiae and bruising. Daily laboratory testing may not reflect the client's condition as quickly as subtle changes in the client's skin. Performing a cardiovascular assessment every 4 hours and checking the clients history for a congenital link to thrombocytopenia don't help detect early signs and symptoms of thrombocytopenia.

You are an oncology nurse caring for a client who tells you that their tastes have changed. They go on to say that "meat tastes bad". What is a nursing intervention to increase protein intake for a client with taste changes? a) Suck on hard candy during treatment. b) Stay away from protein beverages. c) Encourage maximum fluid intake. d) Encourage cheese and sandwiches.

d) Encourage cheese and sandwiches. The nurse encourages the clients with taste changes to eat cheese and sandwiches. Encouraging the client to take in the maximum amount of fluids does not increase protein intake. The nurse advises the client to drink protein beverages. Sucking on hard candies during treatment does not increase protein intake.

A nurse has agreed to draft a medication teaching plan for a patient who is taking the hormonal agent, Aromasin, an aromatase inhibitor for postmenopausal women with breast cancer. The nurse knows that a major teaching point is to tell the patient to: a) Report the unexpected sign of increased appetite and weight gain. b) Report the unusual sign of nausea. c) Be alarmed if she notices fluid retention. d) Increase her intake of calcium-rich foods.

d) Increase her intake of calcium-rich foods. One of the major side effects of Aromasin is hypercalcemia and the subsequent loss of bone. Therefore, the patient needs to have periodic blood work done, have bone density tests done, and follow a diet that will supply needed calcium that is being pulled from the bone tissue.

When caring for a client who is receiving external beam radiation, which is the key point for the nurse to incorporate into the plan of care? a) The use of disposable utensils and wash cloths b) Time, distance, and shielding c) Avoid showering or washing over skin markings. d) Inspect the skin frequently.

d) Inspect the skin frequently. Inspecting the skin frequently will allow early identification and intervention of skin problems associated with external radiation therapy. The external markings should not be removed, but clients may shower and lightly wash over the skin. Time, distance, and shielding are key in the management of sealed, internal radiation therapy and not external beam radiation. The use of disposable utensils and care items would be important when caring for clients following systemic, unsealed, internal radiation therapy.

A client has received several treatments of bleomycin. It is now important for the nurse to assess a) Hand grasp b) Skin integrity c) Urine output d) Lung sounds

d) Lung sounds Bleomycin has cumulative toxic effects on lung function. Thus, it will be important to assess lung sounds.

A patient is taking vincristine, a plant alkaloid for the treatment of cancer. What system should the nurse be sure to assess for symptoms of toxicity? a) Urinary system b) Pulmonary system c) Gastrointestinal system d) Nervous system

d) Nervous system With repeated doses, the taxanes and plant alkaloids, especially vincristine, can cause cumulative peripheral nervous system damage with sensory alterations in the feet and hands.

What is the best way for the nurse to assess the nutritional status of a patient with cancer? a) Monitor daily caloric intake. b) Observe for proper wound healing. c) Assess BUN and creatinine levels. d) Weigh the patient daily.

d) Weigh the patient daily. Assessment of the patient's nutritional status is conducted at diagnosis and monitored throughout the course of treatment and follow-up. Early identification of patients at risk for problems with intake, absorption, and cachexia, particularly during the early stages of disease, can facilitate timely implementation of specifically targeted interventions that attempt to improve quality of life, treatment outcomes, and survival (Gabison et al., 2010). Current weight, weight loss, diet and medication history, patterns of anorexia, nausea and vomiting, and situations and foods that aggravate or relieve symptoms are assessed and addressed.

The nurse is teaching a client newly diagnosed with cancer about chemotherapy. The nurse tells the client he'll receive an antitumor antibiotic. The nurse knows that this type of medications is: a) cell-cycle specific in the P phase. b) cell-cycle specific in the M phase. c) cell-cycle specific in the S phase. d) cell-cycle nonspecific.

d) cell-cycle nonspecific. Antitumor antibiotics are cell-cycle nonspecific; they interfere with deoxyribonucleic acid (DNA) synthesis by binding with the DNA. They also prevent ribonucleic acid synthesis. Other cell-cycle nonspecific drugs include nitrosoureas and hormonal agents. Drugs that are cell-cycle specific in the S phase include topoisomerase I inhibitors and antimetabolites. Miotic spindle poisons are cell-cycle specific in the M phase. There isn't a drug class that's specific to the P phase.

A client is recovering from a craniotomy with tumor debulking. Which comment by the client indicates to the nurse a correct understanding of what the surgery entailed? a. "Thank goodness the tumor is contained and curable." b. "I will be glad to finally be done with treatments for this thing." c. "I am so glad the doctor was able to remove the entire tumor." d. "I guess the doctor could not remove the entire tumor."

d. "I guess the doctor could not remove the entire tumor."

A patient will be receiving radiation for 6 weeks for the treatment of breast cancer and asks the nurse why it takes so long. What is the best response by the nurse? a. "It allows time for you to cope with the treatment." b. "It will decrease the incidence of leukopenia and thrombocytopenia." c. "It is not really understood why you have to go for 6 weeks of treatment." d. "It will allow time for the repair of healthy tissue."

d. "It will allow time for the repair of healthy tissue."

A client diagnosed with acute myelocytic leukemia has been receiving chemotherapy. During the last 2 cycles of chemotherapy, the client developed severe thrombocytopenia requiring multiple platelet transfusions. The client is now scheduled to receive a third cycle. How can the nurse best detect early signs and symptoms of thrombocytopenia? a. Perform a cardiovascular assessment every 4 hours. b. Check the client's history for a congenital link to thrombocytopenia. c. Monitor daily platelet counts. d. Closely observe the client's skin for petechiae and bruising.

d. Closely observe the client's skin for petechiae and bruising.

Based on the understanding of the effects of chemotherapy, the nurse would anticipate which of the following clinical findings in a client 2 weeks post therapy? a. Change in hair color b. Elevated temperature c. Elevated white blood cells count d. Ease of bruising

d. Ease of bruising

The nurse is completing an admission assessment for a client receiving interstitial implants for prostate cancer. The nurse documents this as a. a contact mold b. systemic radiation c. external beam radiation therapy d. brachytherapy

d. brachytherapy

what does the nurse understand is the rationale for administering allopurinol for a patient receiving chemotherapy? a. it stimulates the immune system against the tumor cells b. it treats drug-related anemia c. it prevents alopecia d. it lowers serum and uric acid levels

d. it lowers serum and uric acid levels

what disadvantages of chemotherapy should the patient be informed about prior to starting the regimen? a. it attacks cancer cells during their vulnerable phase b. it functions against disseminated disease c. it causes a systemic reaction d. it targets normal body cells as well as cancer cells

d. it targets normal body cells as well as cancer cells

A serum sodium concentration lower than 115 mEq/L (115 mmol/L) is associated with a. anorexia b. weight gain c. myalgia d. seizure

d. seizure

The root cause of cancer is damage to cellular deoxyribonucleic acid (DNA) which can be caused by many factors, or carcinogens. What factors can be carcinogenic? Select all that apply. age viruses dietary substances gender environmental factors

dietary substances environmental factors viruses

The nurse is evaluating the client's risk for cancer and recommends changes when the client states she

eats red meat such as steaks or hamburgers every day

The nurse is evaluating the client's risk for cancer and recommends changes when the client states she eats red meat such as steaks or hamburgers every day works as a secretary at a medical radiation treatment center drinks 1 glass of wine at dinner each night uses the treadmill for 30 minutes on 5 days each week

eats red meat such as steaks or hamburgers every day

The nurse is evaluating the client's risk for cancer and recommends changes when the client states she a) eats red meat such as steaks or hamburgers every day b) uses the treadmill for 30 minutes on 5 days each week c) drinks 1 glass of wine at dinner each night d) works as a secretary at a medical radiation treatment center

eats red meat such as steaks or hamburgers every day

The nurse is evaluating the client's risk for cancer and recommends changes when the client states she:

eats red meat such as steaks or hamburgers every day

The nurse is evaluating the client's risk for cancer. The nurse should recommend the client change which lifestyle choice?

eats red meat such as steaks or hamburgers every day

The nurse is evaluating the client's risk for cancer and recommends changes when the client states she

eats red meat such as steaks or hamburgers every day Dietary substances such as nitrate-containing, nitrite-containing, and red meats appear to increase the risk of cancer. Exercising 30 minutes on 5 days or more is recommended for adults. Measures are taken to protect those people who work around radiation. It is OK to drink 1 glass of wine per day.

The nurse is evaluating the client's risk for cancer and recommends changes when the client states she a) uses the treadmill for 30 minutes on 5 days each week b) drinks 1 glass of wine at dinner each night c) works as a secretary at a medical radiation treatment center d) eats red meat such as steaks or hamburgers every day

eats red meat such as steaks or hamburgers every day Explanation: Dietary substances such as nitrate-containing, nitrite-containing, and red meats appear to increase the risk of cancer. Exercising 30 minutes on 5 days or more is recommended for adults. Measures are taken to protect those people who work around radiation. It is OK to drink 1 glass of wine per day.

The nurse is working with a client who has had an allohematopoietic stem cell transplant (HSCT). The nurse notices a diffuse rash and diarrhea. The nurse contacts the physician to report that the client has symptoms of

graft-versus-host disease

According to the tumor-node-metastasis (TNM) classification system, T0 means there is

graft-versus-host disease.

The nurse is working with a client who has had an allohematopoietic stem cell transplant (HSCT). The nurse notices a diffuse rash and diarrhea. The nurse contacts the physician to report that the client has symptoms of

graft-versus-host disease. Graft-versus-host disease is a major cause of morbidity and mortality in clients who have had allogeneic transplant. Clinical manifestations of the disease include diffuse rash that progresses to blistering and desquamation, and mucosal inflammation of the eyes and the entire gastrointestinal tract with subsequent diarrhea, abdominal pain, and hepatomegaly.

A benign tumor of the blood vessels is a(n)

hemangioma

Chemotherapeutic agents have which effect associated with the renal system? Hypercalcemia Hypokalemia Hypophosphatemia Increased uric acid excretion

increased uric acid excretion Chemotherapeutic agents can damage the kidneys because of their direct effects during excretion and the accumulation of end products after cell lysis. Urinary excretion of uric acid increases with the use of chemotherapeutic agents.

A nurse caring for a client who has just received chemotherapy infusion is wearing a disposable gown, gloves, and goggles for protection. The nurse knows that accidental exposure to chemotherapy agents can occur through:

inhalation of aerosols.

A patient will be receiving radiation for 6 weeks for the treatment of breast cancer and asks the nurse why it takes so long. What is the best response by the nurse? It allows time for you to cope with the treatment It will allow time for the repair of healthy tissue It will decrease the incidence of leukopenia and thrombocytopenia it is not really understood why you have to go for 6 weeks of treatment.

it will allow time for the repair of healthy tissue the total radiation dose is delivered over several weeks in daily doses called fractions. This allows healthy tissue to repair and achieves greater cell kill by exposing more cells to the radiation as they begin active cell division.

A decrease in circulating white blood cells is

leukopenia.

A decrease in circulating white blood cells is a) granulocytopenia. b) leukopenia. c) neutropenia. d) thrombocytopenia.

leukopenia.

A decrease in circulating white blood cells is

leukopenia. A decrease in circulating WBCs is referred to as leukopenia. Granulocytopenia is a decrease in neutrophils. Thrombocytopenia is a decrease in the number of platelets. Neutropenia is an abnormally low absolute neutrophil count.

A decrease in circulating white blood cells is a) leukopenia. b) granulocytopenia. c) neutropenia. d) thrombocytopenia.

leukopenia. Explanation: A decrease in circulating WBCs is referred to as leukopenia. Granulocytopenia is a decrease in neutrophils. Thrombocytopenia is a decrease in the number of platelets. Neutropenia is an abnormally low ANC

Cancer is the second leading cause of death in the United States, second only to heart disease. Half of all men and one third of all women will develop cancer during their lifetimes. Which types of cancer have the highest prevalence among both men and women? colon and skin skin and brain lung and skin lung and colon

lung and colon

A client reports dyspnea, fatigue, and having had a persistent productive cough for the last few months, which the client attributes to a bout with the flu. The nurse suspects that this client may have: lung cancer. lung abscess. pleural effusion. pleurisy.

lung cancer.

According to the TNM classification system, T0 means there is

no evidence of primary tumor.

According to the TNM classification system, T0 means there is distant metastasis. no distant metastasis. no regional lymph node metastasis. no evidence of primary tumor.

no evidence of primary tumor.

According to the TNM classification system, T0 means there is

no evidence of primary tumor. T0 means that there is no evidence of primary tumor. N0 means that there is no regional lymph node metastasis. M0 means that there is no distant metastasis. M1 means that there is distant metastasis.

A nurse is caring for a client with prostate cancer and assesses bleeding gums and hematuria. What serum indicator should the nurse relate the bleeding?

platelet count of 60,000/mm3

The oncologist advises a client with an extensive family history of breast cancer to consider a mastectomy. This procedure would be considered which type of surgery?

prophylactic

Palliation refers to

relief of symptoms of disease and promotion of comfort and quality of life. Palliation is the goal for care of clients with terminal cancer. Alopecia is the term that refers to hair loss. Metastasis is the term that refers to the spread of cancer cells from the primary tumor to distant sites. Nadir is the term that refers to the lowest point of white blood cell depression after therapy that has toxic effects on bone marrow.

A client who underwent thoracic surgery to remove a lung tumor had a chest tube placed anteriorly. The surgical team places this catheter to: remove fluid from the lungs. remove air from the pleural space. administer IV medication. ventilate the client.

remove air from the pleural space.

A serum sodium concentration lower than 115 mEq/L (115 mmol/L) is associated with anorexia. weight gain. seizure. myalgia.

seizure

A serum sodium level lower than 110 mEq/L is associated with

seizure.

A serum sodium level lower than 110 mEq/L is associated with a) seizure. b) weight gain. c) myalgia. d) anorexia,

seizure. Correct Explanation: Serum sodium levels lower than 110 mEq/L is associated with seizures, abnormal reflexes, papilledema, coma, and death. Anorexia, weight gain, and myalgia are associated with serum sodium levels lower than 120 mEq/L.

The nurse teaches the client whose surgery will result in a sigmoid colostomy that the feces expelled through the colostomy will be mushy. solid. fluid. semimushy.

solid

The nurse is preparing to assess a patient whose chart documents that the patient experienced extravasation when receiving the vesicant Vincristine on the previous shift. The documentation also notes that an antidote was administered immediately. The nurse prepares to assess for which of the following? Select all that apply. a) Tissue necrosis b) Sloughing tissue c) Effectiveness of antidote d) Active bleeding

• Sloughing tissue • Tissue necrosis • Effectiveness of antidote

Which of the following are true statements about effective radiation therapy? Select all that apply.

• Slower-growing tissues at rest (muscle) are more radioresistant. • Tumors that are well oxygenated are more sensitive to radiation. • Tumors that are small in size and dividing rapidly are more sensitive

Which type of vaccine uses the client's own cancer cells, which are killed and prepared for injection back into the client?

Autologous Autologous vaccines are made from the client's own cancer cells, which are obtained during diagnostic biopsy or surgery. Prophylactic vaccines, such as the polio vaccine, are given to prevent people from developing a disease. Therapeutic vaccines are given to kill existing cancer cells and to provide long-lasting immunity against further cancer development. Allogeneic vaccines are made from cancer cells that are obtained from other people who have a specific type of cancer.

The nurse is caring for a client undergoing an incisional biopsy. Which statement does the nurse understand to be true about an incisional biopsy?

It removes a wedge of tissue for diagnosis. The three most common biopsy methods are excisional, incisional, and needle. In an incisional biopsy, a wedge of tissue is removed from the tumor and analyzed. In an excisional biopsy, the surgeon removes the tumor and the surrounding marginal tissues. Needle aspiration biopsy involves aspirating tissue fragments through a needle guided into the cancer cells.

A client is recovering from a craniotomy with tumor debulking. Which comment by the client indicates to the nurse a correct understanding of what the surgery entailed?

"I guess the doctor could not remove the entire tumor."

Which occurs when fluid accumulates in the pericardial space and compresses the heart?

Cardiac tamponade Cardiac tamponade is an accumulation of fluid in the pericardial space. SVCS occurs when the superior vena cava is compressed or invaded by a tumor, lymph nodes are enlarged, intraluminal thrombosis obstructs venous circulation, or drainage occurs from the head, neck, arms, and thorax. SIADH is the continuous, uncontrolled release of ADH. DIC is a complex disorder of coagulation or fibrinolysis that results in thrombosis or bleeding.

According to the tumor-node-metastasis (TNM) classification system, T0 means there is

No evidence of primary tumor T0 means that there is no evidence of primary tumor. N0 means that there is no regional lymph node metastasis. M0 means that there is no distant metastasis. M1 means that there is distant metastasis.


Ensembles d'études connexes

Sections 8.1, 8.2, 8.3, 8.4 and 8.5

View Set

Comparison of sympathetic and parasympathetic effects on organ function

View Set

321 Exam 2- Oxygenation and Perfusion

View Set

Project Management Procore Certification

View Set

Chapter 4: Managing Marketing Information to Gain Customer Insights

View Set

"Chapter 41: Caring for Survivors of Violence" !

View Set

Unit 19 Current of Electricity & Unit 20 DC Circuits

View Set